Class 10 Maths

You might also like

Download as pdf or txt
Download as pdf or txt
You are on page 1of 166

STUDY MATERIAL SESSION (2022-23)

CLASS X MATHEMATICS (BASIC & STANDARD)

CHIEF PATRON
SH. B.L. MORODIA
DEPUTY COMMISSIONER

PATRON PATRON
SH. D R MEENA SH. M R RAWAL
ASSISTANT COMMISSIONER ASSISTANT COMMISSIONER

COORDINATOR
SH. R C BHURIA
PRINCIPAL, KV NO-5, JAIPUR
CONTENT PREPRATION TEAM MEMBERS

S. No. NAME OF TGTs NAME OF KV

1 SH. S C JAIN KV, No-3, JAIPUR

2 SH. M K VERMA KV, No-1, JAIPUR

3 SMT. SUNITA CHHIPA KV, No-2, JAIPUR

4 SMT. PREM SUDHA YADAV KV, No-5,JAIPUR

5 SH. DEEPAK KUMAR CHAUDHARY KV, No-4, JAIPUR

COURSE STRUCTURE
Units Unit Name Marks

I NUMBER SYSTEMS 06

II ALGEBRA 20

III COORDINATE GEOMETRY 06

IV GEOMETRY 15

V TRIGONOMETRY 12

VI MENSURATION 10

VII STATISTICS & PROBABILTY 11

TOTAL MARKS 80
INDEX
S.No CONTENT PAGE NO.
1 Syllabus (2022 – 23) 5–7
2 Chapter 1 - Real Numbers 8–9
3 Chapter 2 - Polynomials 10 - 11
4 Chapter 3 - Pair of Linear Equations in Two Variables 12 – 14
5 Chapter 4 - Quadratic Equations 15 – 16
6 Chapter 5 - Arithmetic Progressions 17 – 18
7 Chapter 6 - Triangles 19 – 20
8 Chapter 7 - Coordinate Geometry 21 – 22
9 Chapter 8 - Introduction to Trigonometry 23 – 25
10 Chapter 9 - Some Applications of Trigonometry 26 – 28
11 Chapter 10 - Circles 29 – 32
12 Chapter 12 - Areas related to Circles 33 – 34
13 Chapter 13 - Surface Areas and Volumes 35 – 38
14 Chapter 14 - Statistics 39 – 44
15 Chapter 15 - Probability 45 – 46
16 Sample Paper & Marking Scheme – 1 (Standard Mathematics) 47 – 58
17 Sample Paper & Marking Scheme – 2 (Standard Mathematics) 59 – 74
18 Sample Paper & Marking Scheme – 3 (Standard Mathematics) 75 – 86
19 Sample Paper & Marking Scheme – 4 (Standard Mathematics) 87 – 98
20 Sample Paper & Marking Scheme – 5 (Standard Mathematics) 99 – 111
21 Sample Paper & Marking Scheme – 1 (Basic Mathematics) 112 – 120
22 Sample Paper & Marking Scheme – 2 (Basic Mathematics) 121 – 133
23 Sample Paper & Marking Scheme – 3 (Basic Mathematics) 134 – 144
24 Sample Paper & Marking Scheme – 4 (Basic Mathematics) 145 – 155
25 Sample Paper & Marking Scheme – 5 (Basic Mathematics) 156 - 166
SYLLABUS
UNIT I: NUMBER SYSTEMS

1. REAL NUMBER (15) Periods


Fundamental Theorem of Arithmetic - statements after reviewing work done earlier and
after illustrating and motivating through examples, Proofs of irrationality of
UNIT II: ALGEBRA

1. POLYNOMIALS (8) Periods


Zeros of a polynomial. Relationship between zeros and coefficients of quadratic
Polynomials.

PAIR OF LINEAR EQUATIONS IN TWO VARIABLES (15) Periods


2.
Pair of linear equations in two variables and graphical method of their solution,
consistency/inconsistency.

Algebraic conditions for number of solutions. Solution of a pair of linear equations in two
variables algebraically - by substitution, by elimination. Simple situational problems.

3. QUADRATIC EQUATIONS (15) Periods


Standard form of a quadratic equation ax2 + bx + c = 0, (a ≠ 0). Solutions of quadratic
equations (only real roots) by factorization, and by using quadratic formula. Relationship
between discriminant and nature of roots.

Situational problems based on quadratic equations related to day to day activities to be


incorporated.

4. ARITHMETIC PROGRESSIONS (10) Periods


Motivation for studying Arithmetic Progression. Derivation of the nth term and sum of the
first n terms of A.P. and their application in solving daily life problems.

UNIT III: COORDINATE GEOMETRY


Coordinate Geometry (15) Periods
Review: Concepts of coordinate geometry, graphs of linear equations. Distance formula.
Section formula (internal division).
UNIT IV: GEOMETRY
1. TRIANGLES
Definitions, examples, counter examples of similar triangles.
1. (Prove) If a line is drawn parallel to one side of a triangle to intersect the other
two sides in distinct points, the other two sides are divided in the same ratio.
(10)
2. (Motivate) If a line divides two sides of a triangle in the same ratio, the line is
Periods
parallel to the third side.
3. (Motivate) If in two triangles, the corresponding angles are equal, their
corresponding sides are proportional and the triangles are similar.
4. (Motivate) If the corresponding sides of two triangles are proportional, their
corresponding angles are equal and the two triangles are similar.
5. (Motivate) If one angle of a triangle is equal to one angle of another triangle
and the sides including these angles are proportional, the two triangles are
similar.

2. CIRCLES (10) Periods

Tangent to a circle at, point of contact

1. (Prove) The tangent at any point of a circle is perpendicular to the radius


through the point of contact.
2. (Prove) The lengths of tangents drawn from an external point to a circle are
equal.

UNIT V: TRIGONOMETRY

1. INTRODUCTION TO TRIGONOMETRY

Trigonometric ratios of an acute angle of a right-angled triangle. Proof of their


existence (well defined); motivate the ratios whichever are defined at 0o and 90o.
Values of the trigonometric ratios of 30 0, 450 and 600. Relationships between the
ratios.

2. TRIGONOMETRIC IDENTITIES (15) Periods

Proof and applications of the identity sin2A + cos2A = 1. Only simple identities to be
given.
3. HEIGHTS AND DISTANCES: Angle of elevation, Angle of Depression. (10)Periods
Simple problems on heights and distances. Problems should not involve more than
two right triangles. Angles of elevation / depression should be only 30°, 45°, and
60°.
UNIT VI: MENSURATION

1. AREAS RELATED TO CIRCLES (12) Periods

Area of sectors and segments of a circle. Problems based on areas and perimeter /
circumference of the above said plane figures. (In calculating area of segment of a circle,
problems should be restricted to central angle of 60°, 90° and 120° only.

2. SURFACE AREAS AND VOLUMES (12) Periods


Surface areas and volumes of combinations of any two of the following: cubes, cuboids,
spheres, hemispheres and right circular cylinders/cones.

UNIT VII: STATISTICS AND PROBABILITY

1. STATISTICS (18) Periods


Mean, median and mode of grouped data (bimodal situation to be avoided).

2. PROBABILITY (10) Periods


Classical definition of probability. Simple problems on finding the probability of an
event.

7
CHAPTER 1
The Fundamental Theorem of Arithmetic
Every composite number can be expressed (factorised) as a product of primes, and this
factorisation is unique apart from the order in which the prime factors occur.

The prime factorisation of a natural number is unique except for the order of its factors.

Property of HCF and LCM of two positive integers ‘a’ and ‘b’:
 HCF(a,b) X LCM(a,b) = a X b
a×b
LCM(a,b) = HCF (a,b)
a×b
 HCF(a,b) =LCM (a,b)

PRIME FACTORISATION METHOD TO FIND HCF AND LCM


HCF(a,b)=Product of the smallest power of each common prime factor in the numbers.
LCM(a,b)=Product of the greatest power of each prime factor involved in the numbers.

S.NO QUESTION
1 Complete the missing entries in the following factor
tree:
(a) 42 and 21 (b) 24 and 12 (c) 7 and 3 (d) 84 and 42

2 The H.C.F. and the L.C.M. of 12, 21, 15 respectively are:


(a) 3, 140 (b) 12, 420 (c) 3, 420 (d) 420, 3 Ans. (c) 3, 420

3 The H.C.F. of smallest prime number and the smallest composite number is .......... .
(a) 1 (b) 2 (c) 4 (d) none of these

4 225 can be expressed as


(a) 52× 32 (b) 52= 5 × 32 (c) 52× 3 (d)52× 325

5 The largest number which divides 70 and 125, leaving remainders 5 and 8
respectively, is
(a) 13 (b) 65 (c) 875 (d) 1750

6 Assertion : √2 is an irrational number


Reason; 2 is the smallest prime number
(a) Both assertion (A) and reason (R) are true and reason (R) is the correct
explanation of assertion (A).
(b) Both assertion (A) and reason (R) are true but reason (R) is not the correct
explanation of assertion (A).
(c) Assertion (A) is true but reason (R) is false.
(d) Assertion (A) is false but reason (R) is true
8
7 Assertion : 𝐹𝑜𝑟 𝑎𝑛𝑦 𝑡𝑤𝑜 𝑝𝑜𝑠𝑖𝑡𝑖𝑣𝑒 𝑛𝑢𝑚𝑏𝑒𝑟𝑠 𝑎 𝑎𝑛𝑑 𝑏 ,HCF (a,b)×LCM (a,b) =a× 𝑏
Reason; HCF of two number is 4 and their product is 192 then LCM is 28
(a) Both assertion (A) and reason (R) are true and reason (R) is the correct
explanation of assertion (A).
(b) Both assertion (A) and reason (R) are true but reason (R) is not the correct
explanation of assertion (A).
(c) Assertion (A) is true but reason (R) is false.
(d) Assertion (A) is false but reason (R) is true
8 Assertion : √3 +√5is an irrational number
Reason; sum of any two irrational number is always irrational
(a) Both assertion (A) and reason (R) are true and reason (R) is the correct
explanation of assertion (A).
(b) Both assertion (A) and reason (R) are true but reason (R) is not the correct
explanation of assertion (A).
(c) Assertion (A) is true but reason (R) is false.
(d) Assertion (A) is false but reason (R) is true
SHORT ANSWER TYPE QUESTION
9 Check whether 5x 3x 11 + 11 and 5 x 7 + 7 x 3 are composite numbers and
justify.
10 Check whether 6n can end with the digit 0, where n is any natural number
11 Given that LCM(26,169) = 338, write HCF(26,169).
12 Find the HCF and LCM of 6,72 and 120 using the prime factorization method
13 A class of 20 boys and 15 girls is divided into n groups so that each group has x boys
and y girls. Find x, y and n.?
14 Prove that √2 is an irrational number.
15 Prove that √3 is an irrational number.
16 Prove that 5√2 is an irrational number.
17 Prove that 7 + √2 is an irrational number.
18 Prove that √2 + √3 is an irrational number

ANSWERS
1)A 2) C 3)B 5)A 6)B 7)C 8)C
9) composite number 10) 6n cannot end with digit zero 11)13
12) HCF=6 LCM=36 13) x=4, y=3 and n=7
Question No (14-18) correct proof

9
CHAPTER 2

S. No Questions
1 Write the zero of the polynomial f(x) = x 2 – x – 6
(a) – 3, 2 (b) – 3, – 2 (c) 3, 2 (d) 3, – 2
2 For what value of k is – 4 a zero of the polynomial f(x) = x 2 – x – (2k + 2)?
(a) 6 (b) – 6 (c) 9 (d) – 9
3 If a and b are the zeroes of a polynomial such that a + b = – 6 and ab = – 4, then
write the polynomial.
(a) x2 – 6x – 4 = 0 (b) x2 + 6x – 4 = 0 (c) x2 + 6x + 4 = 0 (d) x2 – 6x + 4 = 0
4 The zeroes of the polynomial x 2 – 3x – m(m + 3) are:
(a) m, m + 3 (b) – m, m + 3 (c) m, – (m + 3) (d) – m, – (m + 3)
5 The quadratic polynomial, the sum of whose zeroes is – 5 and their product is 6, is:
(a) x2 + 5x + 6 (b) x2 – 5x + 6 (c) x2 – 5x – 6 (d) – x2 + 5x + 6
6 ASSERTION;-2 and -3 are the zeroes of x2+5x+6
Reason; a real number K is set to be zero of P(x) if P(k) =0
(a) Both assertion (A) and reason (R) are true and reason (R) is the correct
explanation of assertion (A).
10
(b) Both assertion (A) and reason (R) are true but reason (R) is not the correct
explanation of assertion (A).
(c) Assertion (A) is true but reason (R) is false.
(d) Assertion (A) is false but reason (R) is true
7 A cubic polynomial has exactly 3 zeroes
Reason; number of zero of a polynomial P(x) is equal to its degree
(a) Both assertion (A) and reason (R) are true and reason (R) is the correct
explanation of assertion (A).
(b) Both assertion (A) and reason (R) are true but reason (R) is not the correct
explanation of assertion (A).
(c) Assertion (A) is true but reason (R) is false.
(d) Assertion (A) is false but reason (R) is true
8 There exist only one real zero of P(x) =(x-2)(x2+3)
−𝑏
Reason; sum of zeroes of quadratic polynomials ax2+bx+c is 𝑎
(a) Both assertion (A) and reason (R) are true and reason (R) is the correct
explanation of assertion (A).
(b) Both assertion (A) and reason (R) are true but reason (R) is not the correct
explanation of assertion (A).
(c) Assertion (A) is true but reason (R) is false.
(d) Assertion (A) is false but reason (R) is true
9 If the product of zeros of ax2–6x–6 is 4, find the value of a.
Hence find the sum of its zeros.
10 If zeros of x2–kx+6 are in the ratio3:2, find k.
11 If one zero of the quadratic polynomial (k2+k)x2+68x+6k is reciprocal of the
other , find k.
12 If α and β are the zeros of the polynomial x2–5x+m such that α–β=1, find m.
13 If the sum of squares of zeros of the polynomial x2–8x+k is 40, find the value of k.
14 Find the value of k such that 3x2+2kx+x–k–5 has the sum of zeros as half of their
product.
15 If α and β are zeros of y2+5y+m , find the value of m such that (α+β)2 – αβ = 24
16
If α and β are zeros of x2–x–2, find a polynomial whose zeros are (2α+1) and (2β+1)
17 If α and β are zeros of the polynomial x2+4x+3, find the polynomial whose zeros are
𝛽 𝛼
1+ 𝛼 and 1+ 𝛽
18 Obtain the zeroes of 4√3x2+5x-2√3 and verify relation between zeroes and coefficient
ANSWERS
1)D 2)C 3)B 4)B 5)A 6)A 7)B 8)B 9)a=-3/2, sum of zeroes =-4
10)-5,5 11)5 12)6 13)12 14)1 15)1 16)x2-4x-5
1 −2 √3
17) 3(3x2-16x+16) 18) 3 ,
√ 4

11
CHAPTER 3

PAIR OF LINEAR EQUATIONS IN TWO VARIABLES

ALGEBRAIC INTERPRETATION OF PAIR OF LINEARE QUATIONS IN TWO VARIABLES


The pair of linear equations represented by these lines a1x+b1y+c1=0 and a2x+b2y+c2=0

S.No. Pair oflines Compare Graphical Algebraic


the ratios representation Interpretation
a1x+b1y+c1=0 𝑎1 𝑏2 Intersecting lines Unique
1 a2x+b2y+c2=0 ≠ solution(Exactly one
𝑎2 𝑏2
solution)
a1x+b1y+c1=0 a1 b1 c1 Coincident Infinitely many
2 = =
a2x+b2y+c2=0 lines solutions
a2 b2 c2
a1x+b1y+c1=0 a1 b1 c1 Parallel lines No solution
3 = ≠
a2x+b2y+c2=0 a2 b2 c2

S.NO QUESTIONS
1 The values of x and y in 2x + 3y = 2 and x – 2y = 8 are:
(a) – 4, 2 (b) – 4, – 2 (c) 4, – 2 (d) 4, 2 Ans. (c) 4, – 2
2 The value of k for which the system of linear equations x + 2y = 3, 5x + ky + 7 = 0 is
inconsistent is:
(a) 3 (b) – 4 (c) 8 (d) 4 (c) 10
3 The pair of linear equations 2x + 3y = 4 and 3x + 4y = 9 has:
(a) infinitely many solutions (b) no solution (c) one unique solution (d) two
solutions
4 The pair of equation x + 2y + 5 = 0 and – 3x – 6y + 1 = 0 have:
(a) a unique solution (b) exactly two solutions (c) infinitely many solutions (d) no
solution
5 If a pair of linear equations is consistent, then the lines will be:
(a) parallel (b) always coincident (c) intersecting or coincident
(d) always intersecting
6 System of linear equation given by x-7y+16=0 and 7x-49y-112=0 ade dependent
Reason; Dependent system of equations can be obtained from each other by
multiplying with a suitable constant
(a) Both assertion (A) and reason (R) are true and reason (R) is the correct

12
explanation of assertion (A).
(b) Both assertion (A) and reason (R) are true but reason (R) is not the correct
explanation of assertion (A).
(c) Assertion (A) is true but reason (R) is false.
(d) Assertion (A) is false but reason (R) is true
7 Assertion; there is no value of k for which the system of equations x+2y=3 ,
5x+ky+7=0 has infinite many solution
𝑎1 𝑏1 𝑐1
Reason; system of linear equations have an infinite solutions if = =
𝑎2 𝑏2 𝑐2

(a) Both assertion (A) and reason (R) are true and reason (R) is the correct
explanation of assertion (A).
(b) Both assertion (A) and reason (R) are true but reason (R) is not the correct
explanation of assertion (A).
(c) Assertion (A) is true but reason (R) is false.
(d) Assertion (A) is false but reason (R) is true
8 Assertion; If 2x+3y=12 and 3x-2y=5 then x=3 , y=2
Reason; method of elimination involves writing y in terms of x from any one of two
equations and then putting this value of y in other equation to get value of x. Finally
substituting value of x in any one equation gives value of y
(a) Both assertion (A) and reason (R) are true and reason (R) is the correct
explanation of assertion (A).
(b) Both assertion (A) and reason (R) are true but reason (R) is not the correct
explanation of assertion (A).
(c) Assertion (A) is true but reason (R) is false.
(d) Assertion (A) is false but reason (R) is true
9 Form a pair of linear equations for: The sum of the numerator and denominator of the
fraction is 3 less than twice the denominator. If the numerator and denominator
both are decreased by 1, the numerator becomes half the denominator.
10 For what value of p the pair of linear equations (p+2)x – (2p+1)y = 3(2p–1) and
2x – 3y = 7 has a unique solution.
11 ABCDE is a pentagon with BE||CD and BC||DE, BC is perpendicular to CD. If the
perimeter of ABCDE is 21cm, find x and y.
12 Solve for x and y : 3x+2y = 11 and 2x + 3y = 4. Also find p if p = 8x+5y

13
13 For what value of k, the following system of equations will be inconsistent

kx+3y=k–3 and 12x+ky=k


14 For what values of a and b the following pair of linear equations have infinite number of
solutions?
2x + 3y = 7 , a(x + y) – b(x – y) = 3a + b – 2

15 Aman travels 600 km to his home partly by train and partly by bus. He takes 8
hours, if he travels 120 km by train and rest by bus. Further, it takes 20 minutes
longer, if he travels 200 km by train and rest by bus. Find the speeds of the train and
the bus.
16 A and B are two points 150 km apart on a highway. Two cars start with different
speeds from A and B at same time. If they move in same direction, they meet in
15hours. If they move in opposite direction, they meet in one hour. Find their
speeds
17 A boat covers 32 km upstream and 36km downstream in 7hours. Also it Covers 40km
upstream and 48km downstream in 9hours. Find the speed of boat in still water and
that of the stream.
18 The sum of the numerator and denominator of a fraction is 4 more than twice the
numerator. If the numerator and denominator are increased by 3, they are in the
ratio2:3. Determine the fraction.
19 8 Women and 12 men can complete a work in10days while 6women and 8 men can
complete the same work in 14 days. Find the time taken by one woman alone and that
one man alone to finish the work
20 Determine graphically, the vertices of the triangle formed by the lines
y = x, 3y = x and x + y = 8.

ANSWERS
1)C 2)D 3)C 4)D 5)C 6)D 7)A
8)A 9)x – y = -3, 2x - y=1 10)p not equal to 4 11)x=5,y=0
(12)x=5,y=-2,p=30 13) k=-6 14)a=5,b=1 15)=60km/hr,80km/hr
16) 80km/hr,70km/hr 17)10km/hr,2km/hr 18)5/9
19) 1 women in 14 days,1 man in 28 days 20) vertices of triangle ,(0,0),(4,4),(6,2)

14
Chapter 4
QUADRATIC EQUATIONS
An equation involving single variable with a term having highest degree 2 of variable is called
quadratic equation.
In general form, ax2 + bx + c = 0, a ≠ 0 is a quadratic equation in variable x.
SOLUTION OF A QUADRATIC EQUATION
The zeroes of the quadratic polynomial or the roots of the quadratic equation ax2 + bx + c = 0
are called the solutions of the quadratic equation. Solutions of a Quadratic Equation can be found
by using following methods:
(i) By Factorisation Method: To find the solution of a quadratic equation by factorisation
method, first represent the given equation as a product of two linear factors by splitting
the middle term or by using identities and then equate each of the factor equal to zero to
get the desired roots.
(ii) By Quadratic Formula: For a quadratic equation ax2 + bx + c = 0, we have
−𝑏±√𝑏2 −4𝑎𝑐
x= 2𝑎
NATURE OF ROOTS OF QUADRATIC EQUATION: By quadratic formula
−𝑏±√𝑏2 −4𝑎𝑐
The roots of the quadratic equation are given by x= ; where
2𝑎
(b2 – 4ac) is called discriminant of the quadratic equation and denoted by D.
The following cases arise:
i. If D = b2 – 4ac > 0, then the roots of the equation are real and distinct.
ii. If D = b2 – 4ac = 0, then roots of the equation are equal and real.
iii. If D = b2 – 4ac < 0, then there does not exist any real root.
iv. If D = b2 – 4ac > 0, and perfect square, then the roots are real, rational and unequal.
v. If D = b2– 4ac > 0 and not a perfect square, then the roots are real, irrational and unequal
QUADRATIC EQUATION WHEN THE ROOTS ARE GIVEN
The quadratic equation whose roots are a and b is given as x2 – (a +b)x +ab = 0
S.NO QUESTION
1 The quadratic equation ax2 – 4ax + 2a + 1 = 0 has repeated roots, if a =
(a) 0 (b) 1/2 (c) 2 (d) 4
3
2 The roots of the equation 2x - -𝑥 = 1are
1 3
(a) 2,−1 (b) 3 ,2 (c)-1, 2 (d) none of these
3 The two roots of a quadratic equation are 2 and – 1. The equation is
(a) x2 + 2x – 2 = 0 (b) x2 + x + 2 = 0
(c) x2 – 2x + 2 = 0 (d) x2 – x – 2 = 0
4 ax2 + bx + c = 0, a > 0, b = 0, c > 0 has
(a) two equal roots (b) one real roots (c) two distinct real roots
(d) no real roots
5 1 5
is a root of the equation x2+ kx – 9= 0, then find the value of k.
3
3 4 2
(a) (b) (c) (d) 3
4 3 3
6 Assertion: 2x2 – 4x + 3 = 0 is a quadratic equation.
Reason : All polynomials of degree n, when n is a whole number can be treated as
quadratic equation.

15
Choose the correct answer out of the following choices :
(a) Assertion and Reason both are correct statements and Reason is the correct
explanation of Assertion.
(b) Assertion and Reason both are correct statements but Reason is not the
correct explanation of Assertion.
(c) Assertion is correct statement but Reason is wrong statement.
(d) Assertion is wrong statement but Reason is correct statement
7 Assertion: 3y2 + 17y – 30 = 0 have distinct roots.
Reason: The quadratic equation ax 2 + bx + c = 0 have distinct roots (real) if D > 0.
(a) Assertion and Reason both are correct statements and Reason is the correct
explanation of Assertion.
(b) Assertion and Reason both are correct statements but Reason is not the
correct explanation of Assertion.
(c) Assertion is correct statement but Reason is wrong statement.
(d) Assertion is wrong statement but Reason is correct statement.
8 Assertion: Both the roots of the equation x2 – x + 1 = 0 are real.
Reason: The roots of the equation ax2 + bx + c = 0 are real if and only if b2 – 4ac ≥ 0
(a) Assertion and Reason both are correct statements and Reason is the correct
explanation of Assertion.
(b) Assertion and Reason both are correct statements but Reason is not the
correct explanation of Assertion.
(c) Assertion is correct statement but Reason is wrong statement.
(d) Assertion is wrong statement but Reason is correct statement
1 1 11
9 solve for x − =
𝑥+4 𝑥−7 30
𝑥−4 𝑥−6 10
10 solve for x + 𝑥−7 =
𝑥−5 3
1 1 1 1
11 solve for x = 𝑥 + 𝑎 +𝑏
𝑎+𝑏+𝑥
2
12 If x = 3and x = – 3 are roots of the quadratic equation ax2 + 7x + b = 0.
Find the value of a and b.
13 Find value of p for which the product of roots of the quadratic equation
px2 + 6x + 4p = 0 is equal to the sum of the roots.
14 A fast train takes 3 hours less than a slow train for a journey of 600 km. If the speed of
the slow train is 10 km/hr less than the fast train, find the speeds of the two trains.
15 The speed of a boat in still water is 15 km/hr. It can go 30 km upstream and return
downstream to the orignal point in 4 hrs 30 minutes. Find the speed of the stream.
16 Sum of areas of two squares is 400 cm2. If the difference of their perimeter is 16 cm.
Find the side of each square.
17 If the roots of the quadratic equation (b – c)x2 + (c – a)x + (a – b) = 0 are equal,
prove: 2b = a + c.
18 A two digit number is such that the product of its digits is 18. When 63 is subtracted
from the number, the digit interchange their places. Find the number.
ANSWERS
1)B 2)C 3)D 4)D 5)B 6)C 7)A 8)D 9) x=1, -4/3
10) x=8, 11/2 11)x=-a, -b 12)a=3, b=-6 13)p= -3/2 14)5km/hr
15)5km/hr 16)x=16m, y=12m or x=12m, y=16m 17) correct proof 18) number 92

16
Chapter 5

ARITHMETIC PROGRESSION

Standard form of an AP is given as a, a+d, a+2d, a+3d,….


Where a is the first term and d is the common difference.
nth term an of the AP with first term a and common difference d is given by an = a+ (n-1)d

nth Term from the end of an AP is given by ln=l-(n-1)d


𝑛
Sum of n terms of of an AP Sn = { 2a+(n-1)d}
2
𝑛
Sum of n terms of of an AP Sn = 2 { a+l} where l is last term of the series

Also an=Sn – Sn-1

S.NO QUESTIONS
1 If k, 2k – 1 and 2k + 1 are three consecutive terms of an A.P., then the value of k is
(a) 2 (b) 3 (c) –3 (d) 5
2 The next term of the A.P. 18, 50 95 ... is
(a) 146 (b) 128 (c) 162 (d) 200
3 The value of a30 – a20 for the A.P. 2, 7, 12, 17, ... is
(a) 100 (b) 10 (c) 50 (d) 20
4 In an A.P., if a = −10, n = 6 and an = 10, then the value of d is
(a) 0 (b) 4 (c) −4 (d) 10/3
5 If the sum of first m terms of an A.P. is 2m + 3m, then what is its second term?
2

(a) 9 (b) 10 (c) 11 (d) 12


6 Assertion : The nth term of a sequence is 3n – 2. It is an A.P.
Reason : A sequence is not an A.P. if its nth term is not a linear expression in n.
(a) Both assertion (A) and reason (R) are true and reason (R) is the correct
explanation of assertion (A).
(b) Both assertion (A) and reason (R) are true but reason (R) is not the correct
explanation of assertion (A).
(c) Assertion (A) is true but reason (R) is false.
(d) Assertion (A) is false but reason (R) is true
7 Assertion : The 10th term from the end of the A.P.7, 10, 13, ...., 184 is 163.
Reason : In an A.P. with first term a, common difference d and last term l, the nth
term from the end is l – (n – 1)d
(a) Both assertion (A) and reason (R) are true and reason (R) is the correct
explanation of assertion (A).
(b) Both assertion (A) and reason (R) are true but reason (R) is not the correct
explanation of assertion (A).
(c) Assertion (A) is true but reason (R) is false.
(d) Assertion (A) is false but reason (R) is true

17
8 Assertion : The common difference of the A.P. 19, 18, 17, .... is 1.
Reason : Let a1, a2, a3, a4, ... is an A.P. Then, common difference of this A.P. will be
the difference between any two consecutive terms, i.e., common difference (d) = a2
– a1 or a3 – a2 or a4 – a3 and so on
(a) Both assertion (A) and reason (R) are true and reason (R) is the correct
explanation of assertion (A).
(b) Both assertion (A) and reason (R) are true but reason (R) is not the correct
explanation of assertion (A).
(c) Assertion (A) is true but reason (R) is false.
(d) Assertion (A) is false but reason (R) is true
9 Find the 9th term from the end of the A.P. 5, 9, 13, ..., 185.
10 Check whether −150 is a term of the A.P. : 17, 12, 7, 2,… or not.
11 What will be the 21st term of the A.P. whose first two terms are –3 and 4?
12 If the sum of first n terms of an A.P. is given by Sn = 5n2 + 3n, then find its nth term.
13 Which term of the A.P. : 21, 42, 63, 84,... is 210
14 Determine the A.P. whose 3rd term is 5 and the 7th term is 9.
15 The sum of the first n terms of an A.P. is 5n – n2 . Find the nth term of this A.P.
16 If Sn denotes the sum of first n terms of an A.P., prove that S 30 = 3[S20 – S10]
17 If the sum of n terms of an A.P. is (pn + qn 2 ), where p and q are constants, find the
common difference
18 Jasleen saved Rs 4 during first week of the year and then increased her weekly
savings by Rs 1.75 each week. In which week, will her weekly savings be `Rs19.75?

ANSWERS

1)B 2)C 3)C 4)B 5)A 6)A 7)D 8)D 9) 153 10) not a term
11)a21=137 12)an=10n-2 13)t10=210 14)ap=3,4,5,6,7,….
15)Tn= -2n+6 16)correct proof 17)d=2q 18) 10 week

18
CHAPTER 6

Similar Triangles: Two triangles are said to be similar if their corresponding angles are equal and
their corresponding sides are proportionate.
Criteria for Similarity:
In ΔABC and ΔDEF
(i) AAA Similarity: ΔABC ~ ΔDEF when
∠A=∠D, ∠B=∠E and ∠C=∠F
(ii) SAS Similarity:
𝐴𝐵 𝐵𝐶
ΔABC ~ ΔDEF when 𝐷𝐸 = 𝐸𝐹 and ∠B = ∠E
(iii) SSS Similarity: ΔABC ~ ΔDEF if 𝐴𝐵 𝐴𝐶 𝐵𝐶
= =
𝐷𝐸 𝐷𝐹 𝐸𝐹

Basic Proportionality Theorem: If a line is drawn parallel to one side of a triangle to intersect the other
two sides in distinct points, the other two sides are divided in the same ratio.

S.NO QUESTIONS
1 If in two triangles DEF and PQR, ∠D = ∠Q and ∠R = ∠E, then which of the following is not
true?
(a) EF/PR = DF/PQ (b) DE/PQ = EF/RP (c) DE/QR = DF/PQ (d) EF/RP = DE/QR
2 If ΔABC ~ ΔEDF and ΔABC is not similar to ΔDEF, then which of the following is not true?
(a) BC . EF = AC. FD (b) AB . EF = AC. DE (c) BC . DE = AB. EF d) BC . DE = AB. FD
3 D and E are respectively the points on the sides AB and AC of a triangle ABC such that AD =
2 cm, BD = 3 cm, BC = 7.5 cm and DE || BC. Then, length of DE (in cm) is-
(a) 2.5 (b) 3 (c) 5 (d) 6
4 A vertical pole of length 6 m casts a shadow 4 m long on the ground and at the same time a
tower casts a shadow 28 m long. Find the height of the tower.
(a) 42 (b) 32 (c) 5 (d)16
5
In this figure if DEIIBC. Then find the value of x
(a) 10 (b) 11 (c) 12 (d)13

.6 In ∆ABC, D and E are mid-points of AC and BC respectively such that DE


|| AB. If AD = 2x,
BE = 2x – 1, CD = x + 1 and CE = x – 1, then find the value of x

7 Students of a school decided to participate in ‘Save girl child’ campaign.


They decided to decorate a triangular path as shown. If AB = AC and BC 2
= AC × CD, then prove that BD = BC.

19
8 The perimeters of two similar triangles are 25 cm and 15 cm respectively. If one side of the
first triangle is 9 cm, find the corresponding side of the second triangle.

9 In ∆ABC, D and E are points on the sides of AB and AC such that DE || BC. If AD = 2.5 cm, BD
= 3 cm, AE = 3.75 cm, find the length of AC.
10 Prove that a line drawn through the mid-point of one side of a triangle parallel to another
side bisects the third side.
11 ABCD is a trapezium in which AB || DC and its diagonals intersect each other at the point O.
𝐴𝑂 𝐶𝑂
Show that = 𝐷𝑂
𝐵𝑂
12 Diagonals AC and BD of a trapezium ABCD with AB || DC intersect each other at the point O.
Using a similarity criterion for two triangles, show that OA/OC=OB/OD∙
13 In the given figure, altitudes AD and CE of ∆ABC intersect
each other at the point P. Show that:
(i) ∆AEP ~ ∆CDP
(ii) ∆ABD ~ ∆CBE
(iii) ∆AEP ~ ∆ADB
(iv) ∆PDC ~ ∆BEC
14 In the given figure, ABC and AMP are two right triangles, right angled at B and M
respectively. Prove that:
(i) ∆ABC~ ∆AMP
𝐶𝐴 𝐵𝐶
(ii) =𝑀𝑃
𝑃𝐴

15 CD and GH are respectively the bisectors of ∠ACB and ∠EGF such that D and H lie on sides
AB and FE of ∆ABC and ∆EFG respectively. If ∆ABC ~ ∆FEG, show that
𝐶𝐷 𝐴𝐶
(i) ∆DCB~ ∆HGE (ii) =𝐹𝐺
𝐺𝐻
16 Sides AB and BC and median AD of a triangle ABC are respectively proportional to sides PQ
and QR and median PM of ∆PQR. Show that ∆ABC ~ ∆PQR.
17 Sides AB and AC and median AD of a triangle ABC are respectively proportional to sides PQ
and PR and median PM of another triangle PQR. Show that ∆ABC ~ ∆PQR.
18 State and proof Basic proportionality theorem.

ANSWERS
1)B 2)C 3)B 4)A 5)B 6)x=1/3
7)correct proof 8)DE=5.4cm 9)AC=8.25cm

Q(10-18) correct proof

20
CHAPTER 7
COORDINATE GEOMETRY

The system of geometry where the position of points on the plane is described using an ordered
pair of numbers.
Distance Formula: Distance between two given points A(x1,y1) And B(x 2,y2) =
√(𝑥2 − 𝑥1 )2 − (𝑦2 − 𝑦1 )2

Section formula
The coordinates of the point P(x, y) which divides the line segment joining the points A(x1, y1) and
B(x2, y2) internally in the ratio m1 : m2 are

𝑚1 𝑥2 +𝑚2 𝑥1 𝑚1 𝑦2 +𝑚2 𝑦1
𝑥= 𝑦=
𝑚1 +𝑚2 𝑚1 +𝑚2

The coordinates of point M(x,y) which is the midpoint of point ( x1, y1) and (x2, y2) are

x1 + x2 y1 + y2
X= , y=
2 2

S.NO QUESTION
1 The distance of the point (2,3) from the x-axis is (a) 2 (b) 3 (c) 1 (d) 5
2 The distance of the point P(-6,8) from the origin is (a) 8 (b) 2√7 (c) 10 (d) 6
3 The distance between the points A(0,6) and B(0,-2) is (a) 6 (b) 8 (c) 4 (d) 2
4 The distance between the points (0,5) and (-5,0) is (a) 5 (b) 5√2 (c) 2√5 (d) 10
5 If the distance between the points (2,-2) and (-1,y) is 5, then the value of y is
(a) -2 (b) 2 (c) -1 (d) 1
6 If the distance between the points (4,p) and (1,0) is 5, then value of p is
(a) 4 only (b) ±4 (c) -4 only (d) 0
7 The mid-point of the line segment joining the points A(-2,8) and B(- 6,-4) is
(a) (-4,-6) (b) (2,6) (c)(-4,2) (d) (4,2)
8 The values of y, for which the distance between the points P(2,-3) and Q(10,y) is 10
units, are (a) 9,6 (b) 3,-9 (c) -3,9 (d) 9,-6
9 ABCD is a rectangle whose three vertices are A(0,3), B(0,0) and C(5,0). Then the
length of its diagonal is (a) 5 (b) 3 (c) √34 (d) 4
10 Find the value of 𝑥 for which the distance between the points P(4,- 5) and Q(12,𝑥) is
10 units.
11 Find a point on x-axis which is equidistant from the points (7,6) and (-3,4).
12 Find a point on y-axis which is equidistant from the points (-5,2) and (9,-2).
13 Find a relation between x and y such that the point P(x, y) is equidistant from the
points A(2,5) and B(-3,7).
14 Find the ratio in which the point (x,1) divides the line segment joining the points
(-3,5) and (2,-5). Also find the value of x
15 Find the coordinates of the point which divides the line segment joining the points
(4,-3) and (8,5) in the ratio 3:1.

21
16 The coordinates of the mid-point of the line segment joining the points (3p,4) and
(-2,2q) are (5,p).Find the values of p and q.
17 If the mid-points of the line segment joining the points A(3,4) and B(k,6) is P(x, y)
and x+y-10=0, find the value of k
18 The mid-points of the sides of a triangle are (3,4), (4,6) and (5,7). Find the
coordinates of vertices of the triangle.
19 A person is riding his bike on a straight road towards East from his college to city A
and then to city B. At some point in between city A and city B, he suddenly realises
that there is not enough petrol for the journey. Also, there is no petrol pump on the
road between these two cities.
Based on the above information, answer the following questions

(I)The value of y is equal to


(II)The value of x is equal to
(III)If M is any point exactly in between city A and city B, then coordinates of M are
OR
The ratio in which A divides the line segment joining the points O and M is (a) 1:2
(b) 2:1

ANSWERS
1)B 2)C 3)B 4)C 5)B 6)B 7)B 8)B 9)C
10(1,-11) 11)(3,0) 12) P=(0,-7) 13)10x-4y+20=0 14)k=3/2,or x=0
15)7,3 16)t=4,q=2 17)k=7 18) A(6,9) B(4,5) C(2,3)
19)(i) y=2 (ii) x=8 (iii) 5,5 or 2,3

22
Chapter 8
Introduction to Trigonometry
Important points
Sometimes we observe imaginary triangle in nature,
e.g. if we look at the top of a tower a right angle can be imagined. As shown in figure

We need to find height BC or distance AB or AC.


These all can be found by using mathematical techniques which comes under
a branch of mathematics called Trigonometry.

Consider a right angled triangle ABC right angled at B.

Fig 1 Fig2

Observing the above two triangles we see that one side i.e. hypotenuse (longest side of right triangle)
is fixed it is opposite to right angle, but other sides varies in respect of angle under consideration.
Here it is to note that we write:
Side opposite to given angle as PERPENDICULAR (P), Side adjacent to given angle as BASE (B)
And the longest side HYPOTENUSE (H).

TRIGONOMETRIC RATIOS
Let’s define certain ratios involving sides of right triangles and call them TRIGONOMETRIC RATIOS

So we have,

23
sin  =P/H cos  =B/H tan  =P/B
cosec  =H/P sec  =H/B cot  =B/P

TRIGNOMETRIC TABLE
0 30 45 60 90
Sin 0 1/21/√2 √3/2 1
Cos 1 √3/2 1/√2 1/2 0
Tan 0 1/√3 1 √3 n.d
Cot n.d √3 1 1/√3 0
Sec 1 2/√3 √2 2 n.d
cosec n.d 2 √2 2/√3 1

S.NO QUESTION
1 If x=2 sin2θ
and y=2cos2 θ+1 then x + y is equal to
(a) 3 (b) 2 (c) 1 (d) 0
2 If tan A = 4/3 , then the value of cosC is
(a) 3/4 (b)4/5 (c) 1 (d) none of these
3 In ∆OPQ, right-angled at P, OP = 7 cm and OQ – PQ = 1 cm, then the values of sin Q.
(a)7/25 (b) 24/25 (c) 1 (d) none of these
4 Given 15 cot A = 8, then sin C =
(a) 0 (b) 8/17 (c) 1 (d) none of these
5 In ∆PQR, right-angled at Q, PR + QR = 25 cm and PQ = 5 cm, then the value of sin P
is (a) 5/13 (b) 12/13 (c) 13/12 (d) 0
6 Q.1. Assertion: The value of sin600 cos300 + sin300 cos600 is 1
Reason: sin900=1 and cos900=0
(a) Both assertion (A) and reason (R) are true and reason (R) is the correct
explanation of assertion (A).
(b) Both assertion (A) and reason (R) are true but reason (R) is not the correct
explanation of assertion (A).
(c) Assertion (A) is true but reason (R) is false.
(d) Assertion (A) is false but reason (R) is true
7 Assertion: In a right ΔABC, right angled at B, if tanA=1, then 2sinA.cosA=1
Reason: cosecA is the abbreviation used for cosecant of angle A.
(a) Both assertion (A) and reason (R) are true and reason (R) is the correct
explanation of assertion (A).
(b) Both assertion (A) and reason (R) are true but reason (R) is not the correct
explanation of assertion (A).
(c) Assertion (A) is true but reason (R) is false.
(d) Assertion (A) is false but reason (R) is true
8 Assertion: sin(A+B)=sinA + sinB
24
Reason: For any value of θ, 1+tan2θ = sec2θ
(a) Both assertion (A) and reason (R) are true and reason (R) is the correct
explanation of assertion (A).
(b) Both assertion (A) and reason (R) are true but reason (R) is not the correct
explanation of assertion (A).
(c) Assertion (A) is true but reason (R) is false.
(d) Assertion (A) is false but reason (R) is true
9 If 3 cot A = 4, find the value of (Cosec2 A + 1)/( Cosec A2 – 1)
10 If tan (3x – 15°) = 1 then find the value of x.
1
11 If sec θ = x + 4𝑥 , prove that sec θ + tan θ = 2x or 1/2x
12 tan θ – cot θ
Prove that :. = tan2 θ- cot2 θ
sinθ cos θ
13 Sec θ + tan θ – 1 𝑐𝑜𝑠𝜃
Prove that =1−𝑠𝑖𝑛𝜃
tan θ – Sec θ +1
14 Prove that: sin θ (1 + tan θ) + cos θ (1 + cot θ) = sec θ + cosec θ.
15 Prove that: sin6 θ + cos6 θ = 1 – 3 sin2 θ cos2 θ.
16 Evaluate sin 60° cos 30° + sin 30° cos 60°
17 Evaluate 2 tan2 45° + cos2 30° – sin2 60°
18 sin 30° + tan 45° – cosec 60°
Evaluate
sec 30° + cos 60° + cot 45°

ANSWER
1)A 2)B 3)A 4)B 5)B 6)B 7)B
8)D 9)13/7 10)20
Q (11to15) correct proof
43−24√3
16)1 17)2 18) 11

25
Chapter 9
Application of Trigonometry
Important Points: -
Trigonometry can be used to measure the height of a building or trees, mountains etc. It is the study
of relationship between the ratios of the right-angled triangle’s sides and its angles. Trigonometry is
being used for finding the heights and distances of various objects without measuring them. In
solving problems of heights and distances two types of angles are involved:
1. The angle of Elevation
2. The angle of Depression
Before knowing these angles, it is necessary to know about the following terms.
 Horizontal Plane: A plane parallel to the earth is called the Horizontal Plane
 Horizontal Line: A line drawn parallel to horizontal plane is called a horizontal line.
Example of:
Angle of Elevation

Angle of Depression

Use of Right-angled Triangle in Trigonometry: -


∆𝐴𝑀𝑃 is right angled at M, PM is perpendicular, AM is base and AP is hypotenuse and ∠𝑃𝐴𝑀 = 𝜃

Most commonly used trigonometric angles are 𝟑𝟎𝟎 , 𝟒𝟓𝟎 and 𝟔𝟎𝟎

26
S.NO QUESTIONS
1 The length of the shadow of a tower on the plane ground is √3 times the height of the
tower. The angle of elevation of sun is : (a) 45° (b) 30° (c) 60° (d) 90°
2 The tops of the poles of height 16 m and 10 m are connected by a wire of length l
metres. If the wire makes an angle of 30° with the horizontal, the l = (a) 26 m (b) 16
m (c) 12 cm (d) 10 m
3 A pole of height 6 m casts a shadow 2 √3 m long on the ground. the angle of elevation
of the sun is (a) 30° (b) 60° (c) 45° (d) 90
4 A ladder leaning aginast a wall makes an angle of 60° with the horizontal. If the foot
of the ladder is 2.5 m away from the wall, then the length of the ladder is —
(a) 3 m (b) 4 m (c) 5 m (d) 6 m
5 If a tower is 30 m hight, costs a shadow 10 √3 m long on the ground, then the angle of
elevation of the sun is:
(a) 30° (b) 45° (c) 60° (d) 90°
6 A tower is 50 m high. When the sun’s altitude is 45° then what will be the length of its
shadow?
50
7 The length of shadow of a pole 50 m high is m. find the sun’s altitude.
3
8 Find the angle of elevation of a point which is at a distance of 30 m from the base of a
tower 10 √3 m high.
9 A kite is flying at a height of 50√3m from the horizontal. It is attached with a string
and makes an angle 60° with the horizontal. Find the length of the string.
10 The upper part of a tree broken over by the wind makes an angle of 30° with the
ground and the distance of the root from the point where the top touches the ground
is 25 m. What was the total height of the tree?
11 A man standing on the deck of a ship, 10 m above the water level observes the angle
of elevation of the top of a hill as 60° and angle of depression the bottom of a hill as
30°. Find the distance of the hill from the ship and height of the hill.
12 A bird is sitting on the top of a tree, which is 80 m high. The angle of elevation of the
bird, from a point on the ground is 45°. The bird flies away from the point of
observation horizontally and remains at a constant height. After 2 seconds, the angle
of elevation of the bird from the point of observation becomes 30°. Find the speed of
flying of the bird.
13 The shadow of a tower standing on a level ground is found to be 40 m longer when
the Sun’s altitude is 30° than when it is 60°. Find the height of the tower
14 The angles of depression of the top and the bottom of an 8 m tall building from the
top of a multi-storeyed building are 30° and 45°, respectively. Find the height of the
multi-storeyed building and the distance between the two buildings.
15 From a point on a bridge across a river, the angles of depression of the banks on
opposite sides of the river are 30° and 45°, respectively. If the bridge is at a height of
3 m from the banks, find the width of the river.
16 From the top of a 7 m high building, the angle of elevation of the top of a cable tower
is 60° and the angle of depression of its foot is 45°. Determine the height of the tower

27
17 A straight highway leads to the foot of a tower. A man standing at the top of the tower
observes a car at an angle of depression of 30°, which is approaching the foot of the
tower with a uniform speed. Six seconds later, the angle of depression of the car is
found to be 60°. Find the time taken by the car to reach the foot of the tower from this
point
18 A hot air balloon is a type of aircraft. It is lifted by heating the air inside the balloon,
usually with fire. Hot air weighs less than the same volume of cold air (it is less
dense), which means that hot air will rise up or float when there is cold air around it,
just like a bubble of air in a pot of water. The greater the difference between the hot
and the cold, the greater the difference in density, and the stronger the balloon will

pull up.
Lakshman is riding on a hot air balloon. After reaching at height x at point P , he spots
a lorry parked at B on the ground at an angle of depression of 30 c. The balloon rises
further by 50 metres at point Q and now he spots the same lorry at an angle of
depression of 45c and a car parked at C at an angle of depression of 30c.
(i) What is the relation between the height x of the balloon at point P and distance d
between point A and B ?
(ii) When balloon rises further 50 metres, then what is the relation between new
height y and d ? (iii) What is the new height of the balloon at point Q ?
Or
What is the distance AB on the ground ?

ANSWER
1)B 2)C 3)B 4)C 5)C 6)15m 7)60O
8)30O 9)100m 10)43.3m 11)40m,17.32m 12)29.28m
13)h=23√3 m 14)4(3+√3) 15)3(√3+1) 16)7(√3+1)
17)3sec 18)case study(i) √3AP (ii) AB=AQ (iii) AQ=25(√3+3) OR 25(√3+3)

28
Chapter 10
Circles

PREREQUISITES TERMONIOLOGY:

1) RADIUS: The distance from the centre to any point on the


surface of a circle is called “Radius”.
2) SECANT: A secant to a circle is a line that cuts the circle at
two distinct points.
3) CHORD: A chord is a line segment whose end points lie on
the circle itself. Diameter is the longest chord in a circle.
4) TANGENT:
A tangent to a circle is a line that touches the circle at exactly one point. For every point on the
circle, there is a unique tangent passing through it.
The point where the tangent touches the circle is called “Point of contact”.
Key points:
 No tangent can be drawn to a circle which passes through a point that lies inside it.
 When a point of tangency lies on the circle, there is exactly one tangent to a circle that passes
through it.
 When the point lies outside of the circle, there are accurately two tangents to a circle through
it.

THEOREMS:
1) The perpendicular from the centre of the circle to a chord bisects the chord.
2) The angle subtended by an arc at the centre of the circle is double the angle
subtended by it at any point on the remaining part of the circle.

3) Angles in the same segment of the circle are always equal.


4) Angle formed in a semicircle is always 90.

5) The sum of all the angles of a quadrilateral is 360.

29
IMPORTANT THEOREMS (WITH PROOF):

Theorem 1: The tangent at any point of a circle is perpendicular to


the radius through the point of contact.

Given: A circle C (O, r) and a tangent XY at point “P”.


To prove: OP is perpendicular to tangent XY.
Construction: A point Q on the tangent line XY, other than P. Join
the points OQ.
Proof:
Point Q should lie outside the circle. Because if point Q lies inside the circle, XY will not be a tangent
to the circle and XY would become a secant of a circle.
Now, OQ should be greater than the radius of the circle OP as it lies outside the circle.
Thus, OQ > OP
As, this condition is obeyed for all points on line XY except P, OP should be the shortest of all
distances from the centre of the circle “O” to the points of line XY.
Therefore, OP is perpendicular to XY.
Hence, proved.

Converse of Theorem 1:
A line drawn through the end of a radius and perpendicular to it is a tangent to the circle.

Theorem 2: The lengths of tangents drawn from an external point to a circle are equal.
Given: A circle C (O, r) and two tangents say PQ and PR from an external point P.
To prove: PQ = PR
Construction: Join OQ, OR and OP.
Proof: In ∆ OQP and ∆ ORP,
OQ = OR = r (radii of the same circle)
OP = OP (common)
∠OQP = ∠ORP = 90
(The angles formed between the tangents and radii are right
angles.)
Hence, using the RHS congruence rule,
∆ OQP ≅∆ ORP
Thus, PQ = PR (Using CPCT)
Hence the theorem is proved.
S.No QUESTIONS
1 Two balls of equal size are touching each other externally at point C and AB is common
tangent to the balls. Then ∠ ACB= (a) 600 (b) 450 (c) 300 (d) 900
2 Radha and Shyama were arguing that how many parallel tangents can a circle have? Can
you help them ?
(a) 1 (b) 2 (c) infinite (d) none of these
3 Three friends Ram, Shyam and Rahim are playing in a
triangular park in which there is a circular rose garden as
shown in the fig. Three friends are standing at points A,B and
C respectively. By the information given in the figure, can you
calculate perimeter of the park ?
(a) 30 cm (b) 60cm (c) 45cm (d) 15cm
30
4 If four sides of the quadrilateral ABCD are tangents to a circle , then
(a) AC+AD=BD+CD (c) AB+CD=BC+AD (b) AB+CD=AC+BC (d)AC+AD=BC+DB
5 AP and AQ are tangents drawn from a point A to a circle with centre O and radius 9 cm.
If OA=15 cm, then AP+AQ=
(a) 12cm (b) 18cm (c) 24cm (d) 36cm
6 If common tangents AB and CD of two wheels with
centre O and O’ intersect at E, then find OEO’=?
(a) a triangle (b) a line
(c) an arc (d) none of these

7 PQ and PR are two tangents from P to a circle with


centreA . If ∠QAR=1300 , find ∠QPR=?
(a) 400 (b) 500 (c)600 (d)20O

8 In two concentric circles, if length of one chord AB touching inner


circle is 12cm then find the length of chord CD ?
(a) 10cm (b) 15cm (c) 12cm (d) 6cm

9 The length of the tangent from a point which is at a distance of 10cm from the centre of
the circle having radius 6cm is ? (a) 8cm (b) 10cm (c) 4cm (d)16cm
10 If AB= 14cm and PE=5cm, then AE=?
(a) 7cm (b) 8cm (c) 19cm (d)9cm

11 Prove that The tangent at any point of a circle is perpendicular to the radius through the
point of contact
12 Prove that The lengths of tangents drawn from an external point to a circle are equal.
13 Two tangents TP and TQ are drawn to a circle with centre O from an external point T.
Prove that ∠ PTQ = 2 ∠ OPQ.
14 PQ is a chord of length 8 cm of a circle of radius 5 cm. The
tangents at P and Q intersect at a point T (see Fig). Find the length
TP

15 Prove that the tangents drawn at the ends of a diameter of a circle are parallel

31
16 A quadrilateral ABCD is drawn to circumscribe a circle (see Fig).
Prove that AB + CD = AD + BC

17 Prove that the parallelogram circumscribing a circle is a rhombus


18 In Fig. , XY and X′Y′ are two parallel tangents to a circle with
centre O and another tangent AB with point of contact C
intersecting XY at A and X′Y′ at B. Prove that ∠ AOB = 90°

19 A triangle ABC is drawn to circumscribe a circle of radius 4 cm


such that the segments BD and DC into which BC is divided by
the point of contact D are of lengths 8 cm and 6 cm respectively
(see Fig.). Find the sides AB and AC

20 Prove that opposite sides of a quadrilateral circumscribing a circle subtend


supplementary angles at the centre of the circle

ANSWERS
1)D 2)B 3)A 4)C 5)C 6)B 7)B
8)C 9)A 10)D Q(11-13)correct proof 14)20/3
Q(15-18) correct proof
19)AB=15cm,AC=13cm 20) correct proof

32
Chapter 12
Areas Related to Circles

IMPORTANT FORMULAS & CONCEPTS


Perimeter and Area of a Circle
1. Perimeter/circumference of a circle = 𝜋 × diameter
= 𝜋 × 2𝑟 (where 𝑟 is the radius of the circle)
= 2𝜋𝑟
2 22
2. Area of a circle= 𝜋𝑟 , where 𝜋 = 7
Areas of Sector and Segment of a Circle
𝜃
1. Area of the sector of angle 𝜃 = 360∘ × 𝜋𝑟 2 , where 𝑟 is the radius of the circle
and 𝜃 the angle of the sector in degrees
𝜃
2. Length of an arc of a sector of angle 𝜃 = 360 ∘ × 2𝜋𝑟, where 𝑟 is the radius of
the circle and 𝜃 the angle of the sector in degrees
3. Area of the segment APB = Area of the sector OAPB - Area of △ OAB
𝜃 𝜃 𝑟2
= ∘
× 𝜋𝑟 2 − area of △ OAB = ∘
× 𝜋𝑟 2 − 𝑠𝑖𝑛𝜃
360 360 2
4. Area of the major sector OAQB = 𝜋𝑟 2 - Area of the minor sector OAPB
5. Area of major segment AQB = 𝜋𝑟 2 - Area of the minor segment APB
Area of segment of a circle = Area of the corresponding sector - Area of the corresponding triangle

S No QUESTIONS
1 If the difference between the circumference and the radius of a circle is 37 cm, then
using π = 22/7 the radius of the circle (in cm) is:
(a) 154 (b) 44 (c) 14 (d) 7
2 In the given figure AOB is a sector of circle of radius 10.5 cm. The
perimeter of the sector (in cm) is:
(a) 32 (b) 21 (c) 11 (d) 35

3 The area of the sector of a circle of radius 6 cm whose central angle is 30°.
(a) 9.42 cm2 (b) 7.42 cm2 (c) 8.42 cm2 (d) 6.42 cm2
4 If π is taken as 22/7 the distance (in meters) covered by a wheel of diameter 35 cm, in
one revolution is:
(a) 2.2 (b) 1.1 (c) 9.625 (d) 96.25
5 The circumference of a circular field is 528 cm. Then the radius will be:
(a) 84 cm (b) 64 cm (c) 55 cm (d) 45 cm
6 If the circumference and the area of a circle are numerically equal, then diameter of the
circle is:
(a) 5 (b) 2p (c) 2 (d) 4
7 The diameter of the driving wheel of a bus is 140 cm. How many revolutions per
minute must the wheel make in order to keep a speed of 66 kmph?
(a) 200 (b) 240 (c) 250 (d) 260
8 A wheel makes 1000 revolutions in covering a distance of 88 km. Find the radius of the
wheel:
(a) 11 m (b) 14 m (c) 12 m (d) 10 m
33
9 The inner circumference of a circular race track, 14 m wide, is 440 m. Find the radius of
the outer circle:
(a) 85 m (b) 82 m (c) 80 m (d) 84 m
10 If the radius of a circle is doubled, its area is increased by:
(a) 100% (b) 200% (c) 300% (d) 400%
11 Area of a sector of a circle of radius 36 cm is 54π cm 2 . Find the length of the
corresponding arc of the sector.
12 The length of the minute hand of a clock is 5 cm. Find the area swept by the minute
hand during the time period 6:05 am to 6:40 am.
13 In a circle with centre O and radius 4 cm, and of angle 30°. Find the area of minor
sector and major sector AOB. (π = 3.14)
14 Find the area of the largest triangle that can be inscribed in a semicircle of radius r unit.
15 The cost of fencing a circular field at the rate of Rs 24 per metre is Rs 5280. The field is
to be ploughed at the rate of Rs 0.50 per m2 . Find the cost of ploughing the field
16 A car has two wipers which do not overlap. Each wiper has a blade of length 25 cm
sweeping through an angle of 115°. Find the total area cleaned at each sweep of the
blades.
17 A horse is tied to a peg at one corner of a square shaped grass field of side 15 m by
means of a 5 m long rope . Find (i) the area of that part of the field in which the horse
can graze. (ii) the increase in the grazing area if the rope were 10 m long instead of 5
m.
18 An umbrella has 8 ribs which are equally spaced .Assuming umbrella to be a flat circle
of radius 45 cm, find the area between the two consecutive ribs of the umbrella

ANSWERS
1)D 2)A 3)A 4)B 5)A 6)D 7)C
5
8)B 9)D 10)C 11)3π 12) 456cm2

13) area of minior sector=4.19cm2, area of major sector=46.1cm2 14)r2unit


15)1925 16)1254.96cm2 17)area of quadrant=19.625 m2,increase in area=58.875m2
22275
18) cm2
28

34
Chapter 13
Surface areas and Volumes

Shape Parameters Surface Area (Square Volume (Cubic


units) units)
Length = l TSA = 2(lb + bh + lh) V=l×b×h
Breadth = b LSA = 2h(l + b)
Cuboid Height = h
Cube Length = Breadth = TSA = 6l2 V = l3
Height = l LSA = 4l2

Cylinder Radius = r CSA = 2π × r × h V = πr2h


Height = h TSA = 2πr(h + r)

Radius = r CSA = πrl V = (1/3)πr2h


Height = h TSA = πr(l + r)
Slant Height = l
Cone
Sphere Radius = r CSA = TSA = 4πr2 V = (4/3)πr3

Hemisphere Radius = r CSA = 2πr2 V = (2/3)πr3


TSA = 3πr2

Formulas to be used in calculating combined shapes surface area and volume


S.n Shape Combinatio Surface area Surface area Volume Volume formula
o n of figures formula
1 Cube + Total surface 2(lb+bh+lh) Volume of l×b×h
cube=cuboi area cuboid
d of cuboid
2 Cube + Total surface 2(lb+bh+lh) Volume of l×b×h
cuboid= area of cuboid cuboid
cuboid
3 Cylinder + CSA of the (2πrh+2πr2) Volume of Πr2h+ (2/3)πr3
hemisphere cylindrical =2πr(h + r) the
part + CSA cylindrical
of part +
35
fixed on hemispheric volume of
ground al part hemispheric
al part
4 2x CSA of the 2x2πr2+2πr2 Volume of
hemisphere cylindrical =6πr2 the πr2h+
+ cylinder part + CSA cylindrical 2x(2/3)πr3
of part +
hemispheric volume of
al part two
hemispheric
al part
5 Cone + CSA of the Πrl + 2πrh Volume of Πr2h+⅓πr2h
cylinder cylindrical =πr(l + 2h) the
part + CSA cylindrical
fixed on ground of part of +
hemispheric volume
al part hemispheric
al part
6 2x cone + 2x CSA of 2πrl+2πrh Volume of Πr2h+[2×(⅓πr
cylinder cone + CSA =2πr(l+ h) cylinder + 2h )]

of cylinder 2x volume of
cone +
7 Cone + Csa of Πrl+2πr2 Volume of ⅓πr2h +
hemisphere conical part the conical (2/3)πr3
s + csa of part +
hemispheric volume of
al part hemispheric
al part
8 Cone + 2xcsa of 2x πrl 2x volume of 2x ⅓πr2h
cone = 2x conical part conical part
cone
9 Cone + Csa of Πrl+2πrh+2π Volume of ⅓πr2h
cylinder + conical part r2 conical part +πr2h+(2/3)πr
hemisphere + csa of + volume of 3

cylinder cylinder
+csa of +volume of
hemispheric hemispheric
al part al part

S.NO QUESTIONS
1 The volume of a cube is 2744 cm3 . Its surface area is
(a) 196 cm2 (b) 1176 cm2 (c) 784 cm2 (d) 588 cm2
2 The ratio of the total surface area to the lateral surface area of a cylinder with base radius
80 cm and height 20 cm is (a) 1 : 2 (b) 2 : 1 (c) 3 : 1 (d) 5 : 1
3 The height of a cylinder is 14 cm and its curved surface area is 264 cm 2 . The volume of
the cylinder is (a) 296 cm3 (b) 396 cm3 (c) 369 cm3 (d) 503 cm3
4 The ratio of the volumes of two spheres is 8 : 27. The ratio between their surface areas is
(a) 2 : 3 (b) 4 : 27 (c) 8 : 9 (d) 4 : 9
36
5 The radii of the base of a cylinder and a cone are in the ratio 3 : 4 and their heights are in
the ratio 2 : 3, then ratio of their volumes is (a) 9 : 8 (b) 9 : 4 (c) 3 : 1 (d) 27 : 64
6 The areas of three adjacent faces of a rectangular block are in the ratio of 2 : 3 : 4 and its
volume is 9000 cu. cm, then the length of the shortest side is (a) 10 cm (b) 12 cm (c) 15
cm (d) 18 cm
7 The height of a conical tent is 14 m and its floor area is 346.5 m 2 . The length of canvas, 1.1
m wide, required for it is (a) 490 m (b) 525 m (c) 665 m (d) 860 m
8 A metal sheet 27 cm long, 8 cm broad and 1 cm thick is melted into a cube. The difference
between surface areas of two solids is (a) 284 cm 2 (b) 285 cm2 (c) 286 cm2 (d) 287 cm2
9 Assertion : If the areas of three adjacent faces of a cuboid are x, y, z respectively then the
volume of the cuboid is √𝑥𝑦𝑧.
Reason : Volume of a cuboid whose edges are l, b and h is lbh units.
(a) Assertion and Reason both are correct statements and Reason is the correct
explanation of Assertion.
(b) Assertion and Reason both are correct statements but Reason is not the correct
explanation of Assertion.
(c) Assertion is correct statement but Reason is wrong statement.
(d) Assertion is wrong statement but Reason is correct statement
10 Assertion : From a solid cylinder, whose height is 12 cm and diameter 10 cm a conical
cavity of same height and same diameter is hollowed out. Then, volume of the cone is
2200
cm3 .
7
Reason : If a conical cavity of same height and same diameter is hollowed out from a
cylinder of height h and base radius r, then volume of the cone will be half of the volume
of the cylinder.
(a) Assertion and Reason both are correct statements and Reason is the correct
explanation of Assertion.
(b) Assertion and Reason both are correct statements but Reason is not the correct
explanation of Assertion.
(c) Assertion is correct statement but Reason is wrong statement.
(d) Assertion is wrong statement but Reason is correct statement
11 The decorative block shown in Fig. 13.7 is made of two solids — a
cube and a hemisphere. The base of the block is a cube with edge 5
cm, and the hemisphere fixed on the top has a diameter of 4.2 cm.
Find the total surface area of the block. (Take π = 22/7

12 Mayank made a bird-bath for his garden in the shape of a cylinder


with a hemispherical depression at one end (see Fig). The height of
the cylinder is 1.45 m and its radius is 30 cm. Find the toal surface
area of the bird-bath(Take π = 22/7)

13 A toy is in the form of a cone of radius 3.5 cm mounted on a hemisphere of same radius.
The total height of the toy is 15.5 cm. Find the total surface area of the toy

37
14 A cubical block of side 7 cm is surmounted by a hemisphere. What is the greatest diameter
the hemisphere can have? Find the surface area of the solid.
15 A wooden article was made by scooping out a hemisphere from each end of a
solid cylinder, as shown in Fig. If the height of the cylinder is 10 cm, and its
base is of radius 3.5 cm, find the total surface area of the article

16 A solid toy is in the form of a hemisphere surmounted by a right circular cone. The height
of the cone is 2 cm and the diameter of the base is 4 cm. Determine the volume of the toy.
If a right circular cylinder circumscribes the toy, find the difference of the volumes of the
cylinder and the toy. (Take π = 3.14)
17 A solid iron pole consists of a cylinder of height 220 cm and base diameter 24 cm, which is
surmounted by another cylinder of height 60 cm and radius 8 cm. Find the mass of the
pole, given that 1 cm3 of iron has approximately 8g mass. (Use π = 3.14)
18 A solid consisting of a right circular cone of height 120 cm and radius 60 cm standing on a
hemisphere of radius 60 cm is placed upright in a right circular cylinder full of water such
that it touches the bottom. Find the volume of water left in the cylinder, if the radius of the
cylinder is 60 cm and its height is 180 cm.

ANSWERS
1)b 2)d 3)b 4)d 5)a 6)c 7)b
8)c 9)a 10)c 11)163.86cm2 12)3.3m2
13)214.5cm2 14)332.5cm2 15)374cm2 16)25.12cm
17)892.26Kg 18)1130400cm3

38
Chapter 14
Statistics
Mean: The arithmetic mean of a given data is the sum of the values of all the observations divided by
the total number of observations. There are two different formulas for calculating the mean for
ungrouped data and the mean for grouped data.
Type 1 (Mean of raw data)
Suppose we have n values in a set of data namely as x1, x2, x3……… x n, then the mean of data is given
by using the formula

Type –2 Mean of grouped data(If Frequency and variable are given)


To calculate the mean of grouped data we have two different methods –
i. Direct method and
ii. Assumed mean method.
The mean of grouped data deals with the
frequencies of different observations or
variables that are grouped together. If
the values of the observations are x1, x2,
x3,............xn and their corresponding
frequencies are f1, f2, f3,...........fn , then the
mean of the data is given by,
Mean, x̄ = (x1f1 + x2f2 + ... + xnfn) / (f1 +
f2 + ... + fn)
x̄ = ∑xi fi / ∑fi, where i = 1, 2, 3, 4,......n

Type- 3 Mean for grouped data. (If Frequency and Class Intervals are given)
Here are the steps that can be followed to find the mean for grouped data using the direct method,
1. Create a table containing four columns such as class interval, class marks, frequency and product of
class marks and frequency.
f x
2. Calculate Mean by the Formula Mean x  i i Where fi is the frequency and xi is the mid value of
f i
the class interval.
3. To calculate the mid value xi, we use this formula xi = (upper limit + lower limit)/2.

Example. The table below gives information about the percentage distribution of female employees in
a company of various branches and a number of departments. Find the mean percentage of female
employee by the assumed mean method.

Female Employee 5-15 15-25 25-35 35-45 45-55 55-65 65-75

No of Departments 1 2 4 4 7 11 6

39
Solution:
Female No of Class Marks di = xi - a fi.di
Employee(C I) Departments( fi ) Xi
5-15 1 10 -30 -3
15-25 2 20 -20 -40
25-35 4 30 -10 -40
35-45 4 40 0 0
45-55 7 50 10 70
55-65 11 60 20 220
65-75 6 70 30 180
Total Σfi =35 Σfidi = 360
Assumed mean = a = 40
Mean = a+ (Σfidi /Σfi) = 40+ (360/35) = 40+(72/7) = 40 + 10.28 = 50.28
Hence, the mean percentage of female employees is 50.28.
Median
Median for raw data: The middle most term of the data arranged in ascending or descending order is
called the Median of the data. After arranging the data in ascending or descending order, the
following method is applied:
  n  1 
th

If number of values or observations in the given data is odd, then the median is given by  
 2 
observation.
th
n
If the number of values or observations is even, then the median is given by the average of  
2
th
n 
and   1  observation.
2 
Cumulative Frequency: The cumulative frequency of a class is the frequency obtained by adding the
frequencies of all the classes preceding the give class

Median for grouped data:


The median for grouped data can be calculated by using the formula,

Where

l = lower limit of median class


N = Sum of frequencies
cf = cumulative frequency of the preceding class
f = frequency of Median class
h = class size

Steps to Find Median of Grouped Data


Median of grouped data is in the form of a frequency distribution arranged in ascending order and is
continuous. To find the median of any given data is simple since the median is the middlemost value
of the data. Since the data is grouped, it is divided into class intervals. The steps to find the median of
grouped data are.
Step 1: Construct the frequency distribution table with class intervals and frequencies.
40
Step 2: Calculate the cumulative frequency of the data by adding the preceding value of the frequency
with the current value.
Step 3: Find the value of N by adding the values in frequency.
Step 4: Find the lower limit of the class interval and the cumulative frequency.
 n  cf 
Step 5: Apply the formula for median for grouped data: Median = l   2  h
 f 
Example
Q: Find the median of the following data is

Marks obtained 0-10 10-20 20-30 30-40 40-50


No of Students 5 7 4 8 6
Solution:
Marks No of Cumulative
obtained students frequency
0-10 5 5
10-20 7 5+7=12
20-30 4 12+4=16
30-40 8 16+8=24
40-50 6 24+6=30
Total N=30
Now N/2 = 30/2 = 15
15 lies in the in cumulative frequency having class intervals 20-30
20-30 is the median class
Now using formula

We have
Lower limit of median class = l = 20
Cumulative frequency preceding the median class= cf =12
Frequency of the median class =f= 4
Class size= difference of limits = h = 10
Median = 20 + (15 -12)/4 * 10
= 20 + 30/4
Median = 20 + 7.5 = 27.5
Mode

Mode: The mode for grouped data is given by the formula

Where,
l = Lower limit of the modal class
h = Class size
f1 = Frequency of the modal class
f0 = Frequency of the class preceding the modal class
f2 = Frequency of the class succeeding the modal class

41
Example
The following data gives the information on the observed life times (in hours) of 150 electrical
components. Find the mode of the data
Life time( In Hours) 0-20 20-40 40-60 60-80 80-100
Frequency 15 10 35 50 40
Solution:
The class 60-80 have the maximum frequency as 50

S.No QUESTION
1 Find the class marks of classes 10–20 and 35–55. (a) 10, 35 (b) 20, 55 (c) 15, 45 (d)
17.5, 45
2 If di = xi – 13, ∑fi di = 30 and ∑fi =120 , then mean, x is equal to
(a) 13 (b) 12.75 (c) 13.25 (d) 14.25
3 The mean of first ten odd natural numbers is (a) 5 (b) 10 (c) 20 (d) 19
4 If the mean of x, x + 3, x + 6, x + 9 and x + 12 is 10, then x equals
(a) 1 (b) 2 (c) 4 (d) 6
5 For a frequency distribution, mean, median and mode are connected by the relation
(a) Mode = 3 Mean – 2 Median (b) Mode = 2 Median – 3 Mean
(c) Mode = 3 Median – 2 Mean (d) Mode = 3 Median + 2 Mean
6 Assertion : Consider the following frequency distribution:
The modal class is 10-15.

Class interval 10-15 15-20 20-25 25-30 30-35


frequency 5 9 12 6 8
Reason : The class having maximum frequency is called the modal class.
(a) Assertion and Reason both are correct statements and Reason is the correct
explanation of Assertion.
(b) Assertion and Reason both are correct statements but Reason is not the correct
explanation of Assertion.
(c) Assertion is correct statement but Reason is wrong statement.
(d) Assertion is wrong statement but Reason is correct statement

42
7 Find the mean of the following distribution:

Class interval 0-6 6-12 12-18 18-24 24-30


frequency 7 5 10 12 2
8 Find the median of the following data :
CI 0-10 10-20 20-30 30-40 40-50 TOTAL
Frequency 8 16 36 34 6 100
9 The following table gives the literacy rate of 40 cities :
Literacy rate 30-40 40-50 50-60 60-70 70-80 80-90
No of cities 6 7 10 6 8 3
Find the modal literacy rate
10 Find the missing frequencies f1, f2 and f3 in the following frequency distribution, when
it is given that f2 : f3 = 4 : 3, and mean = 50
CI 0-20 20-40 40-60 60-80 80-100 TOTAL
Frequency 17 f1 f2 f3 19 120
11 Distance Analysis of Public Transport Buses
Transport department of a city wants to buy some
Electric buses for the city. For which they want to
analyse the distance travelled by existing public
transport buses in a day
Daily distance 200- 210- 220- 230- 240-
travelled (in km) 209 219 229 239 249
Number of buses 4 14 26 10 6
(i)Find the difference between upper limit of a class and lower limit of its succeeding
class
(ii) Find the median class
(iii) The cumulative frequency of the class preceding the median class is_____
Or Find the median of distance travelled
12 The following distribution shows the daily pocket allowance of children of a locality.
The mean pocket allowance is Rs. 18. Find the missing frequency f
Daily pocket 11-13 13-15 15-17 17-19 19-21 21-23 23-25
allowance
No of children 7 6 9 13 f 5 4

43
13 A class teacher has the following absentee record of 40 students of a class for the
whole term. Find the mean number of days a student was absent
No of days 0-6 6-10 10-14 14-20 20-28 28-38 38-40
No of students 11 10 7 4 4 3 1
14 If the median of the distribution given below is 28.5, find the values of x and y
CI 0-10 10-20 20-30 30-40 40-50 50-60 TOTAL
FREQUENCY 5 X 20 15 Y 5 60
15 A life insurance agent found the following data for distribution of ages of 100 policy
holders. Calculate the median age, if policies are given only to persons having age 18
years onwards but less than 60 year
Age (in years) Number of policy holders
Below 20 2
Below 25 6
Below 30 24
Below 35 45
Below 40 78
Below 45 89
Below 50 92
Below 55 98
Below 60 10

ANSWERS

1)C 2)C 3)B 4)C 5)C 6)D 7)14.5


8)27.22 9)54.29 10) f1=28,f2=32,f3=24
11)Case Study (i)1 (ii) 219.5-229.5 (iii) 18 or 224.12 12)20 13)12.48 days
14)x=8,y=7 15)35.76

44
Chapter 15
Probability
1. The theoretical (classical) probability of an event E, written as P(E), is defined as
𝐍𝐮𝐦𝐛𝐞𝐫 𝐨𝐟 𝐨𝐮𝐭𝐜𝐨𝐦𝐞𝐬 𝐟𝐚𝐯𝐨𝐮𝐫𝐚𝐛𝐥𝐞 𝐭𝐨 𝐄
𝐏 (𝐄) = 𝐍𝐮𝐦𝐛𝐞𝐫 𝐨𝐟 𝐚𝐥𝐥 𝐩𝐨𝐬𝐬𝐢𝐛𝐥𝐞 𝐨𝐮𝐭𝐜𝐨𝐦𝐞𝐬 𝐨𝐟 𝐭𝐡𝐞 𝐞𝐱𝐩𝐞𝐫𝐢𝐦𝐞𝐧𝐭
Where, we assume that the outcomes of the experiment are equally likely.
2. The probability of a sure event (or certain event) is 1.
3. The probability of an impossible event is 0.
4. The probability of an event E is a number P(E) such that 0≤P(E) 1
5. An event having only one outcome is called an elementary event. The sum of the probabilities of all
the elementary events of an experiment is 1.
6. For any event E, P( E )  P( E )  1
where E̅ stands for ‘not E’. E and E̅ are called complementary events.
S No QUESTIONS
1 Which of the following cannot be the probability of an event?
(A) 0.7 (B) 2 3 (C) – 1.5 (D) 15%
2 Which of the following can be the probability of an event?
(A) – 0.04 (B) 1.004 (C) 18 /23 (D) 8/ 7
3 An event is very unlikely to happen, its probability is closest to
(A) 0.0001 (B) 0.001 (C) 0.01 (D) 0.1
4 Out of one digit prime numbers, one number is selected at random. The probability of
selecting an even number is:
(A) 1/2 (B) 1/4 (C) 4/9 (D) 2/5
5 When a die is thrown, the probability of getting an odd number less than3 is:
(A) 1/6 (B) 1/3 (C) 1/2 (D) 0
6 A card is drawn from a well shuffled pack of 52 playing cards. The event E is that the
card drawn is not a face card. The number of outcomes favourable to the event E is
(A) 51 (B) 40 (C) 36 (D) 12
7 In a family of 3 children, the probability of having at least one boy is:
(A) 7/ 8 (B) 1/ 8 (C) 5/ 8 (D) 3/ 4
8 The probability of a number selected at random from the numbers 1, 2, 3, .... 15 is a
multiple of 4 is: (A) 4/15 (B) 2/15 (C) 1/5 (D) 1/3
9 The probability that a non-leap year selected at random will contains 53 Mondays is:
(A) 1/7 (B) 2/7 (C) 3/7 (D) 5/7
10 A bag contains 6 red and 5 blue balls. One ball is drawn at random. The probability that
the ball is blue is: (A) 2/11 (B) 5/6 (C) 5/11 (D) 6/11
11 Two dice, one blue and one grey, are thrown at the same time. Write down all the
possible outcomes. What is the probability that the sum of the two numbers appearing
on the top of the dice is (i) 8? (ii) 13 (iii) less than or equal to 12?
12 12 defective pens are accidentally mixed with 132 good ones. It is not possible to just
look at a pen and tell whether or not it is defective. One pen is taken out at random from
this lot. Determine the probability that the pen taken out is a good one

45
13 A box contains 90 discs which are numbered from 1 to 90. If one disc is drawn at
random from the box, find the probability that it bears (i) a two-digit number (ii) a
perfect square number (iii) a number divisible by 5
14 A lot consists of 144 ball pens of which 20 are defective and the others are good. Nuri
will buy a pen if it is good, but will not buy if it is defective. The shopkeeper draws one
pen at random and gives it to her. What is the probability that (i) She will buy it ? (ii)
She will not buy it?
15 A letter of English alphabet is chosen at random. Determine the probability that the
chosen letter is a consonant
16 A card is drawn from a well-shuffled deck of 52 playing cards. Then what is the
probability that the card will not be a diamond?
17 The probability of selecting a rotten apple randomly from a heap of 900 apples is 0.18.
What is the number of rotten apples in the heap?
18 Cards bearing numbers 2 to 21 are placed in a bag and mixed thoroughly. A card is
taken out of the bag at random. What is the probability that the number on the card
taken out is an even number?
19 A card is drawn out from a well-shuffled deck of 52 cards. What is the probability of
getting a red queen?
20 Two different dice are tossed together. Find the probability that (i) the number on each
dice is odd, and (ii) the sum on the numbers, appearing on the two dice, is 5.
21 Rahim tosses two different coins simulta-neously. Find the probability of getting at least
one tail.
22 Two different dice are rolled simultaneously. Find the probability that the sum of the
numbers appearing on the two dice is 10.
23 Two dice are thrown simultaneously. Find the probability that the sum of the numbers
appearing on the two dice is more than 9.
24 . A card is drawn at random from a well-shuffled pack of 52 cards. Find the probability
that the card is neither a red card nor a jack.
25 A box contains 100 red cards, 200 yellow cards and 50 blue cards. If a card is drawn at
random from the box, then find the probability that it will be : (i) a blue card, (ii) not a
yellow card, and (iii) neither yellow nor a blue card.

ANSWERS

1)C 2)C 3)A 4)B 5)A 6)B 7)A


8)C 9)A 10)C 11)5/36,0,1 12)11/12 13) 9/10,1/10,1/5
14)31/36 ,5/16 15)21/26 16)3/4 17)162 18)1/2 19)1/26
20)1/4,1/9 21)3/4 22)1/12 23)1/6 24)6/13 25)1/7,3/7,2/7

46
Class - X Session 2022-23
Subject - Mathematics (Standard - 041)
Sample Question Paper 1
Time Allowed: 3 Hours Maximum Marks: 80
General Instructions:
1. This Question Paper has 5 Sections A, B, C, D, and E.
2. Section A has 20 Multiple Choice Questions (MCQs) carrying 1 mark each.
3. Section B has 5 Short Answer-I (SA-I) type questions carrying 2 marks each.
4. Section C has 6 Short Answer-II (SA-II) type questions carrying 3 marks each.
5. Section D has 4 Long Answer (LA) type questions carrying 5 marks each.
6. Section E has 3 Case Based integrated units of assessment (4 marks each) with sub-parts of the
values of 1, 1 and 2 marks each respectively.
7. All Questions are compulsory. However, an internal choice in 2 Qs of 2 marks, 2 Qs of 3 marks and
2 Questions of 5 marks has been provided. An internal choice has been provided in the 2 marks
questions of Section E.
8. Draw neat figures wherever required. Take π =22/7 wherever required if not stated.
SECTION A
SECTION A CONSIST OF 20 QUESTIONS OF 1 MARKS EACH
S.NO. MARKS
1. The smallest number by which √27 should be multiplied so as to get a rational 1
number is
(a) √27 (b) 3√3 (c) √3 (d) 3
2. The ratio of LCM and HCF of the least composite number and least prime 1
number is
(a) 1: 2(𝑏) 2 ∶ 1 (𝑐 ) 1 ∶ 1 (𝑑 ) 1 ∶ 3
3. If P and Q are zeroes of 3𝑋 2 +2X-9 then the value of P-Q is 1
2 4√7
(a) −3 (𝑏 ) − (𝑐 ) (𝑑 )None of these
3 3
4. The value of K for which the system of equations 2X + 3 Y =5 and 4X+KY =10 1
has infinite number of solution is
(a) 1 (𝑏 ) 3 (𝑐 ) 6 (𝑑 ) 0
5. If one root of the equation 𝑎𝑥 2 + b x+ c =0 is three times the other, then 𝑏2 ∶ 𝑎𝑐 1
(a) 3 : 1 (b) 3 : 16 (c) 16 :3 (d) 16 : 1
6. The ratio in which x- axis divides the line segment joining (3 ,6 ) and (12 , -3 ) is 1
(a) 2 : 1 (b) 1 : 2 (c) 3 :2 (d) 1 : 3
7. In ∆ ABC , D and E are the points on sides AB and AC 1
respectively such that
𝐴𝐷 3
DE ∥ 𝐵𝐶 And 𝐷𝐵 = 1 if EA= 4.2 c m then AC is equal to

(a) 3.3 𝑐 𝑚 (𝑏)12.6 𝑐 𝑚 (𝑐 )5.6 𝑐 𝑚 (𝑑 ) 4.2 𝑐 𝑚


8. Which of the following is true for the following two triangles? 1

47
(a) ∆ 𝐴𝐵𝐶 ~ ∆ 𝐷𝐸𝐹 (𝑏) ∆ 𝐴𝐵𝐶~ 𝐷𝐹𝐸 (𝑐 ) ∆ 𝐵𝐴𝐶~∆ 𝐷𝐸𝐹 (𝑑 )∆𝐴𝐵𝐶 𝑖𝑠 𝑛𝑜𝑡 𝑠𝑖𝑚𝑖𝑙𝑎𝑟 𝑡𝑜∆ 𝐷𝐸
9. 𝐼𝑛 𝑡ℎ𝑖𝑠 𝑓𝑖𝑔𝑢𝑟𝑒 𝑡𝑤𝑜 𝑙𝑖𝑛𝑒 𝑠𝑒𝑔𝑚𝑒𝑛𝑡 𝐴𝐶 𝑎𝑛𝑑 𝐵𝐷 𝑖𝑛𝑡𝑒𝑟𝑠𝑒𝑐𝑡 𝑒𝑎𝑐ℎ 𝑜𝑡ℎ𝑒𝑟 𝑎𝑡 𝑡ℎ𝑒 𝑝𝑜𝑖𝑛𝑡 1
𝑃 such that
𝑃𝐴 = 6 𝑐 𝑚, 𝑃𝐵 = 3𝑐 𝑚, 𝑃𝐶 = 2.5 𝑐 𝑚 𝑃𝐷 = 5𝑐𝑚 , ∠𝐴𝑃𝐵 = 50° 𝑎𝑛𝑑
∠𝐶𝐷𝑃 = 30° 𝑡ℎ𝑒𝑛 ∠ 𝑃𝐵𝐴 𝑖𝑠 𝑒𝑞𝑢𝑎𝑙 𝑡𝑜

(a)500 (b)300 (c) 600 (d)1000


10. If 7𝑐𝑜𝑠30° + 5 𝑡𝑎𝑛 𝑡𝑎𝑛300 + 6 𝑐𝑜𝑡 𝑐𝑜𝑡 𝑖𝑠60°is 1
43 41√3 47 49√3
(a) 2 (b) (c) 2 (d)
√3 2 √3 2
11. 2 2 1
If a cos𝜃 + 𝑏𝑠𝑖𝑛 𝜃 =12 and a sin𝜃 − 𝑏𝑐𝑜𝑠𝜃=5 then 𝑎 + 𝑏 is equal to
(a) 13 (𝑏)12 (𝑐)14 4 (𝑑 )169
12. A bag contains 24 balls of which x are red and 2x are white and 3x are blue, a 1
ball is selected at random the probability that it is white ball is
1 5 3 7
(a) 3 (b) 6 (c) 7 (d) 24
13. 𝑎2 −𝑏2 1
If sin𝜃 = then the value of tan𝜃 is
𝑎2 +𝑏2
𝑎 +𝑏2
2 𝑎2 −𝑏2 2𝑎𝑏 2𝑎𝑏
(a) (𝑏) (𝑐 ) (𝑑 )
2𝑎𝑏 2𝑎𝑏 𝑎2 −𝑏2 𝑎2 +𝑏2
14. The length of minute hand of a clock is 14 c m find the area swept by the minute 1
hand in 5 minutes.
154 160 154 181
(a) 3 𝑐𝑚2 (b) 3 𝑐𝑚2 (c) 6 𝑐𝑚2 (d) 6 𝑐𝑚2
15. Mode for the following distribution is 22 if x< 𝑦 < 10 𝑡ℎ𝑒𝑛 the value of y is 1
CI 0 -10 10 -20 20-30 30-40 40-50 Total
Frequency 5 8 10 x y 30
(a) 2 (b) 5 (c) 3 (d) 4
16. If for a data mean : median = 9 : 8 then median : mode = 1
(a) 8 : 9 (b) 4 :3 (c) 7 :6 (d) 5 : 4
17. Find the area of quadrant of a circle whose circumference is 22 cm. 1
22 77 77 8
(a) 8 𝑐𝑚2 (𝑏) 8 𝑐𝑚2 (𝑐 ) 22 𝑐𝑚2 (𝑑 ) 77 𝑐𝑚2
18. If the perimeter of the bases of two right circular cone are in the ratio 3:4 and 1
their volumes are in the ratio 9:32 then the ratio of their heights is-
(a) 1:3 (b) 2:1 (c) 1:2 (d) 1:3
Direction for questions 19 & 20: In question numbers 19 and 20, a
statement of Assertion (A) is followed by a statement of Reason (R). Choose
the correct option.

48
19. Assertion: If HCF (90,144)= 18 then LCM(90,144) = 720 1
Reason: HCF (a,b) x LCM (a,b)= axb
(a) Both Assertion (A) and Reason (R) are true and Reason (R) is the correct
explanation of Assertion (A).
(b) Both Assertion (A) and Reason (R) are true but Reason (R) is not the correct
explanation of Assertion (A).
(c) Assertion (A) is true but Reason (R) is false.
(d) Assertion (A) is false but Reason (R) is true.
20. Assertion: if the coordinate of the midpoints of sides AB and AC of triangle ABC 1
are D(3,5) and E(-3,-3) respectively then BC= 20 units
Reason:
𝑇ℎ𝑒 𝑙𝑖𝑛𝑒 𝑠𝑒𝑔𝑚𝑒𝑛𝑡 𝑗𝑜𝑖𝑛𝑖𝑛𝑔 𝑡ℎ𝑒 𝑚𝑖𝑑 𝑝𝑜𝑖𝑛𝑡𝑠 𝑜𝑓 𝑡𝑤𝑜 𝑠𝑖𝑑𝑒𝑠 𝑜𝑓 𝑎 𝑡𝑟𝑖𝑎𝑛𝑔𝑙𝑒 𝑖𝑠 𝑝𝑎𝑟𝑎𝑙𝑙𝑒𝑙 𝑡𝑜 𝑡ℎ𝑒 𝑡ℎ𝑖𝑟𝑑 𝑠𝑖𝑑𝑒
(a) Both Assertion (A) and Reason (R) are true and Reason (R) is the correct
explanation of Assertion (A).
(b) Both Assertion (A) and Reason (R) are true but Reason (R) is not the correct
explanation of Assertion (A).
(c) Assertion (A) is true but Reason (R) is false.
(d) Assertion (A) is false but Reason (R) is true.
Section B
Section B consists of 5 questions of 2 marks each.
21. Solve for X and Y: 8X+5Y=9 3X +2Y=4 2
22. Diagonal AC and BD of trapezium ABCD with AB∥ 𝐷𝐶 intersect each other at the 2
point O. using a similarity criterion for two triangles.
𝐴𝑂 𝐵𝑂
Show that 𝐶𝑂 =𝐷𝑂
23. Prove that the tangents drawn at the ends of a diameter of a circle are parallel. 2
24. 7 (1+𝑠𝑖𝑛𝜃) (1−𝑠𝑖𝑛𝜃) 2
If cot𝜃 = 8 then evaluate (1+𝑐𝑜𝑠𝜃)(1−𝑐𝑜𝑠𝜃)
OR
Evaluate
5𝑐𝑜𝑠 2 600 + 4𝑠𝑒𝑐 2 300 − 𝑡𝑎𝑛2 450
𝑠𝑖𝑛2 300 + 𝑐𝑜𝑠 2 300
25. In a circle of radius 21 c m. an arc subtends an angle of 2
600 at the centre. find
(a) Length of an arc
(b) Area of sector
OR
Find the area of shaded region (in this figure ) if radii of
two concentric circles with centre o. are 7 c m and 14
cm respectively and ∠𝐴𝑂𝐶 = 400

Section C
Section C consists of 6 questions of 3 marks each.
26. Prove that7√2 is an irrational number. 3
1
27. Find the zeroes of the polynomial 𝑋 2 +6X -2 and verify the relationship between 3
the zeroes and the coefficient.
28. If the equation (1+𝑚2 ) 𝑥 2 +2mcx + 𝑐 2 - 𝑎2 = 0 has equal roots. Show that 𝑐 2 = 3
𝑎2 (1 + 𝑚2 )
OR
Find two consecutive positive integers. Sum of whose squares is 365.
49
29. Two tangents TP and TQ are drawn to a circle with centre o from external point 3
T. prove that ∠𝑃𝑇𝑄 = 2∠𝑂𝑃𝑄

OR
In this figure XY and 𝑋1 𝑌1 are two parallel tangents to a circle with centre o.
and another tangent AB with point of contact C intersect XY at A and 𝑋1 𝑌1 at B
prove that ∠𝐴𝑂𝐵 = 900

30. 1+𝑠𝑖𝑛𝐴 3
Prove that : √1−𝑠𝑖𝑛𝐴 = 𝑠𝑒𝑐𝐴 + 𝑡𝑎𝑛𝐴
31. A die is thrown twice. What is the probability that 3
(a ) 5 will come up at least once .
(b ) 5 will not come up either time .
Section D
Section D consists of 4 questions of 5 marks each.
32. State and prove that the Basic proportional theorem. 5
Using this theorem ( in this figure ) If DE∥ OQ and DF ∥ OR then show that 𝐸𝐹 ∥ 𝑄𝑅

33. The median of the following data is 525. find the value of X and Y. if total frequency 5
is 100.
CI Frequency
0 - 100 2
100 - 200 5
200 - 300 X
300 - 400 12
400 - 500 17
500 - 600 20
600 - 700 Y
700 - 800 9
800 - 900 7
900 - 1000 4
50
34. RS 9000 were divided equally among a certain number of persons. Had there been 5
20 more persons each would have got RS 160 less. Find the original number of
persons.
OR
3
Two water taps together can fill the tank in 98 hrs. The tap of larger diameter takes
10 hrs less the smaller one to fill the tank separately. Find the time in which each
tap can separately fill the tank.
35. A tent is in the shape of cylinder surmounted by a conical top. If the height and 5
diameter of the cylindrical part are 2.1 m and 4 m respectively and slant height of
top is 2.8 m. find the area of canvas used for making the tent .Also find the cost of
the canvas of tent at the rate of Rs 500 per 𝑚2 .
OR
A Gulabjamun contains sugar syrup upto about 30% of its volume. Find
approximately how much syrup would be found in 45 gulabjamuns, each shaped
like a cylinder with two hemi spherical ends with length 5cm and diameter 2.8 cm.
Section E
Case study based questions are compulsory.
36 As observed from the top of a 60 M high light house from the sea level the angle of
depression of two ships are 280 and 45° one ship exactly behind the other on the
same side of the light house . (use tan 28° = 0.5317)

1
1

2
(i) Find the distance between the two ships.
(ii) Find the distance between the foot of the light house and first ship from the
light house .
(iii) Find the distance from the top of light house (A) to the first ship(D) .
OR
Find the value of cosec B
37. A triangular plot is marked with parallel and vertical lines. A
pair of such lines are taken as x- axis and y- axis respectively
as shown in graph P ,Q ,R are mid points of AB, BC and CA
respectively where three pillars for makings were erected in
the ground and are joined with ropes.

(i) Find coordinates of mid point of rope PQ . 1


(ii) Find coordinates of R. 1
(iii) Find the ratio in which BC is divided by y-axis.
OR 2
Find the ratio in which y-axis divides AC .
51
38. Rishi wants to buy a car and plans to take loan from a bank to
buy the car. He pays his total loan of Rs 1180000 by paying
every month starting with the first instalment of RS 10000. If he
increases the instalment by RS 1000 every month answer the
following.
(i) Find the amount paid by Rishi in 30𝑡ℎ instalment. 1
(ii) Find the amount paid by Rishi in 30 instalment. 1
(iii) If the loan is to be repaid in 40 instalments, thenfind the amount paid in the
last instalment . 2
OR
Find the ratio of the first instalment to the last instalment.

52
MARKING SCHEME Sample Paper 1
Q.No SECTION – A Marks
1. (c ) √3 1
2. (b) 2 : 1 1
3. (c )
4√7 1
3
4. (c) k =6 1
16
5. (c) 1
3
6. (a) 2:1 1
7. ( c) AC =5.6 1
8. (d) ΔABC is not similar to ΔDEF 1
9. (d) 100֯ 1
43
10. (a) 2 1
√3
11. (d) 169 1
1
12. (a) 3 1
13. 𝑎 2 −𝑏 2 1
(b)
2𝑎𝑏
14. 154 1
(a) 3
𝑐𝑚 2
15. (b) y = 5 1
16. (b)4:3 1
77
17. (b) 1
8
18. (c) 1:2 1
19. (a) Both Assertion (A) and Reason (R) are true and Reason (R) is the correct 1
explanation of Assertion (A).
20 (b) Both Assertion (A) and Reason (R) are true but Reason (R) is not the 1
correct explanation of Assertion (A).
21. 8x +5y = 9 1
3x + 2y = 4
X = -2 1
7
And y =
5
22. In ΔAOB and ΔDOC
∠1 = ∠2(alternate interior) 1
∠3 = ∠4 ( alternate interior)
Δ AOB ~ Δ COD (by aa) 1
𝐴𝑂 𝑂𝐵
𝐶𝑂
= 𝑂𝐷
23. ∠1 + ∠2 = 180֯ so l ∥ 𝑚 2
8 7
24. Sin 𝜃= , Cos 𝜃= 1
√113 √113
8 8
(1 + ) (1 - )
√113 √113
7 7 1
(1 + ) (1 - )
√113 √113
49
113 49
64 = 64
113
OR 1

53
12 22
5 2
+4 − 12
√3
2
12 √3
2
+ 2
1+1
5 16
+ −1 67
4 3
1 3 =
+ 12
4 4
25. 𝜋𝑟𝜃 1
(i)Length = 180 = 22 cm

𝜃
(ii) Area of sector= 𝜋𝑟 2
360
1
2
11 x 21= 231 𝑐𝑚
OR
𝜃 2 2 1
Area of shaded region = 360 𝜋(𝑅 − 𝑟 )
308
𝑐𝑚 2 1
3
26. Assume, √2is a rational number, it can be written as p/q, in which p and q are
co-prime integers and q≠0,
i.e. √2= p/q. where, p and q are coprime numbers, and q≠0.
On squaring both sides of the above equation;
2= (p/q)2
2 = p2/q2 1
2q2 = p2 ...(i)
p2 is a multiple of 2 so p is a multiple of 2 ...(ii)
Since, p is a multiple of two.
So p = 2m
p² = 4m² …(iii)
Using equation(i) into the equation (iii), we get; 1
2q² = 4m²
So q² = 2m²
q2 is a multiple of 2 so q is a multiple of 2 ...(iv)
Equation (ii) and (iv), implies that p and q have a common factor 2.
It contradicts the fact that they are co-primes which lead from our wrong
assumption that 2is a rational number.
Multiplication of rational and irrational number is always irrational number so
7 √2 is an irrational number. 1
1
27. { 6𝑥 2 + 𝑥 − 12} = 0 1
6
(3x -4) (2x+3)
4 3 1
X = or−
3 2
verify 1
4 3 1
𝛼 + 𝛽= − = −
3 2 6
∝ 𝛽 = -2
28. 𝑏2 - 4ac = 0 1
2𝑚𝑐 2 – 4 (1 + 𝑚)2 (𝑐 2 - 𝑎2 )= 0 1
𝑐 2 = 𝑎2 ( 1 + 𝑚2 ) 1
OR
𝑥 2 + (𝑥 + 1)2 = 365
1
𝑥 2 + 𝑥 − 182 = 0
54
(x+14) (x-13) = 0 1
So x = 13
1 positive integer = 13 1
2 positive integer = 14
29. Consider the problem
Let us join point O to C
In ΔOPA and ΔOCA
OP=OC (Radii of the same circle) 1
AP=AC (Tangent from point A)
AO=AO (Common side)
ΔOPA≅ΔOCA (SSS congruence criterion)
Therefore, P↔C,A↔A,O↔O 1
∠POA=∠COA.........(1)
Similarly,
∠QOB≅∠OCB
∠QOB=∠COB.........(2)
Since, POQ is a diameter of the circle, it is a straight line.
Therefore, ∠POA+∠COA+∠COB+∠QOB=180°
So, from equation (1) and equation (2) 1
2∠COA+2∠COB=180°∠COA+∠COB=90°∠AOB=90°
OR
We know that length of tangents drawn from an external point to a circle are
equal
∴ TP=TQ−−−(1)
4∴ ∠TQP=∠TPQ (angles of equal sides are equal)−−−(2)
Now, PT is tangent and OP is radius. 1
∴ OP⊥TP (Tangent at any point of circle is perpendicular to the radius through
point of cant act)
∴ ∠OPT=90°

or, ∠OPQ+∠TPQ=90°

or, ∠TPQ=90°−∠OPQ−−−(3) 1

In △PTQ

∠TPQ+∠PQT+∠QTP=180° (∴ Sum of angles triangle is 180°)

or, 90°−∠OPQ+∠TPQ+∠QTP=180°

or, 2(90°−∠OPQ)+∠QTP=180° [from (2) and (3)]

or, 180°−2∠OPQ+∠PTQ=180°
1

∴ 2∠OPQ=∠PTQ−−−− proved
30. 𝟏+𝒔𝒊𝒏𝑨
LHS √𝟏−𝒔𝒊𝒏𝑨

𝟏 + 𝒔𝒊𝒏𝑨( 𝟏 + 𝒔𝒊𝒏𝑨) (𝟏 + 𝒔𝒊𝒏𝑨)𝟐 1


√ = √
𝟏 − 𝒔𝒊𝒏𝑨( 𝟏 + 𝒔𝒊𝒏𝑨) 𝟏 − 𝒔𝒊𝒏𝟐 𝑨
55
(𝟏+𝒔𝒊𝒏𝑨)𝟐 𝟏+𝒔𝒊𝒏𝑨 𝒔𝒊𝒏𝑨 𝟏
=√ = = + 1
𝒄𝒐𝒔𝟐
++

𝑨 𝒄𝒐𝒔𝑨 𝒄𝒐𝒔𝑨 𝒄𝒐𝒔𝑨


= tan A + sec A = RHS 1
31. Throwing a die twice and throwing two dice simultaneously are treated as the
same experiment.
Sample space ={(1,1),(1,2),(1,3),(1,4),(1,5),(1,6)
(2,1),(2,2),(2,3),(2,4),(2,5),(2,6) 1
(3,1),(3,2),(3,3),(3,4),(3,5),(3,6)
(4,1),(4,2),(4,3),(4,4),(4,5),(4,6)
(5,1),(5,2),(5,3),(5,4),(5,5),(5,6)
(6,1),(6,2),(6,3),(6,4),(6,5),(6,6)}
i) P(5 will not come up either time)= Number of times 5 does not show
divided by total number of outcomes
25
P(5 will not come up either time)=36
1
ii) P(5 will come up at least once)= Number of times 5 shows up at least once
divided by total number of outcomes
11
P(5 will come up at least once)=36
1
32. Computation of Median
Class interval Frequency Cumulative frequency (cf)
(f)
0-100 2 2
1
100-200 5 7
200-300 x 7+x
300-400 12 19+x
400-500 17 36+x
500-600 20 56+x
600-700 y 56+x+y
700-800 9 65+x+y
800-900 7 72+x + y
900-1000 4 76+x + y
Total = 100
We have,
N=∑fi=100
⇒76+x+y=100⇒x+y=24
It is given that the median is 525. Clearly, it lies in the class 500−600 1
∴l=500, h=100,f=20,F=36+x and N=100
𝑛
−𝑐𝑓
2
Now,Median= l + ×h 1
𝑓
100
−36−𝑥
2
⇒525=500+ 20 ×100 1
⇒525−500=(14−x)×5
⇒25=70−5x⇒5x=45⇒x=9
1
Putting x=9 in x + y=24, we get y=15.
Hence, x=9 and y=15.
33. Given, to prove, figure 1
Correct proof of bpt theorem
In ΔPOQ, we have 2

56
DE∥ OQ(Given)
By Basic proportionality Theorem, we have
𝑬 𝑸 𝑭𝑹
= (i)
𝑷𝑬 𝑷𝑭
Similarly,
In ΔPOR, we have
DF∥ OR(Given)
𝑫𝑶 𝑭𝑹
∴𝑷𝑫 =𝑷𝑭(ii)
Now, from (i) and (ii), we have
𝑫𝑶 𝑬𝑸
⇒ 𝑷𝑫 = 𝑷𝑬 𝒔𝒐EF∥QR 2
[Applying the converse of Basic proportionality Theorem in ΔPQR]
34. Let there be n persons and each get p rupees
9000
Hence, p= 𝑛
9000 9000
- 𝑛+20 = 160 1
𝑛
2
𝑛 +20n−1125=0
𝑛2 +45n−25n−1125=0
(n+45)(n−25)=0 2
n=25,−45
Thus, number of persons are 25
OR
It is given that the tank is filled in 875 hours that is, the taps fill 758 part of the 1
tank in 1 hour. Then,
1 1 8
+ 𝑥+10 = 75
𝑥
4𝑥 2 −115x+375=0 1
(4x−15)(x−25)=0
4x−15=0
x=415
Or, 2
x−25=0
x=25
When x=415, then, x−10=415−10
=415−40 1
=−425
This cannot be possible because time can never be negative.
When x=25, then, 1
x−10=25−10
x=25
Therefore, the tap of smaller diameter can separately fill the tank in 25 hours.
1
35. CSA of tent = CSA of cylinder+ CSA of cone 1
= 2𝜋𝑟ℎ + 𝜋rl 2
=𝜋𝑟(2ℎ + 𝑙)
22
= × 2(4.2 + 2.8)
7
1
= 44𝑚 2
1
= cost of canvas= 44× 500 = 𝑅𝑆22000
OR
1
Diameter of cylinder = Diameter of hemisphere =2.8 cm
Radius = 1.4 cm, height of cylinder= 2.2 cm
2
Volume of gulabjamun = volume of cylinder +2(𝑣𝑜𝑙𝑢𝑚𝑒 𝑜𝑓 ℎ𝑒𝑚𝑖𝑠𝑝ℎ𝑒𝑟𝑒)
57
4
Volume of 1 gulabjamun = 𝜋𝑟 2 h+3 𝜋𝑟 3
22 4
= 7 × 1.4 × 1.4 (2.2 + 3 × 1.4) 1
3
= 25.06𝑐𝑚
Volume of 45 gulabjamun = 25.06× 45 = 1127.25𝑐𝑚 3
30
30 percentage volume of sugar syrup = 1127.25× 100 =338𝑐𝑚 3
1
36. 1.) 52.85M 2. ) 60 M 3. )60√2 M 1+1
OR
Cosec B = 0 2
37 1) (1 , 4 ) 1
2) Coordinate of point R are ( 4 , - 3 )
3 ) Ratio 1: 5 1
OR
Ratio 1 : 5 2
38 1) 39000 1
2) 735000 1
3 )49000 2
OR
10 : 49

58
KENDRIYA VIDYALAYA SANGHTHAN, JAIPUR REGION
Class - X Session 2022-23
Subject - Mathematics STANDARD (041)
SAMPLE PAPER 2

Time Allowed: 3 Hours Maximum Marks: 80

General Instructions: As per the previous question paper


Section A

Section A consists of 20 questions of 1 mark each.


S No. Marks
1 If one zero of the polynomial f(x) = (k2 + 4)x2 + 13x +4k is 1
reciprocal of the other, then k
(a)2 (b)-2 (c)1 (d)-1
2 If the LCM of two prime number p and q (p>q) is 221 then the value 1
of 3p – q is
(a) 4 (b) 28 (c) 38 (d) 48
3 If 217x + 131y = 913 and 131x + 217y = 827, then x + y is equal to 1
(a)5 (b)6
(c)7 (d)8
4 If 2 is a root of the equation x2 + bx + 12 = 0 and the equation x2 + 1
bx + q = 0 has equal roots, then q is
(a)8 (b)-8 (c)16 (d) -16
5 If ABC and DEF are similar triangles such that A = 470 and E = 1
830, then C is
(a) 500(b)600 (c) 700 (d) 800
6 In adjoining figure, the value of x for 1
which DE ll BC is

(a)4 (b)1

(c)3 (d)2
7 A vertical stick 20 m long casts a shadow 10 m long on the ground. 1
At the same, a tower casts a shadow 50 m long on the ground. The
height of the tower is
(a)100 m (b)120 m
(c) 25 m (d) 200 m
8 If the centroid of the triangle formed by the points (3, -5), (-7, 4), 1
(10,-k) is at the point (k, -1), then k is
(a)3 (b)1
(c)2 (d) 4
9 If x = 2sin  and y = 2cos2 +1, then x + y is equal to
2 1
(a)3 (b)2
(c)1 (d)4
10 If 𝑘 + 1 = sec 2 𝜃(1 + sin 𝜃)(1 − sin 𝜃), 𝑡ℎ𝑒𝑛 𝑡ℎ𝑒 𝑣𝑎𝑙𝑢𝑒 𝑜𝑓 𝑘 𝑖𝑠 1

59
(a)1 (b)2 (c) 0 (d)-1
8
11 If  is an acute angle such that tan2 = 7 , then the value of 1
(1+𝑠𝑖𝑛𝜃)(1−𝑠𝑖𝑛𝜃)
is
(1+𝑐𝑜𝑠𝜃)(1−𝑐𝑜𝑠𝜃)
7 8
(a) (b)7
8
7 64
(c) (d) 49
4
12 In adjoining figure, APB is a tangent to a 1
circle with centre O at point P. if QPB =
500, then the measure of POQ is

(a)1000 (b) 1400


(c) 120 0 (d)1500
13 The minute hand of a clock is 12 cm long. The area swept by minute 1
hand in 35 minutes is
(a) 156 cm2 (b)264 cm2
(c)164 cm2 (d) 120 cm2
14 If an arc of a circle of radius 14 cm subtends an angle of 45o at the 1
centre of the circle, then its is
(a) a minor arc of length of 5.5 cm (b)a major arc of length 77 cm
(c) a major arc of length 38.5 cm (d) a minor arc of length 11cm
15 The maximum volume of a cone that can be carved out of a solid 1
hemisphere of radius r is
𝜋𝑟 3
(a)3𝜋r2 (b) 3
𝜋𝑟 2
(c)3𝜋𝑟3 (d) 3
16 Two dice are thrown together. The probability of getting the 1
difference of numbers on their upper faces equal to 2, is
5 4
(a) 9 (b)9
1 2
(c) 3 (d) 9
17 If the sum of 15 observations of a data is (434 + x) and the mean of 1
the observation is x, then x is equal to
(a)25 (b)27
(c)31 (d) 33
18 If the difference of mode and median of a data is 24, then the 1
difference of median and mean is
(a)12 (b)24
(c)8 (d) 36
Direction for question 19 & 20: In question numbers 19 and 20, a
statement of Assertion (A) is followed by a statement of reason(R).
Choose the correct option.
19 Assertion: If HCF(a, b) = 4 and ab = 96 x 404, then LCM(a, b) = 9696 1
Reason: LCM of two number a and b = HCF(a, b) x ab
(a)Both Assertion (A) and Reason (R) are true and Reason (R) is the
correct explanation of Assertion (A).
(b) Both Assertion (A) and Reason (R) are true but Reason (R) is
not the correct explanation of Assertion (A).
(c) Assertion (A) is true but Reason (R) is false.
60
(d) Assertion (A) is false but Reason (R) is true.

20 Assertion: The perimeter of ∆OAB where O is origin, A(3,0), B(0, 4) 1


is 7 units.
Reason: Perimeter of a triangle is the sum of all three sides of the
triangle.
(a)Both Assertion (A) and Reason (R) are true and Reason (R) is the
correct explanation of Assertion (A).
(b) Both Assertion (A) and Reason (R) are true but Reason (R) is not
the correct explanation of Assertion (A).
(c) Assertion (A) is true but Reason (R) is false.
(d) Assertion (A) is false but Reason (R) is true.
Section B

Section B consists of 5 questions of 2 marks each.


21 For what value of k, will the following system of equation has no 2
solution
(3k + 1)x + 3y – 2 = 0 (k2 + 1)x + (k - 2)y – 5 = 0
22 𝑄𝑅 𝑄𝑇 2
In the adjoining figure, 𝑄𝑆 = 𝑃𝑅 and PQR =
PRQ. Prove that

∆𝑃𝑄𝑆 ~ ∆𝑇𝑄𝑅.

23 Two tangents PA and PB are drawn to a circle with centre O from an 2


external point P. Prove that APB = 2 OAB

24 The perimeter of a sector of a circle of radius 5.8 m is 27.2 m. Find 2


the area of sector.
OR
Area of a sector of central angle 200o of a circle is 770 cm2. Find the
length of the corresponding arc of this sector.
25 Given that cos(A + B) = cosAcosB – sinAsinB 2
Find the value of cos1050
OR
If sin𝜃 + sin 𝜃 = 1, then prove that cos2𝜃 + cos4𝜃= 1
2

Section C

Section C consists of 6 questions of 3 marks each.


26 Prove that √7 is an irrational number. 3

61
27 Find the value of k such that the polynomial x2 – (k + 6)x + 2(2k + 3
1) has sum of its zeroes equal to half of their product.
28 The area of a rectangle gets reduced by 9 sq units if its length is 3
reduced by 5 units and breadth is increased by 3 units. If we
increase the length by 3 units and breadth by 2 units the area
increased by 67 units. Find the dimensions of the rectangle?
OR
The sum of numerator and denominator of a fraction is 4 more than
twice the numerator. If the numerator and denominator are
increased by 3 they are in the ratio 2:3. Find the fraction.
29 Prove that opposite sides of a quadrilateral circumscribing a circle 3
subtend supplementary angles at the centre of the circle.
OR
A circle is touching the side BC of ∆ABC at P and touching AB and AC
produced at Q and R respectively. Prove that
1
AQ = 2 (Perimeter of ∆ABC)
30 If tan𝜃 + sin𝜃 = m and tan𝜃 – sin𝜃 = n show that m2 – n2 = 4√𝑚𝑛 3
31 A card is drawn at random from a well shuffled deck of playing 3
cards. Find the probability the card drawn is
(a) A card of spade and an ace
(b) Either a king or a queen
(c) A red king
Section D

Section D consists of 4 questions of 5 marks each.


32 A fast train takes 3 hours less than a slow train for a journey of 600 5
km. If the speed of the slow train is 10 km/hr less than that of the
fast train, find the speeds of the two train.
OR
A two digit number is such that the product of its digit is 14. If 45 is
added to the number, the digits interchange their places. Find the
number.
33 Sides AB and AC and median AD of a triangle ABC are respectively 5
proportional to sides PQ and PR and median PM of another triangle
PQR. Show that Δ ABC ~ Δ PQR.
34 The interior of a building is in the form of a right circular cylinder of 5
diameter 4.2 m and height 4 m, surmounted by a cone. the vertical
height of the cone is 2.4 m. find the outer surface area and the
volume of the building.
OR
A solid is in the form of a cylinder with hemispherical ends, the total
height of the solid is 19 cm and diameter of the cylinder is 7 cm. Find
the volume and total surface area of solid.
35 If the mean of the following frequency distribution is 188. Find the 5
missing frequency f1 and f2
Classes 0-80 80- 160-240 240- 320- Total
160 320 400
frequency 20 25 f1 f2 10 100

62
Section E
Case study based questions are compulsory.
36 Case study 1
Clock towers are a specific type of building which house a turret
clock and have one or more clock faces on the upper exterior walls.
Many clock towers are freestanding structures but they can also
adjoin or be located on top of another building. Some other buildings
also have clock faces on their exterior but these structures serve
other main functions.
A 1.5 m tall person is standing at a distance of 30√3 m from the
clock tower, observes the angle of elevation of the top of the Clock
Tower to be 300

Based on the above information answer the following questions


(i)What is the distance between viewer’s eye and the top of clock
tower?
1
(ii)What is the the height of the clock tower?
1
(iii)The viewer’s walked to a position from where the elevation of 2
the top becomes 600. How much distance did he walk towards the
tower ?
OR
How much should the height of the tower be increased so that the
elevation of the top observed by the viewer is 600 at the same point?

37
Case study 2
In November 2020, some new animals
were added to a zoo. As a result the
number of visitors to the zoo, increased
daily by 10. A total of 6150 people
visited zoo during that month.

Based on the above information, answer the following question: 1


(i)How many visitors visited the zoo on 1st November? 1
(ii) on which day of the month did 250 visitors visit the zoo? 2
(iii) how much collection (in rupees) from sale of tickets was done
in the zoo on 15 November, if each entry ticket cost Rs 50?
OR
If the zoo is closed on 1st December due to some reason, what will be
the loss (in rupees) in terms of sale of tickets?

63
38 Case study 3
Aditya asked carpenter to make front door of his guest house. The
carpenter suggested him a design which is plotted on a graph as
shown in below figure:

1
Based on the above information, answer the following question: 1
(i) what is the length of line segment AB? 2
(ii) what is the coordinates of the midpoint of BE?
(iii) In what ratio BD divides by y-axis , if we join BD?
OR
In what ratio BG divides by y-axis , if we join BG?

64
Class- X
Mathematics Standard(041)
Marking Scheme Sample Paper 2
Section A
1 (1)2 1
2 (c)38 1
3 (a)5 1
4 (c)16 1
5 (a)500 1
6 (d)2 1
7 (a)100 m 1
8 (c)2 1
9 (a)3 1
10 (b)1 1
7
11 (a)8 1
12 (a)1000 1
13 (b)264 cm2 1
14 (d)44.8 cm2 1
15 𝜋𝑟3 1
(b) 3
2
16 (d)9 1
17 (c)31 1
18 (a)12 1
19 (c) Assertion (A) is true but Reason (R) is false 1
20 (d) Assertion (A) is false but Reason (R) is true 1
Section B
21 The given system of equations are
(3k+1)x + 3y – 2 =0 ½
(k2+1)x + (k-2)y – 5 = 0
For no solution,we must have
𝑎1 𝑏 𝑐 ½
= 𝑏1 ≠ 𝑐1
𝑎2 2 2
The given system of equations will have no solution,if
3𝑘+1 3 −2
= 𝑘−2 ≠ −5
𝑘 2 +1
Now,
3𝑘+1 3
=
𝑘 2 +1 𝑘−2
So, (3k+1)(k-2) = 3(k2+1)
½
3k2-5k-2 = 3k2 + 3
K = -1
3 −2 ½
Clearly, 𝑘−2 ≠ −5 for k≠ 6

65
22 In ΔPQR,
∠1 = ∠2
∠PQR = ∠PRQ [GIVEN]
∴ PR = PQ ……………..…(1) ½
[Sides opposite to equal angles of a triangle are also equal]
Given:
QR/QS = QT/PR
QR/QS = QT/PQ
QS/QR = PQ/QT…... …(ii) [Taking reciprocals]
[From eq (i)]
In ΔPQS and ΔTQR, ½
QS/QR = PQ/QT [From eq (ii)]
∠PQS = ∠TQR [ common]
∴ΔPQS ~ ΔTQR [By SAS similarity criterion] 1
23 Let ∠APB= x°
We know that the tangents to a circle from an external point are equal in length
so PA= PB.
PA =PB
∠PBA = ∠PAB
[Angles opposite to the equal sides of a triangle are equal.] 1
∠APB+ ∠PBA +∠PAB= 180°
[Sum of the angles of a triangle is 180°]
x° + ∠PAB +∠PAB = 180°
[∠PBA = ∠PAB]
x° + 2∠PAB = 180°
∠PAB =½(180° - x°)
∠PAB =90° - x°/2
∠OAB +∠PAB=90°
∠OAB =90° - ∠PAB
∠OAB =90° - (90° - x°/2)
∠OAB =90° - 90° + x°/2
∠OAB = x°/2
∠OAB = ∠APB /2
∠OAB = 1/2∠APB 1
∠APB = 2∠OAB
24 Perimeter of the sector = 27.2 m
Radius of the sector (r) = 5.6 m
Length of the arc of the sector
(l) = Perimeter - 2(radius) 1
= 27.2 - 2(5.6) = 27.2 - 11.2
= 16 Area of the sector
= lr/2
= (16 × 5.6) / 2
= 8 × 5.6 1
= 44.8 m2.
25 cos1050 = cos(600+450)=cos600 cos450 - sin 600 sin 450 1
1 1 √3 1
= 2× - ×
√2 2 √2 1
1−√3
=2
√2
OR
66
Sinθ=1-sin2θ 1
sinθ=cos2θ
So , sin2θ = cos4θ
cos2θ+cos4θ
cos2θ+ sin2θ = 1 1
Section C
26 Let we suppose √7 is rational number so that it can be represented in the form of
p/q where p and q are co-prime integers and q≠0.
hence we can let,
√7=p/q 1
squaring both sides
(√7)²=(p/q)²
7=p²/q²
7q²=p².......(i)
7 divides p²
therefore there exists an integer r
hence,
p=7r
squaring both sides 1
(p)²=(7r)²
p²=49r².........(ii)
From (i) and (ii)
7q²=49r²
q²=49r²/7
q²=7r²
7 divides by q²
7 divide q 1
But we already show that 7 divides p.
This implies that 7 is common factor of p and q.
but this contradict our assumption that p and q have no common factors.
hence our assumption is wrong that √7 is rational.
Therefore √7 is an irrational number.
27 Let α and β are the roots of given quadratic equation
x² - ( k +6)x + 2(2k +1) = 0
𝑏 −{−(𝑘+6)}
Now, sum of roots = α + β = -𝑎 = - = (k + 6)
1
𝑐 2(2𝑘 + 1)
product of roots = αβ = 𝑎 = = 2(2k + 1) 1
1
A/C to question,
1
sum of roots ( zeros ) = 2 × products of roots zeros
1 1
⇒ (k + 6) = 2× 2(2k + 1)
⇒ (k + 6) = (2k + 1)
⇒ k + 6 = 2k + 1
⇒k=5
Hence, k = 5 1
28 Let length and breadth of rectangle be x unit and y unit.
Area = xy
According to the question, ½
⇒ (x - 5) (y + 3) = xy - 9
⇒ 3x - 5y - 6 = 0 ... (i)
67
⇒ (x + 3) (y + 2) = xy + 67 ½
⇒ 2x - 3y – 61 = 0 ... (ii)
2x + 3y = 61 …(2)*3
3x - 5y = 6 …(1)*2 1
Cross multiply the coefficient of x we get
6x + 9 y = 183
6x -10y =12
Subtract now we get
19 y = 171
Y = 171/19 = 9
Plug this value of y in equation first we get 1
2x + 3* 9 = 61
2x = 61 – 27
2x = 34
X = 34/2 = 17
So length is 17 units and width is 9 units
⇒ x = 17, y = 9
OR
Let the numerator be x and the denominator be y
𝑥
fraction = 𝑦 ------- (a)
1
Given that , Sum of a numerator and the denominator of a fraction is 4 more than
twice the numerator
x + y = 2x + 4
2x - x - y = -4
x - y = -4 -------- (1)
Also given that If the numerator and denominator are increased by 3, they are in
the ratio 2:3
(x + 3) : (y + 3) = 2 : 3

2y + 6 = 3x + 9 1
3x - 2y = 6 - 9
3x - 2y = -3 ------- (2)
Multiplying equation (1) by 2
2(x - y) = 2(-4)
2x - 2y = -8 -------- (3)
Subtracting equation (3) from (2)
3x - 2y -(2x - 2y) = -3 - (-8 )
3x - 2y - 2x + 2y = -3 + 8
x = 5 ----- (4)
Hence numerator is 5
Putting x = 5 from (4) to (1)
x - y = -4
5 - y = -4
y=5+4
y = 9 ------ (5)
Hence denomination is 9
5
Hence the fraction is 9 1

68
29 We know that tangents drawn from
a point outside the circle subtend equal angles at the
centre.

In the above figure, P, Q, R, S are points of contact 1


AS = AP (The tangents drawn from an external
point to a circle are equal.)
∠SOA = ∠POA or∠1 = ∠2 (Tangents drawn from
a point outside of the circle, subtend equal angles at
the centre)
Similarly,
∠3 = ∠4, ∠5 = ∠6, ∠7 = ∠8
Since complete angle is 360° at the centre,
We have,
∠1 + ∠2 + ∠3 + ∠4 + ∠5 + ∠6 + ∠7 + ∠8 = 360°
2 (∠1 + ∠8 + ∠4 + ∠5) = 360° (or) 2 (∠2 + ∠3 + ∠6 + ∠7) = 360° 1
∠1 + ∠8 + ∠4 + ∠5 = 180° (or) ∠2 + ∠3 + ∠6 + ∠7 = 180°
From above figure,
∠1 + ∠8 = ∠AOD, ∠4 + ∠5 = ∠BOC and ∠2 + ∠3 = ∠AOB, ∠6 + ∠7 = ∠COD
Thus we have,
∠AOD + ∠BOC = 180° (or) ∠AOB + ∠COD = 180°
∠AOD and ∠BOC are angles subtended by opposite sides
of quadrilateral circumscribing a circle and the sum of the two is 180°.
OR
Given, a circle touches the side BC of a triangle ABC at P 1
The circle touches the extended sides AB and AC of the triangle at Q and R.
1
We have to prove that AQ = 2(BC + CA + AB)
We know that the tangents drawn through an external point to a circle are equal.
So, BP = BQ --------------- (1)
CP = CR -------------------- (2)
AQ = AR -------------------- (3)
1
We know that the perimeter of a triangle is the sum of all the three sides of a
triangle.
Perimeter of triangle ABC = AB + BC + AC

From the figure,


BC = BP + PC
So, AB + BC + AC = AB + (BP + PC) + AC
From (1) and (2),
= AB + BQ + CR + AC
From the figure,
AB + BQ = AQ [Symmetric Property]
CR + AC = AR [Symmetric Property]
As the tangents drawn through an external point to a circle are equal
1
BQ + CR = BC
So, AB + BC + AC = AQ + AR
From (3),
AB + BC + AC = AQ + AQ
AB + BC + AC = 2AQ
So, AQ = 1/2(AB + BC + AC)
69
Therefore, it is proven that AQ = 1/2(AB + BC + AC) 1
30 Tanθ+sinθ=m and tanθ-sinθ=n
∴, m²-n²
=(m+n)(m-n) 1
=(tanθ+sinθ+tanθ-sinθ)(tanθ+sinθ-tanθ+sinθ)
=(2tanθ)(2sinθ)
=4tanθsinθ 1
4√𝑚𝑛 =4√(𝑡𝑎𝑛𝜃 + 𝑠𝑖𝑛𝜃)(𝑡𝑎𝑛𝜃 − 𝑠𝑖𝑛𝜃) =4√(𝑡𝑎𝑛²𝜃 − 𝑠𝑖𝑛²𝜃)
𝑠𝑖𝑛²𝜃 1
=4√{(𝑐𝑜𝑠 2𝜃) − 𝑠𝑖𝑛²𝜃} =4√𝑠𝑖𝑛²𝜃 {(𝑐𝑜𝑠 2𝜃) − 1}

=4sinθ√{(1 − 𝑐𝑜𝑠²𝜃)/𝑐𝑜𝑠²𝜃} =4sinθ√(𝑠𝑖𝑛²𝜃/𝑐𝑜𝑠²𝜃)


=4sinθ√𝑡𝑎𝑛²𝜃 =4sinθtanθ
LHS=RHS (Proved) 1
31 Total outcome = 52 1
(a)Favourable outcome = 1 for
1
Probability the card drawn is spade and an ace = 52 each
(b) Favourable outcome = 8
8 2
Probability the card drawn is either a king or a queen = 52 = 13
(c) Favourable outcome = 2
2 1
Probability the card drawn is a red king = 52 = 26
Section D
32 Let the speed of the slow train be x km/h.
And the speed of the fast train be (x + 10) km/h.
Time taken by slow train to cover 600 km = 600/x hrs
Time taken by fast train to cover 600 km = 600/(x + 10) hrs.
According to the Question, 2
𝟔𝟎𝟎 𝟔𝟎𝟎
⇒ 𝒙 - 𝒙+𝟏𝟎 = 3
⇒ 600(x + 10) - 600x/x(x + 10) = 3
⇒ 6000/x² + 10x = 3
⇒ 3(x² + 10x) = 6000
⇒ x² + 10x - 2000 = 0
1
By using factorization method, we get
⇒ x² + 50x - 40x - 2000 = 0
⇒ x(x + 50) - 40(x + 50) = 0
⇒ (x + 50) (x - 40) = 0
1
⇒ x + 50 = 0 or x - 40 = 0
⇒ x = - 50, 40 (As speed can't be negative)
⇒ x = 40 km/h
Speed of slow train = x = 40 km/h
1
Speed of fast train = x + 10 = 40 + 10 = 50 km/h
OR
Let the tens place digit be x
14 2
And the units place digit be be 𝑥
14
Number = 10x + 𝑥
14
Interchanged number = 10 × +x
𝑥 1
According to the Question,
70
14 14
⇒ 10x + 𝑥 + 45 = 10 × 𝑥 + x
⇒ 10x + 14/x + 45 = 140/x + x
⇒ 9x - 126/x + 45 = 0
⇒ 9x² + 45x - 126 = 0
Dividing equation by 9 , we get
1
⇒ x² + 5x - 14 = 0
By using factorization method, we get
⇒ x² + 7x - 2x - 14 = 0
⇒ (x² + 7) (x - 2) = 0
⇒ x = - 7, 2 (As x can't be negative)
1
⇒x=2
Number = 10x + 14/x = 10 × 2 + 14/2 = 20 + 14/2 = 27
33 Fig and construction

Produce AD to E so that AD = DE. Join CE


Similarly, produce PM to N such that PM = MN , and join RN.
In ΔABD and ΔCDE
AD = DE [By Construction] 1
BD = DC [AD is the median]
∠ADB = ∠CDE [Vertically opposite angles]
Therefore, ΔABD ≅ ΔECD [By SAS criterion of congruence]
⇒ AB = CE [CPCT] ...(i) 1
Also, in ΔPQM and ΔMNR
PM = MN [By Construction]
QM = MR [PM is the median]
∠PMQ = ∠NMR [Vertically opposite angles]
Therefore, ΔPQM = ΔNRM [By SAS criterion of congruence]
⇒ PQ = RN [CPCT]...(ii) 2
Now,
AB / PQ = AC / PR = AD / PM [Given]
⇒ CE / RN = AC / PR = AD / PM [from (i) and (ii)]
⇒ CE / RN = AC / PR = 2AD / 2PM
⇒ CE / RN = AC / PR = AE / PN [ 2AD = AE and 2PM = PN ]
Therefore, ΔACE ~ ΔPRN [By SSS similarity criterion]
Therefore, ∠CAE = ∠RPN
Similarly, ∠BAE = ∠QPN
Hence, ∠CAE + ∠BAE = ∠RPN + ∠QPN
⇒∠BAC = ∠QPR
⇒∠A = ∠P ....(iii)
Now, In ΔABC and ΔPQR
AB/PQ = AC/PR
∠A = ∠P [from (iii)] 1
Therefore, ΔABC ~ ΔPQR [By SAS similarity criterion]

71
34 Diameter (d)=4.2 m
Radius r = 2.1 m
Height (H)=4 m 1
Dimensions of a cone :
radius (r)= 2.1m
height (h)= 2.4 m
slant height (l)=√r²+h²
=√(2.1)²+(2.4)² 2
=√4.41+5.76
=√10.17
≈ 3.18 m
Now, outer surface area of building = CSA of cylinder + CSA of cone
= 2rH + rl
= r(2H + l)
22
= 7 x2.1(2x4 + 3.18)
= 22x0.3x11.18
=73.78 m2
Volume of the building = vol. of cylinder + vol. of cone
= πr²H + 1/3 πr²h
= πr² (H+1/3h)
22 2
= 7 x 2.1 x 2.1(4+2.2/3)
= 22x0.3x2.1x14.2/3= 65.6m3
OR
Total height of solid= 19 cm
Diameter of Cylinder and hemisphere = 7 cm
Radius of Cylinder = radius of hemisphere= 7/2= 3.5 cm
Height of hemisphere = radius of hemisphere = 7/2 cm
Height of 2 hemisphere = 2× 7/2= 7 cm
Height of Cylinder= Total height of solid - Height of 2 hemisphere
Height of Cylinder(h) = 19 - 7 = 12 cm
Volume of solid,V = Volume of Cylinder + Volume of two hemispheres 1
V = πr²h + 2 × 2/3πr³
V = πr²h + 4/3πr³
V = πr²(h + 4/3 r)
V = 22/7 × 7/2 × 7/2 (12 + 4/3 × 7/2)
V = 77/2 (12 + 14/3) 2
V = 77/2 [(36+14)/3]
V = 77/2 [50/3)
V = (77 × 25)/3
V = 1925/3 = 641.67
V = 641.67 cm³
Volume of solid = 641.67 cm³
Total surface area of the solid = surface area of cylinder + surface area of two
hemispheres
= 2πrh + 2(2πr²) 2
= 2πrh + 4πr²
= 2πr(h+2r)
= 2×(22/7)×7/2 (12 + 2 × 7/2)
= 22 (12 + 7)
72
= 22 × 19
= 418 cm²
Total surface area of the solid = 418 cm²
35 C. I. f 𝑥𝑖 𝑓𝑖 𝑥𝑖 3
0-80 20 40 800
80-160 25 120 3000
160-240 𝑓1 200 200𝑓1
240-320 𝑓2 280 280𝑓2
320-400 10 360 3600
total 55+𝑓1 +𝑓2 7400 + 200𝑓1 + 280𝑓2
∑ 𝑓𝑖 𝑥𝑖
Mean = ∑ 𝑓𝑖
1
7400 + 200𝑓1 + 280𝑓2
188 = 100

7400 + 200𝑓1 + 280𝑓2 = 18800


200𝑓1 + 280𝑓2 = 11400
5𝑓1 + 7𝑓2 = 285 ………..(i)
Total frequency = 100
55+𝑓1 + 𝑓2 = 100
𝑓1 + 𝑓2 = 45 ………………(ii) 1
Solving eq (i), (ii)𝑓1 = 15 and 𝑓2 = 30 1
Section E
𝐸𝐶
36 (i) In ∆EDC, cos300 = 𝐸𝐷 1+1
√3 30√3
+2
= ,⇒ ED = 60 m
2 𝐸𝐷
𝐷𝐶
(ii)In ∆EDC, tan300 = 𝐸𝐶
1 𝐷𝐶
= 30 ,⇒DC = 30 m
√3 √3

Height of tower = DC + BC = 30 + 1.5 = 31.5 m


(ii) distance he walk towards the tower = 20√3m
OR
Increased height of tower = 91.5 m
𝑛
37 (i)𝑠𝑛 = 2 {2a+(n-1)d}) 1+1
30 +2
6150 2 (2a+29x10)
A = 60
(ii) 𝑎𝑛 = a + (n - 1)d
250 = 60 + (n-1)10
N = 19
(iii)𝑎15 = a +14d
= 60 +14x10 = 200
Total sale = 200x50 = Rs10000
OR
𝑎31 = a +30d
= 60 +30x10 = 360
Loss in sale = 360x50 = Rs18000

73
38 (i)Co-ordinate of A(0,8) and B(-3, 6) 1+1
Length of line segment AB = √(0 − 3)2 + (8 − 6)2 +
= √(−3)2 + (2)2 = √13 unit
(ii)Co-ordinate of mid point of BE = (-3, 4)
(iii)Let ratio be 1:k
Given BD divide by y-axis so by section formula
3−3𝑘
0 = 𝑘+1 , k=1
So the ratio is 1:1
OR
Ratio in which BG divides by y-axis =1:1

74
Class- X Session- 2022-23
Subject- Mathematics (Standard)
Sample Question Paper 3
Time Allowed: 3 Hrs. Maximum Marks: 80

General Instructions: As per the previous sample paper


S No Section A MARKS
1 The least number which is a perfect square and is divisible by each of 1
16, 20 and 24 is
(a) 240 (b) 1600 (c) 2400 (d) 3600
2 Find the greatest possible length which can be used to measure exactly 1
the length 7 m, 3 m 85 cm and 12 m 95 cm.
(a) 40 (b) 25 cm (c) 35 cm (d) none of these
3 Assertion : The HCF of two numbers is 5 and their product is 150, then 1
their LCM is 30
Reason : For any two positive integers a and b,HCF( a b)+LCM(a,b) =
a× b
(a) Both assertion (A) and reason (R) are true and reason (R) is the
correct explanation of assertion (A).
(b) Both assertion (A) and reason (R) are true but reason (R) is not the
correct explanation of assertion (A).
(c) Assertion (A) is true but reason (R) is false.
(d) Assertion (A) is false but reason (R) is true.
4 If a and b are the zeroes of a polynomial such that a + b = – 6 and ab = – 1
4, then write the polynomial.
(a) x2 – 6x – 4 (b) x2 + 6x – 4 (c) x2 + 6x + 4 (d) x2 – 6x + 4
5 The system of equations 1
2x + 3y = 7
2ax + (a + b) y = 28
has infinitely many solutions, then
(a) a = 2b (b) b =2a (c) a + 2b = 0 (d) 2a + b = 0
6 The difference in the roots of the equation 2x2 – 11x + 5 = 0 is: 1
(a) 4.5 (b) 4 (c) 3.5 (d) 3
7 If the points A(4, 3) and B(x, 5) lies on a circle with the centre O(2,3) 1
then the value of x is
(a) 1 (b) 3 (c) 2 (d) 9
8 Assertion: The value of y is 6, for which the distance between the points 1
P(2,-3) and Q(10,Y) IS 10 .
Reason: Distance between two given points A(x1,y1) and B(x2,y2)) is
given , AB =√(𝑥2 − 𝑥1 )2 + (𝑦2 − 𝑦1 )2
(a) Both assertion (A) and reason (R) are true and reason (R) is the
correct explanation of assertion (A).
(b) Both assertion (A) and reason (R) are true but reason (R) is not the
correct explanation of assertion (A).
(c) Assertion (A) is true but reason (R) is false.
(d) Assertion (A) is false but reason (R) is true
9 In ∆ABC, DE || BC and AD = 4cm, AB = 9cm. AC = 13.5 cm then the value 1
75
of EC is
(a) 6 cm (b) 7.5 cm (c) 9 cm (d) none of these
10 In triangle ABC, a line XY parallel to BC cuts AB at X and AC at Y. If BY 1
bisects ∠𝑋𝑌𝐶, then
(a) BC = CY (b) BC = BY (c) BC ≠ CY (d) BC ≠ BY
11 A vertical tree of 12 m long casts a shadow 8 m long on the ground. At 1
the same time, a tower casts the shadow 40 m long on the ground.
Determine the height of the tower.
(a) 30m (b) 60m (c) 80m (d) 10m
12 𝑠𝑖𝑛 θ − 4𝑐𝑜𝑠 θ 1
If 4 cotθ = 3, then
𝑠𝑖𝑛 θ + 4𝑐𝑜𝑠 θ
2 2 1 3
(𝑎 ) − (b) 7 (c)3 (d) 4
7
1 1
13 If sin α = 2 and cos β = 2, then the value of (α + β) 1
(a) 0° (b)30° (c)60° (d) 90°
14 (cos4 A – sin4 A) = 1
(a) 1 – 2 cos2 A (c) sin2 A – cos2 A
(b) 2sin2 A – 1 (d) 2cos2 A – 1
15 If the perimeter of a circle is equal to that of a square, then the raito of 1
their areas is
(a) 22:7 (b) 14:11 (c) 7:22 (d) 11:14
16 In a circle of radius 14 cm, an arc subtends an angle of 45° at the centre, 1
then the area of the sector is
(a) 71 cm2 (c) 77 cm2
(b) 76 cm 2 (d) 154 cm2
17 The raio of lateral surface area to the total surface area of a cylinder 1
with base diameter 1.6m and height 20cm is
(a) 1:7 (b) 1:5 (c) 7:1 (d) 5:1
18 For the following distribution. 1
Class 0–5 5 – 10 10 – 15 15 – 20 20 – 25
Frequency 10 15 12 20 9
The sum of lower limits of the median class and modal class is
(a) 15 (c) 30
(b) 25 (d) 35
19 Mode of the following grouped frequency distribution is 1
Class Frequency
3–6 2
6–9 5
9 – 12 10
12 – 15 23
15 – 18 21
18 – 21 12
21 – 24 03
(a) 13.6 (c) 14.6
(b) 15.6 (d) 16.6
20 Two dice are thrown together. The probability that sum of the two 1
numbers will be a multiple of 4, is:
1 1
(a) 2 (c) 8
1 1
(b) 3 (d)4
76
SECTION - B
21 Two concentric circles are of radii 7 cm and r cm 2
respectively, where r > 7. A chord of the larger
circle, of length 48 cm, touches the smaller
circle. Find the value of r.

22 Determine the value of k, for which the given system of equations has 2
infinitely many solutions: kx + 3y = k – 3 and 12x + ky = k.
23 Students of a school decided to participate in ‘Save girl 2
child’ campaign. They decided to decorate a triangular
path as shown. If AB = AC and BC2 = AC × CD, then
prove that BD = BC.

24 If sin θ + cos θ = √2cos θ (θ ≠ 90°) then what is the value of tan θ? 2


OR
If √3 sin θ - cos θ = 0, find θ.
25 Inthe given figure, a chord AB of the circle with centre O and radius 10 2
cm, that subtends a right angle at the centre of the circle. Find the area
of the minor segment AQBP. (Use π = 3.14)

OR
In Figure, PQ and AB are two arcs of
concentric circles of radii 7 cm and 3.5
cm respectively, with center O. If ∠POQ =
30°, then find the area of shaded region.
SECTION - C
26 Prove that 4−3√𝟐 is an irrational. Given that √2 is irrational. 3
27 If α and β are the zeroes of the polynomial f (x )= x2-4x-5 then find the 3
value of α2+ β2
28 Find two consecutive numbers whose squares have the sum 85. 3
OR
A charity trust decides to build the player hall having a carpet area of
300 sq m with its length 1 m more than twice its breadth. Find the
length and breadth of the hall.
29 Prove that the parallelogram circumscribing a 3
circle is a rhombus.
OR

In the given figure, PA and PB are tangents to the


circle from an external point P. CD is another
tangent touching the circle at Q. If PA = 12 cm, QC = QD = 3cm, then
find PC + PD.
77
30 1 − 𝑐𝑜𝑠𝜃 3
Prove that(cot𝜃 −cosec𝜃)2 = 1+ 𝑐𝑜𝑠𝜃
31 Two unbiased coins are tossed simultaneously. Find the probability of 3
getting: (i)At least one head (ii)At most one head (iii)No head.
SECTION - D
32 Solve for x 5
2𝑥 1 3𝑥+9
+ 2𝑥+3 + (𝑥−3)(2𝑥+3) =0 , x ≠ –3/2,3
𝑥−3
OR
Find a natural number whose square diminished by 84 is equal to thrice
of 8 more than the given number.
33 In fig. PA, QB and RC are each perpendicular to AC. 5
1 1 1
Prove that 𝑥 + 𝑧 = 𝑦.

34 A cubical block of side 10 cm is surmounted by a hemisphere. What is 5


the largest diameter that the hemisphere can have? Find the cost of
painting the total surface area of the solid so formed, at the rate of Rs.5
per 100 sq. cm. (Use π = 3.14)
OR
Asolid is in the shape of a hemisphere surmounted by a cone. If the
radius of hemisphere and base radius of cone is 7 cm and height of cone
is 3.5 cm, find the volume of the solid.
35 The mean of the following frequency distribution is 62.8 and sum of all 5
the frequencies is 50. Find missing frequencies f 1 and f2.
Class 0 – 20 20–40 40–60 60 – 80 80 – 100 100–120
Frequenc 5 f1 10 f2 7 8
y
SECTION -E
36 Carpooling: It is the sharing of car journeys so that more than one 1+2+1
person travels in a car and prevents the need for others to have to drive
to a location themselves. By having more people using one vehicle,
carpooling reduces each person’s travel costs such as: fuel costs, tolls,
and the stress of driving.
Carpooling is also a more
environmentally friendly and
sustainable way to travel as
sharing journeys reduces air
pollution, carbon emissions,
traffic congestion on the roads,
and the need for parking space.

Three friends Amar, Bandhu and Chakradev live in societies represented


by the points A, B and C respectively. They all work in offices located in a
same building represented by the point O. Since they all go to same
building every day, they decided to do carpooling to save money on
petrol.

78
Based on above information, answer the following questions.
(i) Which society is nearest to the office?
(ii) What is the distance between A and C?
OR
Find the least distance between AB, OA and BC?
(iii) Find the best route to go to the office?
37 The school auditorium was to be constructed to accommodate at least 1+2+1
1500 people. The chairs are to be placed in concentric circular
arrangement in such a way that each succeeding circular row has 10
seats more than the previous one.

(i) If the first circular row has 30 seats, how many seats will be therein
the10th row?
(ii) For 1500seats in the auditorium, how many rows need to be there?
OR
If1500 seats are to be arranged in the auditorium, how many seats are
still left to be put after 10th row?
(iii) If there were 17 rows in the auditorium, how many seats will be
there in the middle row?
38 When an eagle looks at a rat on the ground, eagle does not attack the rat 1+2+1
at its initial position. It takes into account the speed of the rat and the
direction in which rat is moving. After analysing the situation (how? it
may be God’s gift) eagle attacks the rat in such a way that it may
successful in catching the rat.

79
Suppose an eagle sitting on the tree of height 90 m, observe a rat with
angle of depression 45°, and moving away from the tree with some
speed. Eagle start flying with 30° downward and catch rat in 10 second.
(i) What is the horizontal distance between tree and initial
position of rat?
(ii) What is the distance travelled by rat in 10 seconds?
OR
What is the distance travelled by eagle to catch rat?
(iii) What is the speed of rat?

80
Marking Scheme Sample Paper 3
S No Section A MARKS
1 d 1
2 c 1
3 c 1
4 b 1
5 b 1
6 a 1
7 c 1
8 d 1
9 b 1
10 a 1
11 b 1
12 a 1
13 d 1
14 d 1
15 b 1
16 c 1
17 b 1
18 b 1
19 c 1
20 d 1
21 Angle between radius and tangent is 900, angle .
OC ┴ AB
1
AC = 2 AB{perpendicular from centre bisect
1
chord)
1
AC = 2 x 48 = 24
In right angle triangle OAC
OA2 = OC2 + AC2 1
𝑟 2 = 72 + 242 = 49 + 576 = 625
So, r = 25
22 Given, kx + 3y – (k – 3) = 0 …(i) and
12x + ky – k = 0 …(ii)
For this pair of equations to have infinite number of solutions, it must satisfy
the condition of = k/12 = 3/k=(k-3)/k
Take k/12 = (k-3)/k
⇒ k2 – 12k + 36 = 0 1
⇒ k2– 6k – 6k + 36 = 0
⇒ k(k – 6) – 6(k – 6) = 0
⇒ (k – 6)2 = 0 ⇒ k = 6 1
Also k/12 = 3/k
k2 = 36, k = ± 6
Common value k = 6 Thus, the pair of equations will have infinite number of
solutions for k = 6.
23 In △ABC and △BCD
BC2 = AC × CD [Given]
𝐵𝐶 𝐴𝐶
=
𝐶𝐷 𝐵𝐶 1
∠C = ∠C [Common] DABC ~ DBDC [By SAS criterion
𝐴𝐵 𝐴𝐶
= [by c.p.c.t.]
𝐵𝐷 𝐵𝐶
But , AB = AC [Given] 1

81
BD = BC Hence Proved.
24 Dividing both side by cosθ, we get
tan θ + 1 = √2 1
tan θ = √2 – 1 1
OR
Dividing both side by cosθ, we get 1
√3 tan θ – 1 = 0 1
1
tan θ = = tan 30°
√3
θ = 30°
25 Area of minor segment = Area of sector OAPB – Area of ∆OAB
90 1
= 360π(10)2 - 2 x 10 x 10 1
1
= 78.5 – 50 = 28.5 cm2
OR
𝜃 1
Area of shaded region = 360π (R – r )
2 2

30° 22
=360° x 7 (72 – (3.5)2) = 9.625 cm2 1
26 let 4−3√𝟐be rational.
4−3√𝟐=a/b (where a and b are coprime)
𝑎−4𝑏 1
Then −3√2=a/b-4= 𝑎
𝑎−4𝑏
that √2=
−3𝑎
since
𝑎−4𝑏
is a rational number so √2 is also a rational number 1
−3𝑎
but we know that √2 𝑖𝑠 𝑖𝑟𝑟𝑎𝑡𝑖𝑜𝑛𝑎𝑙 1
this contradict that our assumption is wrong
hence 4−3√2 𝑖𝑠 𝑖𝑟𝑟𝑎𝑡𝑖𝑜𝑛𝑎𝑙
27 α +β=4 , αβ=-5 1
α2 + β2 = (α +β )2-2 αβ 1
42-2(-5)=26 1
28 Let the two numbers be x and (x + 1).
Thus, x2 + (x + 1)2 = 85
⇒ x2 + x2 + 2x + 1 = 85 1
⇒ 2x2 + 2x – 84 = 0
⇒ x2 + x – 42 = 0
⇒ x2 + 7x – 6x – 42 = 0 1
⇒ x(x + 7) – 6(x + 7) = 0
⇒ (x + 7)(x – 6) = 0
⇒ x = 6, – 7 1
Thus, the two numbers are either 6 and 7 or – 7 and – 6
OR
. Let breadth of the hall be x m
∴ Length of the hall = (2x + 1) m 1
Since, Area = l × b ⇒ (2x + 1) × x = 300
⇒ 2x2 + x – 300 = 0 1
⇒ 2x2 + 25x – 24x – 300 = 0
⇒ x(2x + 25) –12(2x + 25) = 0
⇒ (x – 12) (2x + 25) = 0
25
⇒ x = 12, x = − 2 (Rejected) ∴ x = 12 m
1
⇒ breadth = 12 m, Length = (2 × 12 + 1) = 25 m

82
29 Given: ABCD is a parallelogram circumscribed in a circle
AB=CD........(1)
BC=AD........(2)
To Prove: ABCD is a rhombus. 1
Proof:
DR=DS (Tangents on the circle from same
point D)
CR=CQ(Tangent on the circle from same
point C)
BP=BQ (Tangent on the circle from same point B )
AP=AS (Tangents on the circle from same point A)
Adding all these equations we get
DR+CR+BP+AP=DS+CQ+BQ+AS
(DR+CR)+(BP+AP)=(CQ+BQ)+(DS+AS)
CD+AB=AD+BC
Putting the value of equation 1 and 2 in the above equation we get
2
2AB=2BC
AB=BC...........(3)
From equation (1), (2) and (3) we get
AB=BC=CD=DA 1
∴ABCD is a Rhombus
Or
Given, PA = PB = 12 cm [Tangents from same point P]
AC = CQ = 3 cm [Tangents from same point C] 2
BD = QD = 3 cm [Tangents from same point D]
So, PC + PD = (PA – AC) + (PB – BD) = (12 – 3) + (12 – 3) = 9 + 9 =
18 cm. Ans.
30 L.H.S. = (cot𝜃 - cosec𝜃 )2
𝑐𝑜𝑠𝜃 1 𝑐𝑜𝑠𝜃 − 1 1 − 𝑐𝑜𝑠𝜃
=( − )2 = ( )2 = ( )2
𝑠𝑖𝑛𝜃 𝑠𝑖𝑛𝜃 𝑠𝑖𝑛𝜃 𝑠𝑖𝑛𝜃
(1 − 𝑐𝑜𝑠𝜃)2 (1 − 𝑐𝑜𝑠𝜃)2
1
= =
𝑠𝑖𝑛2𝜃 1 − 𝑐𝑜𝑠 2 𝜃
(1 − 𝑐𝑜𝑠𝜃)(1 − 𝑐𝑜𝑠𝜃)
=
(1 − 𝑐𝑜𝑠𝜃)(1 + 𝑐𝑜𝑠𝜃) 1
(1 − 𝑐𝑜𝑠𝜃)
= (1 + 𝑐𝑜𝑠𝜃) = R.H.S.
1
3
31 (i) P (at least one head) = 4 1
3
(ii) P (at most one head) = 4 1
1
(iii) P (no head) = 4 1
32 2𝑥 1 3𝑥+9
𝑥−3
+ 2𝑥+3 + (𝑥−3)(2𝑥+3) =0
⇒ 2x (2x + 3) + (x – 3) + (3x + 9) = 0 1
⇒ 4x2 + 6x + x – 3 + 3x + 9 = 0 1
⇒ 4x2 + 10x + 6 = 0
⇒ 2x2 + 5x + 3 = 0 1
⇒ 2x2 + 2x + 3x + 3 = 0
⇒ 2x(x + 1) + 3 (x + 1) = 0 1
⇒ (2x + 3) (x + 1) = 0
x=-3/2 (Rejecting) 1
x=-1
OR
Let the natural number be x.
83
According to the given details, the quadratic equation formed is 1
x2- 84 = 3(x+8)
On simplifying we get, 1
⇒ x2- 3x– 84 – 24 = 0
⇒ x2- 3x - 108= 0 1
⇒ x2- 12x + 9x - 108= 0
⇒x(x– 12) + 9(x – 12) = 0
⇒ (x + 9) (x – 12) = 0 1
So, x = -9, 12 1
We know that natural numbers are positive. So, x=-9 is not possible.
Hence, the required natural number is 12.
33 Given: PA, QB and RC are each perpendicular to AC.

To Prove: 1

Proof:
In ∆PAC, we have BQ∥AP

⇒y/x=CB/CA ......(1)
Now in ∆ACR we have BQ∥CR 1

…… (2) 1
Adding (1) and (2) we have

1
.
34 Diameter of the sphere = 10 cm
Radius of the sphere = 5 cm 1
Total surface area of the solid = Total surface area of the cube – Inner cross –
section area of the hemisphere + curved surface area of the hemisphere 1
= 6a2 – π2 + 2 πr2 1
= 6a2 + πr2 1
= 6 x (10)2 + 3.14 x 52 1
= 600 + 78.5 = 678.5 cm2
5
Cost of painting the total surface area of the solid = (100) x 678.5 = Rs. 33.925
OR

84
1
r = 7 cm 1
height of the cone = 3.5 cm 1
Volume of the solid = Volume of hemisphere + Volume of a cone
2 1
= 3πr3 + 3πr2h 1
= x
2 22 1
x 73 + x
22
x 73 x 3.5 1
3 7 3 7
1
= 3 (2156 + 539)
1
= 3 x 2695 = 898.33 cm2
35 It is given that mean = 62.8 and N=50.
Let the assumed mean A = 50 and h = 20.

.....(1)

1
....(2)
Putting the value of in (2), we get

85
Putting the value of in (1), we get

Hence, the missing frequency f1 = 8 and f2 = 12.


36 (i) From the graph, society C is nearest to the office. 1
(ii) Distance between A and C,
= √(5 − 2)2 + (3 − 8)2
= √9 + 25 = √34 units 2
OR
AB = √(7 − 2) + (7 − 8)2 = √26 units
2

OA = √22 + 82 = 2√17 units


BC = √(7 − 5)2 + (7 − 3)2 = 2√5 units
Distance BC is the least.
(iii) From graph, ABCO is the best route to go to office 1
37 (i) Since each row is increasing by 10 seats, so it is an AP with first term a =
30 & d = 10
10th row = a + 9d
10th row = 30 + 9x10 = 120 1
𝑛
(ii) Sn= 2 (2a + (n - 1)d)
𝑛
1500 = (2 x 30 + (n - 1)10)
2
3000 = 50n + 10n2
n2 + 5n – 300 = 0
(n + 20) (n - 15) = 0 2
Rejecting the negative value, n = 15.
OR
No. of seats already put up to 10th row= S10
10
S10 = (2 x 30 + (10 - 1) 10)
2
5 (60 + 90) = 750
So, the number of the seats still required to be put are 1500 – 750 = 750
(iii) If no. of rows = 17 then the middle row is the 9th row
a8 = a + 8d 1
a8 = 30 + 80 = 110 seats.
38
𝑂𝐸
(i) = tan 45° = 1
𝑂𝑅
1
OE = OR = 90 m
𝑂𝐸
(ii) 𝑂𝑆
= tan 30°
90 1
𝑂𝑆
=
√3
2
OS = 90√3
RS = OS – OR
RS = 90√3 – 90 = 90(√3 - 1) m

OR
𝐸𝑂
= sin 30°
𝐸𝑆
90
=2
1 1
𝐸𝑆
ES = 180 m
𝑅𝑆 90(√3 − 1)
(iii) Speed of rat, 𝑡
= 10
= 9(√3 - 1) m/sec

86
Class- X Session- 2022-23
Subject- Mathematics (Standard)
Sample Question Paper 4
Time Allowed: 3 Hrs. Maximum Marks : 80
General Instructions: As per the previous question papers
Section A (1 Mark each)
S. No Questions MARKS
1 On a morning walk, three persons step off together and their steps 1
measure 40 cm, 42 cm and 45 cm, respectively. What is the minimum
distance each should walk so that each can cover the same distance
in complete steps?
(A) 2520cm (B) 2525cm
(C) 2555cm (D) 2528cm
2 If sum of the squares of zeroes of the quadratic polynomial 6x 2 + x + 1
k is 25/36, the value of k is:
(A) 4 (B) – 4 (C)2 (D) – 2
3 If the lines given by 1
3x + 2ky = 2
2x + 5y + 1 = 0
are parallel, then the value of k is
(A) 5/4 (B) 2/5 (C) 15/4 (D) 3/2
4 1

(A) 3.5 (B) 4 (C) 3 (D) – 3


5 Assertion (A): Mid-point of a line segment divides line in the ratio 1
1 : 1.
Reason (R): The ratio in which the point (–3, k) divides the line
segment joining the points (–5, 4) and (–2, 3) is 1 : 2
(A) Both assertion (A) and reason (R) are true and reason (R) is the
correct explanation of assertion (A).
(B) Both assertion (A) and reason (R) are true but reason (R) is not
the correct explanation of assertion (A).
(C) Assertion (A) is true but reason (R) is false.
(D) Assertion (A) is false but reason (R) is true
6 ABCD is a trapezium in which AB|| DC and P, Q are points on AD and 1
BC respectively such that PQ || DC. If PD = 18 cm, BQ = 35 cm and QC
= 15 cm, find AD.
(A) 55cm (B) 57cm (C) 60cm (D) 62cm
7 The coordinates of the centroid of a triangle whose vertices are (0, 1
6), (8,12) and (8, 0) is
(A) (4, 6) (B) (16, 6) (C) (8, 6) (D) (16/3, 6)
8 If secθ + tanθ = x, then tanθ is: 1
(A) (x2-1) / 2x (B) (x2+1) / 2x (C) (x2-1) / x (D)
(x +1)/x
2

9 Express 98 as a product of its primes 1


(A) 2² × 7 (B) 2² × 7² (C) 2 × 7² (D) 23 × 7
87
10 Assertion: For any two positive integers p and q, HCF (p, q) × LCM 1
(p, q) = p × q
Reason: If the HCF of two numbers is 5 and their product is 150, then
their LCM is 40
(A) Both assertion (A) and reason (R) are true and reason (R) is the
correct explanation of assertion (A).
(B) Both assertion (A) and reason (R) are true but reason (R) is not
the correct explanation of assertion (A).
(C) Assertion (A) is true but reason (R) is false.
(D) Assertion (A) is false but reason (R) is true.
11 If the length of an arc of a circle of radius r is equal to that of an arc 1
of a circle of radius 2r, then
(A) the angle of the corresponding sector of the first circle is double
the angle of the corresponding sector of the other circle.
(B) the angle of the corresponding sector of the first circle is equal
the angle of the corresponding sector of the other circle.
(C) the angle of the corresponding sector of the first circle is half the
angle of the corresponding sector of the other circle.
(D) the angle of the corresponding sector of the first circle is 4 times
the angle of the corresponding sector of the other circle.
12 The surface area of a sphere is 616 cm2. Its radius is 1
(A) 7 cm (B) 14 cm (C) 21 cm (D) 28 cm
13 The median and mode respectively of a frequency distribution are 26 1
and 29, Then its mean is
(A) 27.5 (B) 24.5 (C) 28.4 (D) 25.8
14 The median of set of 9 distinct observations is 20.5. If each of the 1
largest 4 observations of the set is increased by 2, then the median of
the new set
(A) is increased by 2 (B) is decreased by 2
(C) is two times of the original number
(D) Remains the same as that of the original set
15 A bag contains 5 red balls and some blue balls .If the probability of 1
drawing a blue ball is double that of a red ball, then the number of
blue balls in a bag is:
(A) 5 (B) 10 (C) 15 (D) 20
16 A vertical pole of length 6 m casts a shadow 4 m long on the ground 1
and at the same time a tower casts a shadow 28 m long. Find the
height of the tower.
(A) 32 (B) 15 (C) 42 (D) 22
17 In given figure, find tan P – cot R. 1
(A) 5
(B) 0
(C) 1
(D) 2
18 If the perimeter and the area of a circle are numerically equal, then 1
the radius of the circle is
(A) 2 units (B) n units (C) 4 units (D) 7 units
19 If x tan 45° sin 30° = cos 30° tan 30°, then x is equal to 1
(A) √3 (B) 12 (C) 2√2 (D) 1

88
𝐴𝐵 𝐵𝐶 𝐶𝐴
20 If in two triangles ABC and DEF, 𝐷𝐹 =𝐸𝐹 = 𝐸𝐷 , then 1
(A) ∆ABC ~ ∆DEF (B) ∆ABC ~ ∆EDF
(C) ∆ABC ~ ∆EFD (D) ∆ABC ~ ∆DFE
Section B
21 Solve the following pairs of linear equations by the substitution 2
𝑠 𝑡
method: s-t=3 and 3 + 2 =6
22 D is a point on the side BC of a triangle ABC such that ADC = BAC, 2
show that CA2 = CB.CD

23 Prove that the perpendicular at the point of contact to the tangent to 2


a circle passes through the centre.
24 Evaluate 2 tan2450+ cos2300-Sin2600 2
OR
1
If tan (A + B) = √3 and tan (A – B) = 3 0° < A + B ≤ 90°; A > B, find

A and B.
25 If the perimeter of a protractor is 72cm,calculate its area 2

OR
The inner circumference of a circular track is 220 m. The track is 7 m
wide everywhere. Calculate the cost of putting up a fence along the
outer circle at the rate of ₹2 per metre.
Section C
26 If one zero of the polynomial 3x2 – 8x + (2k+1) is seven times the 3
others, find the value of k.
27 If -5 is a root of the quadratic equation 2x² + px – 15 = 0 and the 3
quadratic equation p(x² + x) + k = 0 has equal roots, then find the
value of k.
OR
If ad ≠ bc, then prove that the equation
(a² + b²) x² + 2(ac + bd)x + (c²+ d²) = 0 has no real roots.
28 In figure, XY and X’Y’ are two parallel tangents to a circlewith centre 3
O and another tangent AB with point of contact C intersecting XY at A
and X’Y’ at B. Prove that ∠AOB = 90°.

OR

89
In the figure, quadrilateral ABCD is circumscribing a circle with
centre O and AD⊥AB. If radius of incircle is 10cm, then find the
value of x.

29 Prove that 5-√3 is irrational, given that √3 is irrational. 3


30 Prove that (1 – sinA + cosA)2 = 2(1 - sinA)(1 + cosA) 3
31 Two different dice are thrown together. Find the probability that the 3
number obtained :
(i) have a sum 11 (ii) have a product 12.
(iii)is a doublet of prime numbers
Section C
32 A motor boat whose speed is 24 km/h in still water takes 1 hour 5
more to go 32 km upstream than to return downstream to the same
spot. Find the speed of the stream
OR
8
Two taps running together can fill a cistern in 2 11minutes. If one tap
takes 1 minute more than the other to fill the cistern, find the time in
which each tap separately can fill the cistern.
33 Two poles of height a metrs and b metresare p metres apart. Prove 5
that the height of the point of intersection of the lines joining the top
𝑎𝑏
of each pole to the foot of the opposite pole is given by metres.
𝑎+𝑏
34 A tent is in the shape of a cylinder surmounted by a conical top. If the 5
height and diameter of the cylindrical part are 2.1 m and 4 m
respectively, and the slant height of the top is 2.8 m, find the area of
the canvas used for making the tent. Also, find the cost of the canvas
of the tent at the rate of Rs 500 per m2.Also find the volume of air
enclosed in the tent?
OR
A vessel is in the form of an inverted cone. Its height is 8 cm and the
radius of its top, which is open, is 5 cm. It is filled with water up to
the brim. When lead shots, each of which is a sphere of radius 0.5 cm
are dropped into the vessel, one-fourth of the water flows out. Find
the number of lead shots dropped in the vessel.
35 Find the mean and mode of the following frequency distribution: 5
Class 25- 30-35 35-40 40-45 45-50 50-55
Interval 30
Frequency 25 34 50 42 38 14
Section E
36 Mr.Dinkar has just admitted his son Dipu in class I .Dipu is a very
bright student and doing well. His fatherbelongs to lower middle
income group so he has planned to save the certain amount of money
every year onthe name of higher studies of his son. Suppose Dipu
90
never fails in any class and did not
repeat in same class.
As per his plan Dinkar will need an
amount of Rs 5 lakhs for the higher
studies of his son after completion
ofschool (10+2). So he started saving
an amount of Rs 20,000 in the first
year of his study and increased
hisyearly savings by Rs 5,000 each year. When Dipu will pass his
class XII, it will be his choice to decide about his
study.
(i) What will be his saving amount for the year in which Dipu will be
in class XII ?
(ii) Will Dinkarbe able to have his gross saving sufficient for Dipu’s
higher study in the year in whichDipu will pass out Class XII.
OR
What will be the saving after 7thclass.
(iii) How much amount will be left with Dinkar if he paid 5 lakhs to
Dipu for his higher study as per his plan?
37 A school has triangular shaped ground with
vertices
P(0 , 4) , Q(-5 , 0) and R(5 , 0)

(i)what is the length of PQ, QR and PR?


(ii)what is the type of triangular ground of school?
(iii) If Ravi has to post a flag exactly half way between the line
segment joining the twopoints Q and R where should he post his flag?
OR
If a line is drawn with chalk powder from P to the point where flag is
posted. Then, find the distance between P and posted flag?
38 When an eagle looks at a rat on the ground,
eagle does not attack the rat at its initial
position. It takes into account the speed of the
rat and the direction in which rat is moving.
After analysing the situation (how? it may be
God’s gift) eagle attacks the rat in such a way
that it may successful in catching the rat.
Suppose an eagle sitting on the tree of height 90 m, observe a rat
with angle of depression 450, and moving away from the tree with
some speed. Eagle start flying with 300 downward and catch rat in 10
second.
(i)What is the horizontal distance between tree and initial position of
rat?
(ii) What is the distance travelled by eagle to catch rat?
(iii)What is the speed of rat?
OR
What is the speed of eagle?

91
Class- X
Mathematics Standard(041)
Marking Scheme 4
Section A
1 (A)2520 cm 1
2 (D)-2 1
3 (C)15/4 1
4 (C)3 1
5 (C) Assertion (A) is true but Reason (R) is false 1
6 (C)60 cm 1
7 (D)(16/3, 6) 1
8 (A)(x2 - 1)/2x 1
9 (C) 2 × 7² 1
10 (C) Assertion (A) is true but Reason (R) is false 1
11 (A)the angle of the corresponding sector of the first circle is double the angle of 1
the corresponding sector of the other circle.
12 (A)7 cm 1
13 (B)24.5 1
14 (D) Remains the same as that of the original set 1
15 (B)10 1
16 (C)42 1
17 (B)0 1
18 (A)2 units 1
19 (D)1 1
20 (D) ∆ABC ~ ∆DFE 1
Section B
21 s-t=3 ------------------------------------------(i)
𝑠 𝑡
3
+ 2 =6 -----------------------------------(ii)
From equation (i), s=3+t
Putting value of s in equation (ii) we get
3+𝑡 𝑡
+ 2 =6 1
3
2(3+𝑡)+3𝑡
6
=6
6+2t+3t=36
5t=36-6=30
T=30/5 =6
Putting t=6 in equation 1,we get
s=3+6=9 so s=9,t=6
1
22 In ∆ABC and ∆DAC, we have
∠BAC =∠ADC (Given)
and ∠C =∠C(Common)
∆ABC ~ ∆DAC (By AA criterion of similarity) 1
𝐴𝐵 𝐵𝐶 𝐴𝐶
= =
𝐷𝐴 𝐴𝐶 𝐷𝐶

𝐵𝐶 𝐴𝐶
𝐴𝐶
=𝐷𝐶
CA2=CB x CD 1
92
23 Given a circle with center O and AB the tangent intersecting circle at point P
and prove that OP⊥AB
We know that tangent of the circle is perpendicular to radius at points of contact Hence
OP⊥AB
So, ∠OPB =900..........(i)
Now let’s assume some point X 1
Such that XP⊥AN
Hence ∠XPB=900.........(ii)
From eq (i) & (ii)
∠OPB=∠XPB=900
Which is possible only if line XP passes though O 1
Hence perpendicular to tangent passes though centre
24 2tan2450 +cos2300-sin2600 1
√3 √3
=2(1)2 +( 2 )2 -( 2 )2 =2
1
OR
Solution:
tan (A+B) =√3 =tan 60
A+B =60-------------(i) 1
tan (A-B) =1/√3 =tan 30
A-B =30 --------------(ii)
Adding equation I & ii
2A=90
A=45
45+B=60 1
B=15
25 Perimeterof (semicircular arc )+diameter=72cm
πr+2r=72cm
22
r[ 7 +2]=72
22+14
r[ ]=72 1
7
36
r=72
7
R=14cm
1 1
Area of protractor= πr2
2
1 22
× 7 ×14×14=308cm2
2
OR
Let the inner and outer radii of the circular track
be r and R respectively. 1
then inner circumference=2πr=220 m
22
2× 7 ×r=220
220×7
r= 2×22 =35m
since the track is 7m wide everywhere ,therefore
R=outer radius =r+7=35+7=42m
outer circumference=2πR
22
2× 7 ×42=264m
1
Rate of fencing = ₹2 per metre
Total cost of fencing=264×2=₹528

93
Section C
26 Let one zero of the zero be α.
Than other zero = 7α.
−(−8)
Sum of zero, α + 7α = 3 1
8 1
8α = 3, α = 3
2𝑘+1 1
Product of zero, α x 7α = 3
2𝑘+1
7 α2 = 3
21 2𝑘+1
9
= 3
,
2 1
k= .
3
27 Since – 5 is a root of the equation 2x² + px – 15 = 0
∴ 2(-5)² + p(-5) – 15 = 0 1
⇒ 50 – 5p – 15 = 0 or 5p = 35 or p = 7
Again p(x² + x) + k = 0 or 7x² + 7x + k = 0 has equal roots 1
∴D=0
i.e., b2 – 4ac = 0 or 49- 4 × 7k = 0
1
⇒ k = 49/28 = 7/4
OR
The given quadratic equation is (a² + b²)x² + 2(ac + bd)x +(c²+ d²) = 0
D = b² – 4ac 1
= 4(ac + bd)2 – 4(a² + b²) (c²+ d²)
= -4(a²d² + b²c²– 2abcd) = – 4(ad – bc)2
Since ad ≠ bc 1
Therefore D < 0
Hence, the equation has no real roots. 1
28 ˂A = ˂OPA =˂OSA = 90˚ 1
Hence, ˂SOP=90˚ Also, AP=AS 1
Hence, OSAP is a square
AP=AS=10cm CR=CQ=27cm 1
BQ=BC-CQ=38-27=11cm BP=BQ=11 cm X=AB=AP+BP=10+11=21 cm
29 Let 5 -√3 be a rational number
We can find co-prime a and b (b≠0) such that 5 -√3=a/b 1
5 -a/b=√3
So we get,(5b-a)/b=√3 1
Since a and b are integers, we get (5b-a)/b is irrational and so
√3 is rational. But √3 is an irrational number Which contradicts
our statement
Therefore 5 -√3 is irrational 1
30 LHS: (1-sinA+cosA)2 = [(1-sinA) + cosA]2
= (1-sinA)2 + cos2A + 2(1-sinA)cosA 1
= 1 + sin2A − 2sinA + cos2A + 2(1-sinA)cosA
= 1 + (sin2A + cos2A) − 2sinA + 2(1-sinA)cosA 1
= 1 + 1 − 2sinA + 2(1-sinA)cosA [Since, sin2A + cos2A =1]
= 2 − 2sinA + 2(1-sinA)cosA
= 2(1 − sinA) + 2(1-sinA)cosA
= 2(1 − sinA)(1 + cosA) 1
= RHS
31 There are 36 possible outcomes of rolling two dices n(S)=36
(i)have a sum 11 1
Favourable outcome are (5,6),(6,5)
94
Number of favourable outcome n(E1)=2
𝟐 𝟏
P(have sum 11)=𝟑𝟔=𝟏𝟖
1
(ii)have product 12
Favourable outcome are (2,6),(3,4),(4,3),(6,2)
Number of favourable outcome n(E2)=4
𝟒 𝟏
P(have a product less than 16)=𝟑𝟔=𝟗
(iii) is a doublet of prime numbers 1
Favourable outcome are (2,2),(3,3),(5,5)
3 1
No of favourable outcome n(E3)=36=12
Section D
32 Let the speed of stream = x km/hr
Then speed of boat upstream = (24-x)km/hr
Speed of boat downstream=(24+x)km/hr
According to question we have
32 32
- =1 1
24−𝑥 24+𝑥
1 1
32[24−𝑥-24+𝑥]=1 1
24+𝑥−24+𝑥
32[ 576−𝑥2 ]=1
32(24+x-24+4)=576-x2 1
64x=576-x2
x2+64x-576=0 1
x2 +72x-8x-576=0
x(x+72)-8(x+72)=0
(x-8)(x+72)=0
X=8, -72 1
Speed of stream = 8km/hr
Or
8 30
Two taps together fillthe cistern in 211minutes=11 minutes
Thus it will fill in 1 minutes=11/30 ciston
Let first tap fills the cistern in x minutes
And 2nd tap will take=(x+1) minutes
1 1 11
Thus + = 1
𝑥 𝑥+1 30
𝑥+1+𝑥 11
=30
𝑥(𝑥+1)
60x+30=11x2 +11x
11x2 -49x-30=0 1
11x2-55x+6x-30=0
11x(x-5)+6(x-5)=0 1
(x-5)(11x+6)=0
6
X=5, x=-11
x=-11
6 1
is not possible so 1st tap take 5 minute and 2nd tap takes 6 minute
1
33 In , ,
So, from the diagram, is similar to ,

95
1

1
Now, in , ,

So, from the diagram, is similar to ,

Now, on adding equation (1) and equation (2),

is the distance between two poles. 1

Hence proved.
34 From the question, we know that

=>The diameter = D = 4 m, radius r = 2 m

=>l = 2.8 m (slant height)

=>Also, we know the height of the cylinder (h) is 2.1 m


8√6
Height of cone H = √𝑙 2 − 𝑟 2 = m
10

=>So, the required surface area of the given tent = surface area of the cone (the 1
top) + surface area of the cylinder(the base)

= πrl + 2πrh
22
= πr (l+2h) = x2(2.8+2x2.1)= 44 m2
7

=>∴ The cost of the canvas at the rate of Rs. 500 per m2 for the tent will be 1

96
= Surface area × cost/ m2= 44 × 500

=Rs. 22000
1
Volume of air in tent =vol of cone + vol of cylinder
= r2H + r2h
= r2(h + H)
22 8 √6 1
= x 2x2( 10 + 2.1)
7
= 4.60 m3 (approx)

1
OR
Radius of cone,r=5cm
Height of cone,h=8cm
1
Volume of water in the cone=[3π(5)2×8] 1
200𝜋
= cm3
3
1 200𝜋
Volume of water flow out=[4 × ]
3 1
50𝜋
= 3 cm3
Radius of spherical shot=0.5cm
4
Volume of one spherical shot= π(0.5)3 1
3
𝜋
= 6 cm3 1
𝑣𝑜𝑙𝑢𝑚𝑒 𝑜𝑓 𝑤𝑎𝑡𝑒𝑟 𝑓𝑙𝑜𝑤𝑠
Number of lead shot dropped=𝑣𝑜𝑙𝑢𝑚𝑒 𝑜𝑓 𝑜𝑛𝑒 𝑠𝑝ℎ𝑒𝑟𝑖𝑐𝑎𝑙 𝑠ℎ𝑜𝑡

50𝜋
50×6
= 3
𝜋 = =100 1
3
6
35 Classes 35-40 has maximum frequency 50,therefore this is model class
Now l=35 , f1=50, f0=34, f2=42, h=5 CLASS INTERVAL REQUENCY
𝑓1−𝑓𝑜
Mode= l+ ( )×h 25-30 25
2𝑓1−𝑓0−𝑓2
50−34
=35 + (2×50−35−42)×5 30-35 34
16×5 35-40 50
=35 + ( ) 40-45 42 1
24
10
=35+ 3 =38.33 45-50 38
Mean 50-55 14
Let a = 37.5
C. I. f 𝑥𝑖 (𝑥𝑖 −𝑎) 𝑓𝑖 𝑢𝑖 1
𝑢𝑖 = ℎ 1
25-30 25 27.5 -2 -50
30-35 34 32.5 -1 -34
35-40 50 37.5 0 0
40-45 42 42.5 1 42
45-50 38 47.5 2 76
∑ 𝑓𝑖 𝑢𝑖
Mean = a + xh 1
∑ 𝑓𝑖
34
= 37.5 + 189 x 5 1
= 37.5 + 0.89 = 38.39

97
Section E
36 (i)His savings per year are 20000, 25000, 30000, 35000... up to 12 terms Partial credit 1+2
12th year saving amount= 20000+11x5000 = Rs 75000. +1
(ii) Gross Saving of DINKAR = 20000+25000+30000+…………………+75000.
S12= Rs 5,70,000
OR
S7 = Rs 245000
(III) Rs 70000
37 (i)√𝟒𝟏, √𝟏𝟎𝟎√𝟒𝟏 1+1
(ii)isosceles triangular ground +2
(iii)(0, 0)
OR
4 units
38 (i)As per information given in question we have made diagram below. Here, E is initial
position of eagle and R is initial position of rat. Eagle catch rat at S

𝑂𝐸
(i) =tan450 =1
𝑂𝑅
OE=OR=90m
𝐸𝑂
(ii) =cos60
𝐸𝑆
90 1
=
𝐸𝑆 2
ES=90×2=180m
𝑅𝑆 90(√3−1)
(iii)Speed of rat= 𝑡 = 10
=9(√3-1)m/s
OR
𝐸𝑆 180
Speed of Eagle= 𝑡 = 10 =18m/s=64.8km/h

98
KENDRIYA VIDYALAYA SANGHTHAN, JAIPUR REGION
Class - X Session 2022-23
Subject - Mathematics STANDARD (041)
SAMPLE PAPER 5

Time Allowed: 3 Hours Maximum Marks: 80

General Instructions: As per the previous question papers.


S Marks
Section A
No
Section A consists of 20 questions of 1 mark each.

.1 The largest number which divides 70 and 125 leaving remainder 5 and 8 1
respectively is
(a) 13 (b) 65 (c) 875 (d) 1750
2 A number when added to its reciprocal the result becomes double of the number. 1
The number is:
(a) 3 (b) 2 (c) 0 (d) -1
3 The zeroes of the polynomial x -3x- m(m+3) are
2 1
(a) m,m+3 b)- m,m+3 c) m,-(m+3) d) -m,-(m+3)
4 The value of k for which the system of equations x+y-4=0 and 2x+ky= 3 has no 1
solution, is
(a) -2 b) 3 c) 2 d) ≠ 2
5 Centre of a circle is at ( -1,3) and one end of the diameter has coordinates (2,5) then 1
the coordinates of the other end are
(a) (-4,1) (b) (4,-1) (c) (1,-4) (d) (3,2)
6 Consider the statements below. 1
(i) All circles are similar. (ii) All squares are similar. (iii) All right triangles are
congruent. (iv) All equilateral triangles are congruent.
Which statements are correct?
(a) (i) and (ii) (b) (ii) and (iv) (c) (i) and (iii) (d) (iii) and (iv)
7 In ∆ PQR, right-angled at Q, PR = 13 cm and PQ = 5 cm. Determine the value of sin P 1
13 12 13 5
(a) 12 (b) 13 (c) 5 d) 13
8 If x = a cosθand y = b sin θ, then the value of b2x2 + a2y2 is 1
(a) b2 + a2 (b) a2b2 (c) a2/b2 (d) ) b2 - a2
9 In the given below figure, the value of ∠P is 1

a) 40° b) 60° c) 80° d)100°


10 Which is equivalent to the sum of the lengths of arc corresponding to the minor and 1
major segment of a circle of radius 12cm?
a) 24𝜋 cm (b) 48 𝜋 cm (c) 12 𝜋 cm (d) 144 𝜋 cm
99
11 A solid is in the shape of a cone standing on a hemisphere with both their radii 1
being equal to 1 cm and the height of the cone is equal to its radius. The volume of
the solid will be?
1
a) 3 𝜋 cm3 (b) 4 𝜋 cm3 (c) 2 𝜋 cm3 (d) 𝜋 cm3
12 A chord of a circle of radius 10 cm subtends a right angle at its centre. What is the 1
length of the chord (in cm).
a) 12√2 b) 20√2 c) 100√2 d) 10√2
13 In ∆ABC,DE || BC, find the value of x. 1

a) 3 b) 2 c) 5 (d) 10
14 In a frequency distribution , the class width is 4 and the lower limit of first class is 1
10. If there are 6 classes, the upper limit of the last class is
(a) 22 (b) 26 (c) 30 (d) 34
15 The arithmetic mean of 12 observations is 7.5. If the arithmetic mean of 7 of these 1
observations is 6.5, the mean of the remaining observations is
5.5 (b) 8.5 (c) 8.9 (d) 9.2
16 The diameter of a roller is 84 cm and its length is 120 cm. It takes 500 complete 1
revolutions to move once over to level a playground. The area of the playground in
m2 is:
(a) 1584 (b) 1284 (c) 1384 (d) 1184
17 A box contains 90 discs which are numbered from 1 to 90. If one disc is drawn at 1
random from the box, find the probability that it bears a two-digit number?
2 1 3 9
(a) (b) (c) d)
9 9 9 10
18 (cosec θ– sin θ) (sec θ– cosθ) (tan θ+ cot θ) is equal to 1
(a) 1 (b) 2 (c) -1 (d) 0
19 Assertion(A) : For any two positive integers a and b, HCF(a,b) × LCM(a,b) = a × b 1
Reason (R) : The HCF of two numbers is 5 and their product is 150.
then their LCM is 30.
(a) Both A and R are true and R is the correct explaination for A.
(b) Both A and R true and R is not the correct explaination for A.
(c) A is true but R is false.
(d) A is false but R is true.
20 Assertion (A) : Pair of linear equations 9x + 3y + 12 = 0, 18x + 6y +24 = 0 have 1
infinitely many solutions.
Reason (R) : Pair of linear equations a1x + b1y + c1 =0 and a2x + b2y + c2 = 0 have
a 𝑏 c
infinitely many solutions, if 𝑎1 = 𝑏1 = 𝑐1
2 2 2

(a) Both A and R are true and R is the correct explaination for A.
(b) Both A and R are true and R is not the correct explaination for A.
(c) A is true but R is false.
(d) A is false but R is true.

100
Section B

Section B consists of 5 questions of 2 marks each.


21 A contractor plans to install two slides for the children to play in a park. For the 2
children below the age of 5 years, she prefers to have a slide whose top is at a height
of 1.5 m, and is inclined at an angle of 30° to the ground, whereas for elder children,
she wants to have a steep slide at a height of 3m, and inclined at an angle of 60° to
the ground. What should be the length of the slide in each case?
OR
In ∆ PQR, right-angled at Q, PR + QR = 25 cm and PQ = 5 cm. Determine the values
of sin P.
22 For what value of k, will the following pair of equations have infinitely many 2
solutions:
2x + 3y = 7 and (k+ 1)x+ (2k–1)y = 4k+ 1.
23 In given Fig. , if PQ || RS, prove that ΔPOQ ~ ΔSOR. 2

24 In the given figure, PA and PB are tangents to the circle with centre O. If ∠APB = 2
60°, then calculate ∠OAB.

25 A chord of a circle of radius 7cm subtends a right angle at the center . Find the area 2
of the corresponding minor segment.

OR
The area of a sector is one-twelfth that of the complete circle. Find the angle of the
sector.
Section C

Section C consists of 6 questions of 3 marks each.


26 Prove that√5 is irrational and hence show that 3+ √5 is also irrational. 3

27 If α and β are zeros of the quadratic polynomial x2 – (k + 6)x + 2(2k - 1). Find the 3
1
value of k if α+β=2αβ
101
28 The taxi charges in a city consist of a fixed charge together with the charge for the 3
distance covered. For a distance of 10 km, the charge paid is ₨ 105 and for a
journey of 15 km, the charge paid is ₨ 155. What are the fixed charges and the
charge per km? How much does a person have to pay for travelling a distance of 25
km?
OR
The sum of the digits of a two-digit number is 9. Also, nine times this number is
twice the number obtained by reversing the order of the digits. Find the number.
29 Prove the following 3
tan2 A+ cot2 A = sec2A cosec2A-2
30 Two different dice are thrown together. Find the probability that the numbers 3
obtained.
(i)Have a sum less than 7.
(ii)Have a product less than 16.
(iii) Is a doublet of odd numbers
31 Prove that: The tangent at any point of a circle is perpendicular to the radius 3
through the point of contact.

Using the theorem, Find the value of ∠PTQ from the


given figure.

OR
Prove that: The lengths of tangents from an external point to a circle are equal.
Using the above theorem, Find the perimeter of ∆APQ in fig. given below when AB =
5cm

Section D

Section D consists of 4 questions of 5 marks each.


32 Prove that If a line is drawn parallel to one side of 5
a triangle to intersect the
other two sides in distinct points, the other two
sides are divided in the same ratio.
Using above theorem find EC if DE || BC in fig. given
below:
33 Thepercentageof marks obtainedby100 students inan examination aregiven 5
below:
Marks 30- 35- 40- 45- 50- 55-60 60-65
60-
35 40 45 50 55 65
Frequen 14 16 18 23 18 8 3
3

102
cy
Determine the mean and median percentage of marks.
34 Sam’s house has an over head tank in the shape of a cylinder. This is filled by 5
pumping water from a sump (an underground tank) which is in the shape of a
cuboid. The sump has dimensions 1.57 m× 1.44 m×95cm. The over head tank has
a radius of 60 cm and a height of 95 cm. Find the height of the water left in the
sump after the over head tank has been completely filled with water from the
sump which had been full. Compare the capacity of the tank with that of the sump.
(Use𝜋=3.14)
OR
A cistern, internally measuring 150 cm x 120 cm x 110 cm has 129600 cm3 of
water in it. Porous bricks are placed in the water until the cistern is full to the
brim. Each brick absorbs one-seventeenth of its own volume of water. How
many bricks can be put in without overflowing the water, each brick being
22.5cm x 7.5 cm x 6.5 cm?
35 Two water taps together can fill a tank in 9 hours 36 minutes. The tap of larger 5
diameter takes 8 hours less than the smaller one to fill the tank separately. Find the
time in which each tap can separately fill the tank.
OR
An express train takes 1 hour less than a passenger train to travel 132 km between
Mysore and Bangalore (without taking into consideration the time they stop at
intermediate stations). If the average speed of the express train is 11km/h more
than that of the passenger train, find the average speed of the two trains.
Section E

Case study based questions are compulsory.


36 The top of a table is shown in the figure given below: 1+1
+
2

(i) Find the distance between points A and B.


(ii) Find the coordinates of the mid-point of line segment joining points M and Q
using formula.
(iii) Find the coordinates of vertices of rectangle RGFQ and hence find its area.
OR
Find the distance PQ and FE and find FE- PQ.
103
37 Deepa has to buy a scooty. 1+2
She can buy scooty either making cash down payment of Rs. 25,000 or by making +
15 monthly installments as below. 1
I month - Rs. 3425,
st

2nd month - Rs. 3225,


3rd month - Rs. 3025,
4th month - Rs. 2825 and so on.

(i) Find the amount of 10thinstallment. (Use Formula)


(ii) Total amount paid in 15 installments. (Use Formula)
OR
What is the difference between 8th and 10thinstallment?
(iii) Which one is better choice for Deepa: By cash or by making monthly
installment and why?
38 The houses of Ajay and Sooraj are at 100 m distance and the height of their houses 1+1
is the same as approx 150m. One big tower was situated near their house. Once +
both friends decided to measure the height of the tower. They measure the angle of 2
elevation of the top of the tower from the roof of their houses. The angle of
elevation of Ajay's house to the tower and Sooraj's house to the tower are 45o and
30o respectively as shown in the figure.

i) What is the distance between the tower and Ajay’s house..


ii) What is the distance between the tower and the house of Sooraj?
iii) Find the height of the tower taking √3 = 1.73
OR
If angle of elevation of Ajay’s house to the tower changes to 60o then find
distance between tower and Sooraj’s house?

104
Marking Scheme SAMPLE PAPER STANDARD 5

Section A
Q No. Marks
1 a)13 1
2 (d) -1 1
3 (b)- m,m+3 1
4 (c) 2 1
5 (a)(-4,1) 1
6 (a) (i) and (ii) 1
12
7 (b) 13 1
8 (b) a b 1
9 (a) 40° 1
10 (a) 24𝜋 cm 1
11 (d) 𝜋 cm3 1
12 d) 10√2 1
13 (a) 3 1
14 (d) 34 1
15 (c) 8.9 1
16 (a)1584 1
9
17 (d) 10 1
18 (a) 1 1
19 (b) 1
20 (a) 1
SECTION B
21 Finding length of the slide for below 5year = 3 m and 1
Length of the slide for elders children = 2√3 m 1
OR
Getting the values of hypotenuse PR = 13 and QR = 12 using Pythagoras theorem. 1
𝑂𝑝𝑝𝑜𝑠𝑖𝑡𝑒 𝑆𝑖𝑑𝑒 𝑄𝑅 12
sin p = = 𝑃𝑅 = 13 1
Hypotenuse
a1 𝑏 c
22 The system of equations has infinitely many solutions, if = 1= 1. ½
𝑎2 𝑏2 𝑐2
2 3 7
Therefore, 𝑘+1 = 2𝑘−1 = 4𝑘+1. 1
⇒4k-2 = 3k+3
⇒k= 5 1/2
23 ∠ POQ = ∠ SOR (Vertically opposite angles) ½
∠ P = ∠ S (Alternate angles) ½
ΔPOQ ~ ΔSOR. (AA similarity criterion) 1

105
24 Getting ∠ OAP= 90°(radius is perpendicular to tangent at point of contact) ½
In ΔAPB,∠ P=60°
Since PA and PB are tangents to the circle from P. Therefore, PA = PB
∴∠PAB=∠PBA
∠P+∠PAB+∠PBA=180∘ [angle sum property of triangle] ∠ BAP= 60°
60∘+2∠PAB=180∘

⇒∠PAB=60∘ 1
⇒∠PAB+∠OAP=90∘
⇒∠ OAP= 90°-60°= 30° 1/2
25 area of the corresponding minor segment= area of sector- area of triangle
1 22 1
=4 x 7 x7x7- 2x7x7 1
1
= 14 cm2
OR
θ
Area of a sector subtending an angle θ at the centre is given by πr2×360°.
½
θ 1
As per the question, πr2×
360°
.=12 ×πr2 1
⇒θ=30° ½
Section C
26 Let us assume, to the contrary, that √5 is rational. ½
So, we can find integers p and q (q ≠ 0), such that
√5 = p/q, where p and q are coprime. Squaring both sides, we get
5 = p2/q2 ⇒ 5q2 = p2 ⇒ 5 divides p2
⇒ 5 divides p
So, let p = 5r
Putting the value of p in (i), we get 1
5q = (5r) ⇒ 5q = 25r
2 2 2 2

⇒ q2 = 5r2
⇒ 5 divides q2
5 divides q
So, p and q have at least 5 as a common factor.
But this contradicts the fact that p and q have no common factor. So, our assumption
is wrong, √5 is irrational.
√5 isirrational,3 is a rational number.
So, we conclude that3+ √5isirrational. 1½
27 α,β are the zeroes of x2−(k+6)x+2(2k−1)
i.e, α+β=k+6 ( Sum of roots of quadratic ) 1
and αβ=2(2k−1) ( Product of roots ) 1
α+β=k+6 and αβ=4k–2
α+β=αβ/2
⇒k+6=2k–1
1
∴k=7

106
28 Let the fixed charge be x & the variable charge be y.
Given that charge paid for 10 km is Rs 105 and for 15 km is Rs 155
So,
Fixed charge +10×(Charge per km)=Rs105
x+10y=105⇒x=105−10y (1) ½
And for 15 kilometers,
x+15y=155⇒x=155−15y (2) ½
From (1) and (2)
105−10y=155−15y⇒y=10 1
putting value of y in equation (1)
x=105−10×10 ½
x=5
For travelling 25 km, he has to pay
=x+25y
(5 + 25 × 10) ½
= Rs.255.
OR
Let the unit digit and tens digits of the number be x and y
Number = 10y + x
Number after reversing the digits = 10x + y
According to the question, ½
⇒ x + y = 9 ... (i) ½
Also,
9(10y + x) = 2(10x + y) 1
⇒ 88y - 11x = 0
=>11(-x +8y)=0
⇒ -x + 8y =0 ... (ii)
Adding equation (i) and (ii), we get
⇒ 9y = 9
⇒ y = 1 ... (iii)
Putting the value in equation (i), we get
⇒x=8
Hence, the number is 10y + x = 10 × 1 + 8 = 18. 1
29 LHS=tan2A+cot2A
𝑠𝑖𝑛 2 𝐴 𝑐𝑜𝑠 2𝐴
=𝑐𝑜𝑠 2𝐴 + 𝑠𝑖𝑛2 𝐴
½
𝑠𝑖𝑛 4 𝐴+𝑐𝑜𝑠 4𝐴 (𝑠𝑖𝑛 2 𝐴+𝑐𝑜𝑠 2𝐴)2 −2𝑠𝑖𝑛 2 𝐴𝑐𝑜𝑠 2𝐴
= 𝑠𝑖𝑛2 𝐴𝑐𝑜𝑠 2𝐴 = 𝑠𝑖𝑛 2 𝐴𝑐𝑜𝑠 2𝐴
½
2 2
1 − 2𝑠𝑖𝑛 𝐴𝑐𝑜𝑠 𝐴 1 1
2 2
= . −2
𝑠𝑖𝑛 𝐴𝑐𝑜𝑠 𝐴 𝑐𝑜𝑠 𝐴 𝑠𝑖𝑛2 𝐴
2
1
=sec2A.cosec2A−2= RHS
Hence proved 1

107
30 Total number of outcomes = 36 1
i) Let A be the event of getting the numbers whose sum is less than 7. mark
The outcomes in favour of event A are For
(1, 1), (1,2), (1,3), (1,4), (1,5), (2,1), (2,2), (2,3), (2,4), (3,1), (3,2), (3,3), (4,1), (4,2) Each
and (5,1). part
Number of favourable outcomes = 15
𝑁𝑢𝑚𝑏𝑒𝑟 𝑜𝑓 𝑓𝑎𝑣𝑜𝑢𝑟𝑎𝑏𝑙𝑒 𝑜𝑢𝑡𝑐𝑜𝑚𝑒𝑠 15 5
∴ P(A ) = =36=12
Total number of outcome
(ii) Let B be the event of getting the numbers whose product is less than 16.
The outcomes in favour of event B are (1,1), (1,2), (1,3), (1,4), (1,5), (1,6), (2,1), (2,2),
(2,3), (2,4), (2,5), (2,6), (3,1), (3,2), (3,3), (3,4), (3,5), (4,1), (4,2), (4,3), (5,1), (5,2),
(5,3), (6,1) and (6,2).
Number of favourable outcomes = 25
25
∴P(B ) =36
(iii) Let C be the event of getting the numbers which are doublets of odd numbers.
The outcomes in favour of event C are (1,1), (3,3) and (5,5).
Number of favourable outcomes = 3
3
∴ P(C ) = 36
31 For Given , To prove , Construction and correct figure ½
For correct proof 1½
In the above figure, OPTQ is a quadrilateral and ∠P and ∠Q are 90°
The sum of the interior angles of a quadrilateral is 360°.
Therefore, in OPTQ,
∠Q + ∠P + ∠POQ + ∠PTQ = 360°
90° + 90° + 110° + ∠PTQ = 360°
290° + ∠PTQ = 360°
∠PTQ = 360° - 290°
∠PTQ = 70° 1
OR
For Given , To prove , Construction and correct figure ½
For correct proof 1½
we know that From a point we can draw two equal tangent on the circle
hence AB=AC=5 cm
And Also, PB=PX
and QC=QX........(1)
Again AB=AC=5
AP+PB=AQ+QC=5
AP+PX=AQ+QX=5 [From eqn (1)]
Perimeter of triangle △APQ
p=AP+PQ+AQ
p=(AP+Px)+(QX+AQ)
p=5+5
p=10 cm 1
Section D
32 For given, to prove , construction and correct figures. 1½
For correct proof. 2½
Given : DE∥BC in △ ABC,
Using Basic proportionality theorem,
∴AD/DB=AE/EC
⇒1.5/3=1/EC

108
⇒EC=3/1.5
EC=2 cm. 1
33 Let the assumed mean, A be 47.5
Marks 𝑥𝑖 𝑓𝑖 𝑑𝑖 = 𝑥𝑖 − 47.5 𝑑𝑖 𝑓𝑖 𝑢𝑖
𝑢𝑖=
5
30-35 32.5 14 -15 -3 -42
35-40 37.5 16 -10 -2 -32
40-45 42.5 18 -5 -1 -18
45-50 47.5 23 0 0 0
50-55 5 5 18 5 1 18
55-60 57.5 8 10 2 16
60-65 62.5 3 15 3 9
∑ 𝑓𝑖=110 ∑ 𝑓𝑖 𝑢𝑖 = −49
∑ 𝑓𝑖 𝑢𝑖
Mean=A+ h x( ∑ 𝑓𝑖
)
−49
= 47.5+5x 100
= 47.5- 2.45 3
=45.05
Marks 𝑓𝑖 𝑓𝑖
30-35 1 14
35-40 16 30
40-45 18 48
45-50 23 71
50-55 18 89
55-60 8 97
60-65 3 100
∑ 𝑓𝑖=100
𝑁
N==100 as = 50 so median class is 45-50
2
𝑛
−𝑐𝑓 50−48
Median=𝑙1 + h x( 2 𝑓 ) = 45+ x5
23 2
=45.43
34 The volume of water in the overhead tank equals the volume of the water removed
from the sump. ½
Now, the volume of water in the overhead tank (cylinder) = πr2h
= 3.14 × 0.6 × 0.6 × 0.95 m3 ½
The volume of water in the sump when full = l × b × h= 1.57 × 1.44 × 0.95 m3
The volume of water left in the sump after filling the tank
= [(1.57 × 1.44 × 0.95) - (3.14 × 0.6 × 0.6 × 0.95)] m3 2
= (2.147-1.074) m3
=1.073 m3
volume of water left in the sump
So, the height of the water left in the sump = l×b
1.073
=1.57×1.44 m
1
= 0.475 m = 47.5 cm
Also,
Capacity of tank 3.14×0.6×0.6×0.95 1
= 1.57×1.44×0.95 = 2
Capacity of sump
Therefore, the capacity of the tank is half the capacity of the sump. 1
109
OR
Volume of cistern =150×120×110=1980000 cm3
Volume to be filled in cistern =(1980000-129600) cm3 ½
=1850400 cm 3
Let x numbers of porous bricks were placed in the cistern.
Volume of n bricks = n×22.5×7.5×6.5
=1096.875n
1
As each brick absorbs one-seventeenth of its volume, therefore, volume absorbed by these
bricks
𝑛
=17(1096.875)
𝑛
⇒1850400=1096.875n-- (1096.875)
17
16𝑛
⇒1850400 = 17 (1096.875) 2
n=1792.41
Therefore, 1792 bricks were placed in the cistern. 1½
35 Let, tap A be of larger diameter.
If tap A takes x hrs to fill the tank,
⇒ Time taken by smaller tank to fill the tank =(x+8) hrs.
1
∴ Rate at which tap A fills the tank=𝑥
1
Rate at which tap B fills the tank=𝑥+8
36 48
two tap together, time taken = 9 hrs 36 min =9+ 60= 5 1
5
∴ work done per hr =
48
now
1
1 1 5
⇒𝑥 + 𝑥+8= 48
⇒48(2x+8)=5(x2+8x)
⇒96x+384=5x2+40x 1
⇒5x2−56x−384=0
∴x=16 , discarding negative value of x
2
∴ tap A can fill the tank in 16 hrs and
tap B can fill In (16+8)=24 hrs .
OR 1
Let x km per hour be the average speed of the passenger train, then the average speed of
the express train will be (x+11) km per hour. 1
132
The time taken by the passenger train is hour and the time taken by the express train
𝑥
132
is hour. So,
𝑥+11
132 132
𝑥+11
−1= 𝑥
x2+11x−1452=0 2
x2+44x−33x−1452=0
(x−33)(x+44)=0
x=33,x=−44 1
The speed cannot be negative, so the speed of the passenger train is 33 km per hour and
the speed of the express train is 33+11=44 km per hour.
36 (i) 4√2
(ii) (7,7)
1
(iii) R(5,3), G(5,1) F(9,1) Q(9,3) and area = 8 sq units
+1+2
OR
(iv) FE- PQ = 4√2 −2√2= 2√2
37 (i) Rs 1625 1+2+
(ii) Rs 30,375 OR Rs 400 1

110
(iii) By cash as cash payment is less than instalment payment
38 (i) Solving and getting 50(√3 + 1) m
1+1+
(ii) Solving and getting QB= 150+50 √3 m 2
(iii) 208.65 m OR 150 m

111
Class - X Session 2022-23
Subject - Mathematics (Basic - 241)
Sample Question Paper 1

Time Allowed: 3 Hours Maximum Marks: 80


General Instructions: As per the previous question papers.
SECTION A
SECTION A CONSIST OF 20 QUESTIONS OF 1 MARKS EACH
S.NO. MARKS
1. If HCF (a, b) = 12 and a x b = 1800 then LCM (a, b) = 1
(a) 3600 (b) 900 (c) 150 (d) 90
2. Which of the following are irrational whose sum and product both rational. 1
(a) √2+3, √2 -3 (b) √2 +√3 ,√2 − √3
(c) 3+ √2 , 3 − √2 (d) √2 +1 , √2 − 1
3. If one zero of the quadratic polynomial 𝑘𝑥 2 + 3𝑥 + 𝑘 𝑖𝑠 2, then the value of k is 1
5 5 6 6
(a) 6 (𝑏 ) − (𝑐 ) (𝑑 ) −
6 5 5
4. If 𝑎𝑥 2 + 𝑏𝑥 + 𝑐 = 0 ℎ𝑎𝑠 𝑒𝑞𝑢𝑎𝑙 𝑟𝑜𝑜𝑡𝑠, 𝑡ℎ𝑒𝑛 𝑐 = 1
𝑏 𝑏 𝑏2 𝑏2
(a) − 2𝑎 (𝑏 ) (𝑐 ) − (𝑑 )
2𝑎 4𝑎 4𝑎
5. The value of k for which the system of equation 𝑘𝑥 − 𝑦 = 2 𝑎𝑛𝑑 6𝑥 − 2𝑦 = 3 has 1
a unique solution is
(a) = 3 (b) ≠ 3 (c) ≠ 0 (d) = 0
6. The distance of the point (4,7) form x-axis is 1
(a) 4 (b) 7 (c) 11 (d) √65
7. If ABC and DEF are similar triangle such that ∠𝐴 = 47𝑜 𝑎𝑛𝑑 ∠𝐸 = 83𝑜 𝑡ℎ𝑒𝑛 ∠𝐶 = 1
(𝑎)50𝑜 (𝑏) 60𝑜 (𝑐 ) 70𝑜 (𝑑)80𝑜
1
8. If 𝐶𝑜𝑠𝜃 = 2 , 𝑡ℎ𝑒𝑛 𝐶𝑜𝑠𝜃 − 𝑆𝑒𝑐𝜃 𝑖𝑠 𝑒𝑞𝑢𝑎𝑙 𝑡𝑜 1
3 3 3 − √3
(a) 2 (𝑏 ) − (𝑐 ) √ (𝑑 )
2 2 2
1 1
9. 𝑜
If 0 ≤ 𝐴, 𝐵 ≤ 90 𝑠𝑢𝑐ℎ 𝑡ℎ𝑎𝑡 𝑆𝑖𝑛𝐴 = 2 𝑎𝑛𝑑 𝐶𝑜𝑠𝐵 = 2 𝑡ℎ𝑒𝑛 𝐴 + 𝐵 1
(a) 0𝑜 (𝑏) 60𝑜 (𝑐 ) 90𝑜 (𝑑 )30𝑜
3
10. If 𝐶𝑜𝑠𝐴 = 5 𝑡ℎ𝑒𝑛 𝑡ℎ𝑒 𝑣𝑎𝑙𝑢𝑒 𝑜𝑓 9 + 9𝐴 𝑖𝑠 1
(a) 9 (b) 16 (c) 25 (d) 34
𝐴𝑁
11. In this fig. AB = 8cm and AM = 2cm. if LM ∥ 𝐵𝐶 𝑎𝑛𝑑 𝐿𝑁 ∥ 𝐶𝐷 𝑡ℎ𝑒𝑛 = 1
𝑁𝐷

3 1
(a) 4 (𝑏 )
3
1 2
(b) (𝑐 ) 4 (𝑑 )
3

12. If ∆𝑃𝑄𝑅 ~∆𝑋𝑌𝑍 𝑎𝑛𝑑 𝑋𝑌 = 4𝑐𝑚, 𝑌𝑍 = 4.5𝑐𝑚, 𝑍𝑋 = 6.5𝑐𝑚 𝑎𝑛𝑑 𝑃𝑄 = 8𝑐𝑚, 1


𝑡ℎ𝑒𝑛 𝑝𝑒𝑟𝑖𝑚𝑒𝑡𝑒𝑟 𝑜𝑓 ∆𝑃𝑄𝑅 𝑖𝑠
(a) 25 cm (b) 23 cm (c) 15 cm (d) 30
cm
112
3
13. If E be an event such that 𝑃(𝐸 ) = 7 then P(not E) is equal to 1
4 6 5
(a) 7 (𝑏) 7 (𝑐 ) 7 (𝑑 ) 𝑛𝑜𝑛𝑒 𝑜𝑓 𝑡ℎ𝑒𝑠𝑒
14. The middle most observation of a statistical data has value which is called 1
(a) Mean of data (b) Median of data (c) Mode of data (d) None of these
15. The mean of distribution 1
𝑥 ∶ 10 12 20 25 35
𝑓 ∶ 3 10 15 7 5
Is equal to
(a) 25 (b) 24 (c) 20 (d) 18
16. The volume of cube is 1728 cm3 , then the length of its edge is equal to 1
(a) 12 cm (b) 14 cm (c) 16 cm (d) 24 cm
17. Find the area of sector, if radius is 24 cm and sector angle is 45 o 1
(a) 256𝜋 𝑐𝑚2 (𝑏) 144 𝜋 𝑐𝑚2 (𝑐 ) 288𝜋 𝑐𝑚2 (𝑑 ) 𝑁𝑜𝑛𝑒 𝑜𝑓 𝑡ℎ𝑒𝑠𝑒
18. If the perimeter and the area of circle are numerically equal, then the radius of 1
circle is
(a) 2 unit (b) 1 unit (c) 4 unit (d) 7 unit
Direction for questions 19 & 20: In question numbers 19 and 20, a statement of
Assertion (A) is followed by a statement of Reason (R). Choose the correct option.
19. Assertion: 3 × 5 × 7 + 3 × 5 is composite number. 1
Reason:sum of products ofprime numbers is always composite number.
(a) Both Assertion (A) and Reason (R) are true and Reason (R) is the correct
explanation of Assertion (A).
(b) Both Assertion (A) and Reason (R) are true but Reason (R) is not the correct
explanation of Assertion (A).
(c) Assertion (A) is true but Reason (R) is false.
(d) Assertion (A) is false but Reason (R) is true.
20. Assertion: 1
𝐴 𝑡𝑟𝑖𝑎𝑛𝑔𝑙𝑒 𝑤𝑖𝑡ℎ 𝑣𝑒𝑟𝑡𝑖𝑐𝑒𝑠 𝑎𝑡 (4,0), (−1, −1)𝑎𝑛𝑑 (3,5) 𝑖𝑠 𝐼𝑠𝑜𝑠𝑐𝑒𝑙𝑒𝑠 𝑅𝑖𝑔ℎ𝑡 𝑇𝑟𝑖𝑎𝑛𝑔𝑙𝑒.
Reason:If ABC is an Isosceles Triangle then it is right angled.
(a) Both Assertion (A) and Reason (R) are true and Reason (R) is the correct
explanation of Assertion (A).
(b) Both Assertion (A) and Reason (R) are true but Reason (R) is not the correct
explanation of Assertion (A).
(c) Assertion (A) is true but Reason (R) is false.
(d) Assertion (A) is false but Reason (R) is true.
Section B
Section B consists of 5 questions of 2 marks each.
21. Solve for X and Y: X+Y=14 and X-Y=4 2
22. ABCD is a trapezium in which AB∥ 𝐷𝐶 and its diagonals intersect each other at 2
the point o.
𝐴𝑂 𝐶𝑂
Show that 𝐵𝑂 =𝐷𝑂
23. The length of tangent from a point A at distance 5 cm from the centre of the circle 2
is 4 cm. Find the radius of circle.
3
24. If sin A = 4 then calculate cos A and tan A 2
OR
Evaluate sin600 cos300 + sin 300 cos 600
25. The radii of two circles are 19 cm and 9 c. m respectively. Find the radius of the 2

113
circle which has circumference equal to the sum of the circumference of the two
circle.
OR
The radii of two circles are 8 cm and 6 cm respectively. Find the radius of circle
having area equal to the sum of the area of the two circles.
Section C
Section C consists of 6 questions of 3 marks each.
26. Prove that√3 is an irrational number. 3
27. Find the zeroes of the quadratic polynomial 𝑋 2 +7X +10 and verify the 3
relationship between the zeroes and the coefficient.
28. Find the roots of the equation. 3
1 1 11
- 𝑋−7 = 30 X≠ -4 , 7
𝑋+4
OR
Find two numbers whose sum is 27 and product is 182.
29. Prove that the parallelogram circumscribing by a circle is a rhombus. 3
OR
A Quadrilateral ABCD is drawn to circumscribe a circle (see in figure ) Prove that
AB + CD = AD + BC

30. Prove that 3


𝑐𝑜𝑠𝐴 1+𝑠𝑖𝑛𝐴
+ =2 sec A
1+𝑠𝑖𝑛𝐴 cosA
31. A die is thrown once. find the probability of 3
(a ) Prime number (b ) A number lying between 2 and 6 (c ) An odd number
Section D
Section D consists of 4 questions of 5 marks each.
32. State and prove that the Basic proportional theorem. 5
Using this theorem ( in this figure ) If DE∥ BC and DF ∥ AE
then prove that
𝐵𝐹 𝐵𝐸
=𝐸𝐶
𝐹𝐸

33. Find the mean and mode of the data 5


CI Frequency
40- 50 10
50-60 2
60-70 28
70-80 12

114
80-90 10
90-100 15
Total 100
34. A train travels 360 km at a uniform speed. If the speed had been 5 km/hr more. 5
It would have taken 1 hour less for the same journey. Find the speed of train.
OR
The diagonal of a rectangular field is 60 Meters more than the shorter side. If the
larger side is 30 Meters more than the shorter side. Find the sides of the fields.
35. A Solid is in the shape of cone standing on a hemisphere with both radii being 5
equal to 1c.m and height of the cone is equal to the radius. Find the volume of
solid.
OR
A medicine capsule is in the shape of a cylinder with two hemispheres stuck to
each of its ends. The length of the entire capsule is 14 mm and the diameter of the
capsule is 5 mm. Find the surface area.
Section E
Case study based questions are compulsory.
36. India is competitive manufacturing location due to the low cost manpower and
strong technical and engineering capabilities contributing to higher quality
production runs. The production of TV set in a factory increases uniformly by a
fixed number every year. It produced 16000 sets in 6th years and 22,600 in 9th
year. Based on the above information. Answer the following:

1
1
(i) Find the production of TV sets during the first year.
(ii) Find the number of TV sets produced during the 8 th year.
(iii)In which year the production was 29,200 TV sets 2
OR
What is the difference of the production during the 7 th year and 4th year
37. The Class X students of a school in Rohini have been allotted a rectangular plot of
a land for their gardening activity. Saplings of Gulmohar are planted on the
boundary at distance of 1 m from each other. There is a triangular grassy loan in
the plot as shown in the following figure. The students are to sow seeds of the
flowering plants on the remaining area of the plot.

115
(i) Taking A as the origin AD and AB as the coordinate axes, find the co- 1
ordinates of P.
(ii) Taking C as the origin CB and CD as the coordinate axes, find the co-ordinate 1
of R.
(iii) Taking A as the origin, find the distance of PR 2
OR
Taking C as the origin, find the distance of PR
38. A group of students of class X visited India Gate on an educational trip. The
teacher as well as students had interest in the history behind India Gate. The
teacher narrated that India Gate’s official name is Delhi Memorial, Originally
called All India War Memorial, a monumental sand stone arch in New Delhi,
dedicated to the troops of British India who died in wars fought between 1914
and 1919. The teacher also said the India Gate which is located at the eastern end
of the Rajpath (formerly called the Kingsway), is about 138 feet (42 meters) in
height.

(i) What is the angle of elevation if they are standing at a distance of 42 m 2


away from the monuments?

(ii) If they want to see the top at an angle of 60o then the distance where they
should stand is?
(iii)If the altitude of the sun at 60o then the height of the vertical tower that
will cast a shadow of length 20 m is?
OR
If the ratio of the length of the rod and its shadow is 1 : 1, then the angle
of the elevation of the sun is?

116
Subject - Mathematics (Basic - 241)
MARKING SCHEME 1
Time Allowed: 3 Hours Maximum Marks: 80
SECTION – A Marks
1. (c) 150 1
2. (c) 3+√2 ,3 − √2 1
6
3. (d) − 5 1
4. 𝑏2 1
(d) 4𝑎
5. (b) k≠3 1
6. (b) 7 1
7. (a) 50° 1
−3
8. (b) 1
2
9. (c) 90° 1
10. (c) 25 1
1
11. (b) 3 1
12. (d) 30cm 1
4
13. (a) 7 1
14. (b) median of data 1
15. (c) 20 1
16. (a) 12 1
17. (d) none of these 1
18. (a) 2 units 1
19. (c ) 1
20 (b) 1
Section B
21. x+y=14 1
x-y=4
adding
2x=18
X=9
Y=5 1
22. In ∆𝐷𝐴𝐵 1
𝐹𝑂 ∥ 𝐴𝐵
𝐷𝐹 𝐷𝑂
=
𝐹𝐴 𝑂𝐵
Again in ∆𝐷𝐶𝐴
𝐹𝑂 ∥ 𝐷𝐶
𝐷𝐹 𝐶𝑂
=
𝐹𝐴 𝑂𝐴
𝐴𝑂 𝑂𝐶
So = 1
𝑂𝐵 𝑂𝐷
23. (52)=R2+42 1
R2=9
R=3cm 1

117
24. √7
SinA= 4 1
3
1
TanA=
√7
(OR)
√3 √3 1 1 1
× + ×
2 2 2 2
3 1 4
+ = =1 1
4 4 4
25. C1+C2=C 1
19+9=r=28 1
(OR)
A1+A2=A 1
82+62=r2
100=r2
r=10 1
Section C
26. Assume, √ 3is a rational number, it can be written as p/q, in
which p and q are co-prime integers and q≠0,
i.e. √3= p/q. where, p and q are coprime numbers, and q≠0.
On squaring both sides of the above equation; 1
3= (p/q)2
3 = p2/q2
3q2 = p2 ...(i)
p2 is a multiple of 3 so p is a multiple of 3 ...(ii)
Since, p is a multiple of two.
So p = 3m
p² = 9m² …(iii) 1
Using equation(i) into the equation (iii), we get;
3q² = 9m²
So q² = 3m²
q2 is a multiple of 3 so q is a multiple of 3 ...(iv)
Equation (ii) and (iv), implies that p and q have a common factor 3.
It contradicts the fact that they are co-primes which lead from our wrong
assumption that 3is a rational number. 1
Hence, 3 is an irrational number(proved)
27. X2+5x+2x+10 1
(x+5)(x+2)
X=-5,-2 1
Verify
𝛼 + 𝛽 = −5 − 2 = −7
𝛼. 𝛽 = (−5)(−2) = 10 1
1
28. X -3x+2=0
2
12
(x-2)(x-1) 1
X=1,2 12
OR 1
Let first no. be x
Second no. be 27 – x 1
Product = x (27-x) =182
2𝑥 2 + 3x -90 =0
1
118
(x-6) (2x+15)
So x =6
29. AP=AS
PB=BQ
CR=CQ
DR=DS 2
(AP+PB)+(CR+DR)=(AS+DS)+(BQ+CQ)
AB+CD=AD+BC 1
(OR)
Using property of ∥gm ABCD is a Rhombus
30. 𝑐𝑜𝑠 2 𝐴 + (1 + 𝑠𝑖𝑛𝐴)2 1
(1 + 𝑠𝑖𝑛𝐴)(𝑐𝑜𝑠𝐴)
2(1 + 𝑠𝑖𝑛𝐴) 2
= = 2𝑠𝑒𝑐𝐴 2
𝑐𝑜𝑠𝐴(1 + 𝑠𝑖𝑛𝐴) 𝑐𝑜𝑠𝐴
3 1
31. (i) P(A)=6 = 2 1
3 1
(ii) P(A)=6 = 2 1
3 1
(iii) P(A)=6 = 2 1
Section D
1
32. Given, to prove, figure 12
Correct proof 1
𝐵𝐹 𝐵𝐸 1
Correct proof 𝐹𝐸 = 𝐸𝐶 2
2
33. C.I f x Fx
40-50 10 45 450 2
50-60 25 55 1375
60-70 28 65 1820
70-80 12 75 900
80-90 10 85 850
90-100 15 95 425
Total 100 6820

∑𝑓𝑥 6820
Mean= = = 68.2 1
∑𝑓 100
Mode 1
2
C.I F
40-50 10
50-60 25
60-70 28
70-80 12
80-90 10
90-100 15
𝑓1−𝑓0
Mode=𝑙 + 2𝑓1−𝑓0−𝑓2 × ℎ
28 − 25
60 + × 10
2 × 28 − 25 − 12 1
1
2
=61.57

119
34. 360 360
− =1
𝑥 𝑥+5 3
𝑥 2 + 5𝑥 − 1800 = 0
(𝑥 − 40)(𝑥 + 45) = 0 2
So 𝑥 = 40 𝑠𝑝𝑒𝑒𝑑 𝑜𝑓 𝑡𝑟𝑎𝑖𝑛
(OR)
Shortest side=𝑥
Longer side=𝑥 + 30
Diagonal=𝑥 + 60
(𝑥 + 60)2 = 𝑥 2 + (𝑥 + 30)2 3
𝑥 2 − 60𝑥 − 2700 = 0
(𝑥 − 90)(𝑥 + 30) 2
So 𝑥 = 90
Longer side of rectangle= 120𝑐𝑚
Shorter side= 90𝑐𝑚
35. Volume of solid= volume of cone + volume of hemisphere 1
1 2
= 𝜋𝑟 2 ℎ + 𝜋𝑟 3
3 3 2
= 𝜋𝑐𝑚3 2
(OR)
5
𝑟 = 𝑚𝑚
2 1
Height of cylinder= 14 − 5 = 9𝑚𝑚
Surface area of capsule = Surface area of cylinder - 2×Surface area of
hemisphere 2
= 2𝜋𝑟ℎ + 4𝜋𝑟 2 1
= 220𝑚𝑚3
Section E
36. (i) 5000sets 1
(ii) 20400 1
(iii) 12 Year 2
(OR)
6600 2
37. (i) P (4,6) 1
(ii) R (10,3) 1
(iii) 𝑃𝑅 = √5𝑢𝑛𝑖𝑡 2
(OR)
𝑃𝑅 = √5𝑢𝑛𝑖𝑡 2
38. (i) 𝜃 = 45° 1
(ii) 14√3 1
(iii) 20√3 2
(OR)
Angle of elevation= 45° 2

120
KENDRIYA VIDYALAYA SANGHTHAN, JAIPUR REGION
Class - X Session 2022-23
Subject - Mathematics BASIC(241)
SAMPLE PAPER 2

Time Allowed: 3 Hours Maximum Marks: 80


General Instructions: As per the previous question papers.
Section A

Section A consists of 20 questions of 1 mark each.


S No. Marks

1 If LCM(x, 18) = 36 and HCF(x, 18) = 2, then x is 1


(a)2 (b)3 (c)4 (d)5
2 A quadratic polynomial, the sum of whose zeroes is 0 and one zero is 3, is 1
(a)x2 – 9 (b)x2+ 9
(c)x2 + 3 (d)x2 - 3
3 If a pair of linear equations in two variables is consistant, then the lines 1
represented by two equations are
(a)intersecting (b)parallel
(c)always coincident (d)intersecting and coincident
4 A quadratic equation can have 1
(a)at least two roots (b)at most two roots
(c)exactly two roots (d)any number of roots
5 Three bells ring at intervals of 4, 7 and 14 minutes. All the three rang at 6 1
AM. When will they ring together again?
(a)6:07 AM (b)6:14 AM
(c)6:28 AM (d)6:25 AM
6 The ratio in which (4, 5)divides the join of (2, 3)and (7, 8) is 1
(a)-2:3 (b)-3:2
(c) 3:2 (d)2:3
7 If∆ PQR ~ ∆XYZ and XY = 4 cm, YZ = 4.5 cm, ZX = 6.5 cm and PQ = 8 cm, 1
then perimeter of ∆PQR is
(a)25 cm (b)15 cm
(c)23 cm (d)30 cm
8 If x = a cos and y = bsin, then b2a2 + a2y2 is 1
(a) a2b2 (b)ab
(c) a4b4 (d)a2 + b2
9 The radius of a wheel is 0.25 m. The number of revolutions it will make to 1
travel a distance of 11 km will be
(a) 4000 (b)5500
(c)2800 (d)7000
10 If a solid sphere with total surface area 48 cm2 is bisected into two 1
hemispheres, then the total surface area of any one of the hemisphere is
(a)48 cm2 (b)60 cm2
(c)24 cm2 (d)36 cm2
11 If x tan450 cos600 = sin600 cot600, then x is equal to 1

121
(a) 1 (b)√3
1 1
(c) 2 (d)√2
12 For a frequency distribution, mean, median and mode are connected by 1
the relation
(a) Mode = 3Mean - 2Median (b) Mode = 2Median - 3Mean
(c) Mode = 3Median - 2Mean (d)Mode = 3Median + 2Mean
13 If the centroid of the triangle formed by the points (3, -5), (-7, 4), (10, -k) 1
is at the point (k, -1), then k is
(a) 3 (b)2
(c) 1 (d)4
14 If the angle between the radii of a circle is 1000, then the angle between 1
the tangents at the end of these two radii is
(a) 500 (b)600
(c) 80 0 (d)900
15 If 3sin2 - cos2 = 2, then  is equal to 1
(a) 00(b)600
(c) 450 (d)300
16 If the mode of the data: 16, 15, 17, 16, 15, x, 19, 17, 14 is 15, then x is 1
equal to
(a)15(b)16
(c)17 (d)19
17 Which of the following can not be the probability of an event 1
2
(a) 3 (b)1.5
(c) 15% (d)0.7
7
18 If the area of a sector of a circle is 20 of the area of the circle, then the 1
sector angle is equal to
(a) 1100 (b)900
(c) 130 0 (d)1260
Direction for question 19 & 20: In question numbers 19 and 20, a
statement of Assertion (A) is followed by a statement of reason (R).
Choose the correct option.
19 Assertion:HCF (234, 47) = 1. 1
Reason: HCF of two co-primes is always 1.

(a)Both Assertion (A)and Reason (R) are true and Reason (R) is the
correct explanation of Assertion (A). (b) Both
Assertion (A) and Reason (R) are true but Reason (R) is not the correct
explanation of Assertion (A).
(c) Assertion (A) is true but Reason (R) is false.
(d) Assertion (A) is false but Reason (R) is true.
20 Assertion: Three points A, B, C are such that AB + BC > AC, then they are 1
collinear.
Reason:Three points are collinear if they lie on a straight line.
(a)Both Assertion (A) and Reason (R) are true and Reason (R) is the
correct explanation of Assertion (A). (b) Both
Assertion (A) and Reason (R) are true but Reason (R) is not the correct
explanation of Assertion (A).
(c) Assertion (A) is true but Reason (R) is false.
122
(d) Assertion (A) is false but Reason (R) is true.

Section B

Section B consists of 5 questions of 2 marks each.


21 The circumference of a circle exceeds its diameter by 30 cm. Find the 2
circumference of circle?
OR
The radius of a semi-circular protactor is 14 cm. Find its perimeter?
22 If tan + cot = 2 find the value of tan2 + cot2. 2

23 Prove that the tangents drawn at the ends of a diameter of a circle are 2
parallel.
24 For what value of k, will the following system of equation have infinite 2
many solution
2x + 3y = 4 (k+2)x + 6y + 3k + 2
25 ABCD is a trapezium in which AB II DC. The diagonals AC and DB intersect 2
𝑂𝐴 𝑂𝐵
at O. Prove that𝑂𝐶 = 𝑂𝐷
OR
In the given figure, AB II PQ and AC II PR. Prove that BC II QR.

Section C
Section C consists of 6 questions of 3 marks each.
26 Given that √2 is an irrational, prove that 5 + 3√2 is an irrational. 3

27 Find the zeroes of the quadratic polynomial 3𝑥 2 + 5𝑥 − 2 and verify the 3


relationship between the zeroes and the coefficient.
28 If twice the son’s age in years is added to the father’s age, the sum is 70. 3
But if twice the father’s age is added to the son’s age, the sum is 95. Find
the ages of father and son.
OR
5 books and 7 pens together cost Rs 79 whereas 7 books and 5 pens
together cost Rs. 77. Find the total cost of 1 book and 2 pens.
29 Prove that the lengths of tangents drawn from an external point to a 3
circle are equal.
30 Prove that: 3
𝑠𝑒𝑐 𝜃 − 1 𝑠𝑒𝑐𝜃 + 1
√ + √ = 2 𝑐𝑜𝑠𝑒𝑐 𝜃
𝑠𝑒𝑐𝜃 + 1 𝑠𝑒𝑐𝜃 − 1
OR
tan 𝜃 + sec 𝜃 − 1 1 + sin 𝜃
=
𝑡𝑎𝑛𝜃 − sec 𝜃 + 1 cos 𝜃
31 In a single throw of two dice, find the probability of getting 3

123
(a) A doublet of prime numbers
(b) 8 as the sum
(c) An odd number on the first dice and 6 on the second dice
Section D
Section D consists of 4 questions of 5 marks each.
32 The speed of a boat in still water is 8km/hr. it can go 15km upstream and 5
22km downstream in 5 hours, find the speed of the stream.
OR
Rs. 250 are divided equally among a certain number of children. If there
were 25 children more, each would have 50 paise less. Find the number
of children.
33 Prove that if a line is drawn parallel to one side of a 5
triangle the other two sides divided in the same
ratio.
Using the above result prove the following:
𝐴𝐸 4
In given figure, DE ll BC. If = and AB = 20.4 cm,
𝐸𝐶 13
find AD.
34 A solid consisting of a right circular cone of height 120 cm and radius 60 5
cm standing ona hemisphere of radius 60 cm is placed upright in a right
circular cylinder full of watersuch that it touches the bottom. Find the
volume of water left in the cylinder, if the radiusof the cylinder is 60 cm
and its height is 180 cm.
OR
From a solid cylinder whose height is 2.4 cm and diameter 1.4 cm, a
conical cavity of the same height and same diameter is hollowed out. Find
the total surface area of the remaining solid to the nearest cm2.
35 Find the mean of the following data using step deviation method. 5

C. I. 40-50 50-60 60-70 70-80 80-90 90-100


F 10 25 28 12 10 15
Section E
Case study based questions are compulsory.
36 A mathematics teacher of a school has taken his children to a science
centre to visit the science and maths exhibition. When the students were
visiting different stalls, one student Rani observed that some rectangular
bars are arranged in ascending order as shown below. In the meantime,
her teacher reached at the stall and asked questions to Rani and the other
students to verify whether the pattern is in AP or not? Just by observing
the pattern shown below Rani and her friends answered.

Based on above information answer the following questions:


Q1 What is the height of 6th bar?
124
Q 2 What will be the expression for the nth term of the above sequence? 1
Q3 If there are 30 bars, then what is the height of 7 th bar from the end? 1
OR
Which bar will have the height 34cm from left? 2
37 To conduct Sports Day activities, inyour
rectangular shapedschoolground ABCD,
lines have beendrawn with chalk
powderatadistance of 1m each. 100 flower
potshave been placed at adistance of
1mfrom each other along AD, as shown
in. Niharika runs1/4th thedistance AD on
the 2nd line andposts a green flag. Preet
runs1/5 ththe distance AD on
theeighthlineand posts a red flag.
Based on the above information answer the
following questions
(i) What is the coordinate of the
green flag?
(ii) What is the coordinate of the red 1
flag? 1
(iii) What is the distance between the two flags? 2
OR
If Rashmi has to post a blue flag exactly half way between the line
segment joining the two flags, where she should post her flag?
38
The angle of elevation of the top of a tower from the top of a 7m tall
building is 60o and the angle of depression of the foot C of the tower from
the same point is found to be 450.

Based on the above information answer the following questions


(i) What is the distance between the tower and the building?
(ii) How much is the distance of AC?
(iii) What is the height of the tower?
1
OR
If the angle of the depression of the foot of the tower from the
1
top of the building is 60o, then what is the height of the tower?
2

125
Marking Scheme
Sample Question Paper – 2 (Basic)
Section A
1 (c)4 1
2 (a)X2 – 9 1
3 (d)intersecting and coincident 1
4 (c)exactly two roots 1
5 (c)6:28 AM 1
6 (d)2:3 1
7 (d)30 cm 1
8 (a)a2b2 1
9 (d)7000 1
10 (d)36 cm2 1
11 (a)1 1
12 (c)Mode = 3Median – 2Mean 1
13 (b)2 1
14 (c)800 1
15 (b)600 1
16 (a)15 1
17 (b)1.5 1
18 (d)1260 1
19 (a) Both Assertion (A) and Reason (R) are true and Reason (R) is the correct 1
explanation of Assertion (A).
20 (d) Assertion (A) is false but Reason (R) is true. 1
Section B
21 C = 30 + D
2πr = 30 + 2r ½
22
2x 7 x r = 30 + 2r
r= 7
22 ½
circumference = 2πr = 2x 7 x7 = 44 cm 1
OR
Radius = 14 cm
22
Perimeter of protactor = πr + D = 7 x14 + 2x14 1½
= 62 cm ½
22 tan + cot = 2
squaring both the sides
(tan + cot)2 = 4
Tan2𝜃 + 2tan𝜃cot𝜃 + cot2𝜃 = 4 1
1
Tan2𝜃 + 2tan𝜃x 𝑡𝑎𝑛𝜃 + cot2𝜃 = 4
Tan2𝜃 + 2 + cot2𝜃 = 4
1
Tan2𝜃 + cot2𝜃 = 4 -2 =2
23 Let's draw the tangents PQ and RS to the circle at the ends of the diameter AB.

126
½

According to Theorem 10.1: The tangent at any point of a circle is


perpendicular to the radius through the point of contact.

We know that radius is perpendicular to the tangent at the point of contact.

Thus, OA ⊥ PQ and OB ⊥ RS ½
Since the tangents are perpendicular to the radius,

∠PAO = 90°, ∠RBO = 90°

and ∠OAQ = 90°, ∠OBS = 90°

Here ∠OAQ is equal to ∠OBR and ∠PAO is equal to ∠OBS, which are two pairs
of alternate interior angles.

If the alternate interior angles are equal, then lines PQ and RS should be
parallel.
1
We know that PQ and RS are the tangents drawn to the circle at the ends of
the diameter AB.

Hence, it is proved that tangents drawn at the ends of a diameter of a circle


are parallel.
24 If system of equation have infinitely many solution, then
𝑎1 𝑏 𝑐 ½
= 𝑏1 = 𝑐1
𝑎2 2 2

2 3 4
= = ½
𝑘+2 6 3𝑘+2

2 3
=6
𝑘+2
4=k+2
1
K=2
25 Let's draw a trapezium ABCD with AB || DC.

127
1

1
If two angles of one triangle are respectively equal to two angles of
another triangle, then the two triangles are similar.
This is referred to as AA similarity criterion for two triangles.
In ΔAOB and ΔCOD
∠AOB = ∠COD (vertically opposite angles)
∠BAO = ∠DCO (alternate interior angles) ½
⇒ ΔAOB ∼ ΔCOD (AA criterion)
Hence, OA/OC = OB/OD
OR ½
given AB ll PQ , by BPT
𝑂𝐵 𝑂𝐴
= 𝐴𝑃 ………(i)
𝐵𝑄
AC ll PR , by BPT 1
𝑂𝐶 𝑂𝐴
= 𝐴𝑃 ………..(ii)
𝐶𝑅
From equation i and ii
𝑂𝐵 𝑂𝐶
= 𝐶𝑅
𝐵𝑄
By converse of BPT , BC ll QR
Section C
26 Let us assume that 5 + 3√2 is a rational number with p and q as co-
prime integers and q ≠ 0 1
𝑝
⇒ 5 + 3 √2 = 𝑞
𝑝 1
⇒ 3 √2 = 𝑞 - 5
𝑝−5𝑞
⇒ √2 = 3𝑞
𝑝−5𝑞
⇒ is a rational number
3𝑞
However, we know that √2 is an irrational number 1
This leads to the contradiction that 5 + 3√2 is an irrational number
27 The given polynomial is

Equate the given polynomial equal top zero to find the zeroes.

Splitting the middle term, we get


1

Using zero product property, we get

1
Therefore, the zeroes of the polynomial are -2 and 1/3.
If α and β are two zeroes of a polynomial , then
128
For the given polynomial α and β are -2 and 1/3 respectively.
1

28 let the age of the father be x and the age of the son be y
according to the conditions,
2y+x=70 ..............(1)
2x+y=95 ..............(2)
multiplying equation (1) into 2 we get 1
subtracting equation (2) from equation (3)
4y+2x=140 .........(3)
y+2x= 95
---------------
3y = 45 1
y=15 ..................(4)
substituting (4) in (2) we get
15 +2x=95
2x=80
x=40. 1
Therefore the age of the father is 40yrs and that of the son is 15 yrs
OR
Let the cost of 1 book be x.
And the cost of 1 pen be y.
According to the Question,
⇒ 5x + 7y = 79 ..... (i) 1
⇒ 7x + 5y = 77 ......(ii)
Multiplying both Eq (i) with 7 and Eq (ii) with 5, we get
35x + 49y = 553 ...(iii)
35x + 25y = 385 ...(iv) 1
Substracting both the Eq (iii) and (iv), we get
⇒ 24y = 168
⇒ y = 168/24
⇒y=7
Putting y's value in equation (i), we get
⇒x=6
Cost of book = x = 6
Cost of pen = y = 7
Total cost = 6 + 14 = Rs.20
Hence, the total cost of 1 book and 2 pens is Rs.20. 1
29 Fig, given, to prove 1½
Correct proof 1½
30
𝑠𝑒𝑐 𝜃 − 1 𝑠𝑒𝑐𝜃 + 1
√ + √
𝑠𝑒𝑐𝜃 + 1 𝑠𝑒𝑐𝜃 − 1

𝑠𝑒𝑐𝜃 − 1 + 𝑠𝑒𝑐𝜃 + 1 1
√𝑠𝑒𝑐 2 𝜃 −1

129
2𝑠𝑒𝑐𝜃 1
√𝑡𝑎𝑛2 𝜃
2𝑠𝑒𝑐𝜃
= 2cosec𝜃
𝑡𝑎𝑛𝜃 1
OR
tan 𝜃 + sec 𝜃 − 1
𝑡𝑎𝑛𝜃 − sec 𝜃 + 1

(tan 𝜃 + sec 𝜃) − (𝑠𝑒𝑐 2 𝜃 − 𝑡𝑎𝑛2 𝜃)


𝑡𝑎𝑛𝜃 − sec 𝜃 + 1 1
(tan 𝜃 + sec 𝜃) − (𝑠𝑒𝑐𝜃 + 𝑡𝑎𝑛𝜃)(𝑠𝑒𝑐𝜃 − 𝑡𝑎𝑛𝜃)
𝑡𝑎𝑛𝜃 − sec 𝜃 + 1

(tan 𝜃 + sec 𝜃)(1 − 𝑠𝑒𝑐𝜃 + 𝑡𝑎𝑛𝜃)


1
𝑡𝑎𝑛𝜃 − sec 𝜃 + 1

Tan𝜃 + 𝑠𝑒𝑐𝜃

𝑠𝑖𝑛𝜃 1
+ 𝑐𝑜𝑠𝜃
𝑐𝑜𝑠𝜃

1 + 𝑠𝑖𝑛𝜃 1
𝑐𝑜𝑠𝜃
31 Total outcome = 36
(a)Favourable outcome = (2,2),(3,3),(5,5)
3 1
Probability of getting doublet of prime numbers = 36 = 12 1
(b)Favourable outcome = (2,6),(6,2),(3,5),(5,3),(4,4)

5 1
Probability of getting 8 as the sum = 36

(c)Favourable outcome = (1,6),(3,6),(5,6)

Probability of getting an odd number on the first dice and 6 on the second
3 1
dice = 36 = 12 1
Section D
32 speed of boat in still water is 8km/hr.
So, speed of downstream = (8 + X) km/hr
And, speed of upstream = (8 - x)km/hr ½
Using, speed = distance/time
Time taken by the boat to go 15 km upstream = 15/(8 - x)hr
And, time taken by the boat 22 km downstream = 22/(8 + x)hr ½
From the question. the boat returns to the same point in 5 hr.
So,
15/(8-x) + 22/(8+x) = 5
[15/(8+x) + 22(8-x)]/(8-x)(8+x) = 5
(120 + 15x +176 +22x) /64 - x² = 5
130
(296 - 7x) / (64 - x²) = 5
5x² - 7x + 296 - 320 = 0
5x² - 7x - 24 = 0
5x² - 15x + 8x - 24 = 0 1
5x(x - 3) + 8(x - 3) = 0
(x - 3)(5x + 8) = 0
x = 3 or x = - 8/5
As the speed of the stream can never be negative, only the positive solution is
considered. 1
Therefore, the speed of the stream is 3 km/hr.
OR
let the no. of students be x
no. of money each one will get =250/x
new no. of students= x+25 no. of money each one will get=250/x+25 1
by condition
250/x -250/x+25=0.5
250(x+25)-250x/x²+25x=5/10
250x+6250-250x/x²+25x=5/10 1
6250×10=5×x²+25x
12500=x²+25x
x²+25x-12500=0
(x+125)(x-100)=0
x=-125 or x=100
As no. of students can not be negative
discarding x = -125 1
therefore no. of students are 100
33 Fig, given, to prove, construction 2
Correct proof 2
Using above theorem
In ∆𝐴𝐵𝐶, by BPT
𝐴𝐷 𝐴𝐸
= 𝐴𝐶
𝐴𝐵
½
𝐴𝐷 4
= 17
20.4
½
AD = 4.8 cm
34 Radius of the circular cone ,hemisphere and cylinder, r = 60 cm
Height of the circular cone , h = 120 cm
Height of the cylinder , H = 180 cm 1
Now,
Volume of water left in the cylinder = Volume of the cylinder – (volume of the
circular cone + volume of the hemisphere )
V = πr²H - (1/3πr²h + ⅔ × πr³) 1
V = πr²( H - ⅓ h - ⅔ r) 1
V = 22/7 × 60² (180 - ⅓ × 120 - ⅔ × 60)
V = 22/7 × 3600 (180 - 40 - 40) 1
V = 22/7 × 3600 × (180 - 80)
V = 22/7 × 3600 × 100
V = 7,920,000/7
V = 1,131,428.57 cm³
131
V = 1,131,428.57/ 1000000 m³
[1 cm³ = 1/1000000 m³] 1
V = 1.131 m³
OR
The outer surface area of the cylinder = 2πrh
= 2 x 3.14 x 0.7 x 2.4
= 10.55 cm² 1
slant height of the cone, L = √(h² + r²)
=√(2.4² + 0.7²)
= √6.25
= 2.5 cm 1
hence outer surface area of the cone = πrL
= 3.14 x 0.7 x 2.5
= 5.5 cm²
This outer surface area of the cone is equal to the inner surface area of the 1
hollow portion of the cylinder left.
surface area of the cylindrical base = πr²
= 3.14 x 0.7²
= 1.54 cm²
Hence total surface area of the remaining solid = The outer surface area of 1
the cylinder + inner surface area of the hollow portion of the cylinder left +
surface area of the cylindrical base
= 10.55 + 5.5 + 1.54
= 17.59 cm² 1
= 18 cm² (rounded off to nearest cm²)
35 Let a = 65
C. I. f 𝑥𝑖 (𝑥 −𝑎) 𝑓𝑖 𝑢𝑖
𝑢𝑖 = 𝑖ℎ
3
40-50 10 45 -2 -20
50-60 25 55 -1 - 25
60-70 28 65 0 0
70-80 12 75 1 12
80-90 10 85 2 20
90-100 15 95 3 45
∑ 𝑓𝑖 𝑢 𝑖
Mean = a + ∑ 𝑓𝑖
xh

32
= 65 + 100 x 10 1

= 65 + 3.2 = 68.2 1
Section E
36 (i)a = 7, d = 10-7 = 3 1+1+2
𝑎6 = a + 5d
= 7 + 5 x 3 = 22
(ii) 𝑎𝑛 = a + (n - 1)d
= 7 + (n - 1)3 = 4 + 3n
(iii) height of 7th bar from end
N = 30 – 7 + 1 = 24
𝑎24 = a + 23d = 7 + 23 x 3 = 76
OR
132
𝑎30 = a +29d = 7 + 29 x 3 = 94
94, 91, 88, …………… 13, 10, 7
𝑎𝑛 = a + (n - 1)d
34 = 94 + (n-1)(-3)
N = 21
37 (i)co-ordinate of green flag = (2, 1/4x100) = (2, 25) 1+1+2
(ii) co-ordinate of red flag = (8, 1/5x100) = (8, 20)
(iii)distance between two flag = √(2 − 8)2 + (25 − 20)2
= √(−6)2 + (5)2 = √61
OR
2+8 25+20
Co-ordinate of flag posted by Rashmi = ( 2 , 2 ) = (5, 22.5)
𝐴𝐵
38 (i)In ∆ ABC, tan450 = 𝐵𝐶 1+1+2
7
1 = 𝐵𝐶 , BC =7m
𝐴𝐵 1 7
(ii) In ∆ ABC, sin450 = 𝐴𝐶 , = 𝐴𝐶 , AC = 7√2 m
√ 2
(iii) height of tower = ED + DC
𝐸𝐷 𝐸𝐷
In ∆ ADE, , tan600 = 𝐷𝐶 , √3 = 7 , ED = 7√3 m
Height of tower = 7√3+ 7 = 7(√3 + 1)m
OR
𝐴𝐵 𝐷𝐶
In ∆ ABC, tan600 = 𝐵𝐶 , √3 = 7 , DC =7√3 m

Height of tower = ED + DC = = 7√3+ 7 = 7(√3 + 1)m

133
Class - X Session 2022-23
Subject - Mathematics (Basic)
Sample Question Paper 3
Time Allowed: 3 Hours Maximum Marks: 80
General Instructions: As per the previous question papers.
S No Section A MARKS
1 If n = 23 x 34 x 54 x 7, then the number of consecutive zeroes in n, where 1
n is a natural number, is
(a) 2 (b) 3 (c) 4 (d) 7
2 If α and β are the zeroes of the polynomial x2 + x + 1, then 1/α + 1/β = 1
(a) 1 (b) -1 (c) 0 (d) none of these
3 For what value of k, the equations 3x – y + 8 = 0 and 6x – ky = -16 1
represents coincident lines?
(a) 12 (b) –12 (c) 2 (d) -2
4 The equation 2x² + kx + 3 = 0 has two equal roots, then the value of k 1
is
(a) ±√6 (b) ± 4 (c) ±3√2 (d) ±2√6
5 If the distance between the points (x, -1) and (3, 2) is 5 units, then the 1
value of x is
(a) -7 or -1 (b) -7 or 1 (c) 7 or 1 (d) 7 or -1
6 In triangle ABC, DE || AB. If CD = 3 cm, EC = 4 cm, BE = 6 cm, then DA 1
is equal to
(a) 7.5 cm (b) 3 cm (c) 4.5 cm (d) 6 cm
7 4sinθ−cosθ 1
If 4 tan θ = 3, then 4sinθ+cosθis equal to
2 1 1 3
(a) (b) (c) (d)
3 3 2 4
8 The perimeter of the sector with radius 10.5 cm and sector angle 60° is 1
(a) 32 cm (b) 23 cm (c) 41 cm (d) 11 cm
9 The volumes of two spheres are in the ratio 27: 8. The ratio of their 1
curved surface is:
(a) 9 : 4 (b) 4 : 9 (c) 3 : 2 (d) 2 : 3
10 Find the class marks of classes 10–20 and 35–55. 1
(a) 10, 35 (b) 20, 55 (c) 15, 45 (d) 17.5, 45
11 The probability that it will rain tomorrow is 0.85. What is the 1
probability that it will not rain tomorrow
(a) 0.25 (b) 0.145 (c) 3/20 (d) none of these
12 There is a circular path around a sports field. Priya takes 18 minutes to 1
drive one round of the field. Harish takes 12 minutes. Suppose they
both start at the same point and at the same time and go in the same
direction. After how many minutes will they meet ?
(a) 36 minutes (b) 18 minutes (c) 6 minutes (d) They will
not meet
13 Assertion : 12n ends with the digit zero, where n is natural number. 1
Reason : Any number ends with digit zero, if its prime factor is of the
form 2m x 5 n , where m, n are natural numbers.
(a)Both assertion (A) and reason (R) are true and reason (R) is the
correct explanation of assertion (A).
134
(b)Both assertion (A) and reason (R) are true but reason (R) is not the
correct explanation of assertion (A).
(c)Assertion (A) is true but reason (R) is false.
(d)Assertion (A) is false but reason (R) is true.
14 Assertion: C is the mid-point of PQ, if P is (4, x), C is (y,- 1) and Q is (- 2, 1
4), then x and y respectively are -6 and 1.
Reason: The mid-point of the line segment joining the points P(x1 , y1 )
𝑥 +𝑥 𝑦 +𝑦
and Q(x2 , y2) is 1 2 2 , 1 2 2
(a)Both assertion (A) and reason (R) are true and reason (R) is the
correct explanation of assertion (A).
(b)Both assertion (A) and reason (R) are true but reason (R) is not the
correct explanation of assertion (A).
(c)Assertion (A) is true but reason (R) is false.
(d)Assertion (A) is false but reason (R) is true.
15 In the adjoining figure, ABCD is a trapezium in 1
which CD ∥ AB and its diagonals intersect at O. If
AO = (2x + 1) cm, OC = (5x – 7) cm, DO = (7x −
5) cm and OB = (7x + 1) cm, find the value of x.
(a) 2 cm (b) 3 cm (c) 4 cm
(d) none of the above
16 ∆ABC is such that AB = 3 cm, BC = 2 cm and CA = 2.5 cm. If ∆DEF 1
∼∆ABC and FE = 4 cm, then find the perimeter of ∆DEF.
(a) 12 cm (b) 13 cm (c) 14 cm (d) 15 cm
17 The value of 2(sin245°+ cot230°) – 6(cos245°— tan230°) is 1
a) 6 (b) 3 (c) 2 (d) 4
4
18 If cosA =5 then the value of tanA is 1
3 3 4 5
(a) (b) (c) (d)
5 4 5 4
19 The area of a circle is 220 cm2. The area of square inscribed in it is 1
(a) 49 cm2 (b) 70 cm2 (c) 140 cm (d) 150 cm2
20 For the following distribution 1
Marks 0-10 10-20 20-30 30-40 40-50
No of students 3 9 13 10 5
the number of students who got marks less than 30 is
(a) 13 (b) 25 (c) 10 (d) 12
SECTION B
21 For what value of k, will the system of equations x + 2y = 5 and 3x + 2
ky -15 = 0 have a unique solution
22 Prove that the line joining the mid-points of any two sides of a triangle 2
is parallel to the third side.
23 Quadrilateral ABCD is drawn to circumscribe a circle. Prove that AB + 2
CD = AD + BC.

1
24 If tan (A + B) = √3and tan (A - B) = 30<A+B ≤90, A > B, find A and B. 2

135
OR
3
If sin A = 5calculate cos A
25 The length of the minute hand of a clock is 14 cm. Find the area swept 2
by the minute hand in 5 minutes
OR
A brooch is made with silver wire in the form of a
circle with diameter 35 mm. The wire is also used in
making 5 diameters which divide the circle into 10
equal sectors as shown in Fig. Find the total length of
the silver wire required.
SECTION C
26 Prove that √5 is an irrational. 3
27 Find the zeroes of the quadratic polynomial 4u2+8u and verify the 3
relationship between the zeroes and the coefficients of the polynomial.
28 The sum of two natural numbers is 8 and their product is 15. Find the 3
numbers.
OR
Solve for x
1 1 1 1
= + + a + b + x ≠ 0, a, b, x ≠ 0
𝑎+𝑏+𝑥 𝑎 𝑏 𝑥

29 Prove that the lengths of tangents drawn from an external point to a 3


circle are equal.
OR
Prove that opposite sides of a quadrilateral circumscribing a circle
subtend supplementary angles at the centre of the circle.
30 Prove that sec A (1 – sin A)(sec A + tan A) = 1 3
31 From a pack of 52 playing cards, jacks, queens and kings of red colour 3
are removed. From the remaining, a card is drawn at random. Find the
probability that drawn card is :
(i) a black king (ii) a card of red colour (iii) a card of black colour
SECION D
32 The difference of the squares of two natural numbers is 45. The square 5
of the smaller number is 4 times the larger number. Find the numbers.
OR
If𝛼and 𝛽are the zeroes of the quadratic polynomial f(x) =4x2–5x–1,
then find the value of 𝛼2 +𝛽 2.
33 Prove that if a line is drawn parallel to one side of a triangle 5
intersecting the other two sides in distinct points, then the other two
sides are divided in the same ratio.
34 A solid is in the shape of a cone surmounted on a hemisphere, the 5
radius of each of them being 3.5 cm and the total height of solid is 9.5
cm. Find the volume of the solid
OR
In the given figure, there is a decorative block, made up of two solids - a
cube and a hemisphere. The base of the block is a cube of side 6 cm and
the hemisphere fixed on the top has a diameter of 3.5 cm. Find the total
surface area of the block.

136
35 If the median of the distribution given below is 27. Find the value of x 5
and y
Class 0-10 10-20 20-30 30-40 40-50 50-60 Total
interval
Frequency 5 x 20 14 y 8 68
SECTION E (CASE STUDY BASED)
36 Amit was playing a number card game. In the game,
some number cards (having both +ve or –ve
numbers) are arranged in a row such that they are
following an arithmetic progression. On his first
turn, Amit picks up 6th and 14th card and finds their
sum to be –76. On the second turn he picks up 8th
and 16th card and finds their sum to be – 96.

Based on the above information, answer the following questions: 2


(i) What is the difference between the numbers on any two
consecutive cards?
OR
What is the sum of 9th and 15th card? 1
(ii) Find the number on first card. 1
(iii) What is the number on the 19th card?
37 A group of Class X students goes to picnic during vacation, There were
three different slides and three
friends Ajay, Ram and Shyam
are sliding in the three slides.
The position of the three
friends shown by P, Q and R in
three different slides are given 2
below:
Consider O as orgin, answer the below questions:
(i) The co-ordinates of the point ‘Q’ which divides the line segment PR
in the ratio 1:2 internally: 1
OR 1
Find the co-ordinates of point on x-axis which is at equal distance PQ.
(ii) Find the distance PR.
(iii)Find the coordinates of the mid-point of PR.
38 Radio towers are used for transmitting a range of communication
services including radio and television. The tower will either act as an
antenna itself or support one or more antennas on its structure,
including microwave dishes. They are among the tallest human-made
structures. There are 2 main types: guyed and self-supporting
structures. On a similar concept, a radio station tower was built in two
sections A and B . Tower is supported by wires from a point O .
Distance between the base of the tower and point O is 36 m. From
137
point O , the angle of elevation of the top of section B is 30° and the
angle of elevation of the top of section A is 45°. (√3 = 1.732)

1
1

On the basis of the above information, answer the following questions:


(i) What is the height of the section B?
OR
What is the height of the section A?
(ii)What is the length of the wire structure from the point O to the top
of section A?
(iii) What is the length of the wire structure from the point O to the top
of section B?

138
Marking Scheme 3
S No Section A MARKS
1 B 1
2 B 1
3 C 1
4 D 1
5 D 1
6 C 1
7 C 1
8 A 1
9 A 1
10 C 1
11 C 1
12 A 1
13 D 1
14 A 1
15 A 1
16 D 1
17 A 1
18 B 1
19 C 1
20 B 1
21 𝑎1 𝑏1
For unique solution 𝑎2 ≠ 𝑏2
1 2 1
≠ 1
3 𝑘
K≠ 6
𝐴𝐷
22 =1
𝐷𝐵
𝐴𝐸 1
And 𝐸𝐶 =1
𝐴𝐷 𝐴𝐸 𝐴𝐷 𝐴𝐸
= 𝐸𝐶 ⇒𝐷𝐵 ∥ 𝐸𝐶 (by converse of BPT) 1
𝐷𝐵
23 Lengths of tangents from external points are equal
AP=AS
BP=BQ
CR=CQ
DR=DS 1
Adding above equation
AP+BP+CR+DR=AS+BQ+CQ+DS
(AP+BP) +(CR+DR)=(AS+ DS)+(BQ+CQ)
AB+CD=AD+BC 1
24 If tan (A + B) = √3
tan (A + B) = tan 60°
A + B = 60° ………………(i)
1
tan (A - B) =
√3
tan (A - B) = tan 30° 1
A - B = 30° ………………(ii)
Adding (i) and (ii), we have 2A = 90°

139
A = 45° Putting the value of A in (i), we have 45° + B = 60° \ B = 60° -
45° =15° Hence, A = 45° and B =15°. 1
OR
3
If sin A = 5
1
cos2A=1- sin2A
9 16
1-25=25 1
4
cosA=5
25 [Length of minute hand] = [radius of the circle] ⇒ r = 14 cm
ä Angle swept by the minute hand in 60 minutes = 360°
∴ Angle swept by the minute hand in 5 minutes = 360 /60 × 5 =30 ° 1
Now, area of the sector with r = 14 cm and θ = 30°
θ 30 22 154 1
π × r2 =360 × 7 14×14 cm = 3 cm2
360
OR
Diameter of the circle = 35 mm ∴ Radius (r) = 35/ 2 mm
22 35
Circumference = 2π r = 2 7 × 2 = 22 × 5 = 110 mm Length of 1 piece
of wire used to make diameter to divide the circle into 10 equal sectors 1
= 35 mm ∴ Length of 5 pieces = 5 × 35 = 175 mm
∴ Total length of the silver wire = 110 + 175 mm = 285 mm 1
26 Let us assume, that √5 is rational. So √5= a/b, a and b are coprime,
b≠0. 1
So, b√5 = a⋅
Squaring on both sides, we get 5b2 = a2.
Therefore, a2 is divisible by 5,
it follows that a is also divisible by 5.
So, we can write a = 5c for some integer c.
Substituting for a, we get 5b2 = 25c2, 1
that is, b2 = 5c2.
This means that b2 is divisible by 5, and
so b is also divisible by 5.
Therefore, a and b have at least 5 as a common factor.
But this contradicts the fact that a and b are co-prime. This 1
contradiction has arisen because of our incorrect assumption that 5 is
rational. So, we conclude that 5 is irrational.
27 4u2 + 8u= 4u(u + 2)
4u = 0 or u + 2 = 0, i.e., u = 0 or u = - 2 1
Therefore, the zeroes of 4u2 + 8u are 0 and - 2. 1
Sum of zeroes = 0 + (-2)=-2=−(8)/4=−(Coefficient of u)/Coefficient
of u2 1
Product of zeroes = 0 × (-2) = 0 =0/4=Constant term/Coefficient of u2
28 Let one of the numbers be x. So, the other number is (8 – x).
Now, x(8 – x) = 15 1
⇒ 8x – x2 = 15
⇒ x2– 8x + 15 = 0
⇒ x2– 5x – 3x + 15 = 0
⇒x(x – 5) – 3(x – 5) = 0 1
⇒ (x – 5) (x – 3) = 0
⇒ x = 3 or 5
∴ Thus, the two numbers are 3 and 5
140
OR 1
1 1 1 1 1
− = +
𝑎+𝑏+𝑥 𝑥 𝑎 𝑏
𝑥−𝑎−𝑏−𝑥 𝑎+𝑏 1
=
(𝑎 + 𝑏 + 𝑥 )𝑥 𝑎𝑏
– (a + b) ab = (a + b) (a + b + x) x
x2 + xa + bx + ab = 0
(x + a) (x + b) = 0,
x = – a, x = – b 1
29 Given: A circle with centre O; PA
and PB are two tangents to the
circle drawn from an external 1
point P.
To prove: PA = PB
Construction: Join OA, OB, and OP.
Proof 1
It is known that a tangent at any
point of a circle is perpendicular to the radius through the point of
contact.
OA⊥PA, OB⊥PB
In triangles OPA and OPB
∠OAP=∠OBP
OA=OB (radii) 1
OP=OP(common)
Therefore ∆OPA ≅∆OPB by RHS criterion.
Which means PA=PB
OR
Given: ABCD be a quadrilateral circumscribing a circle with centre O
such that it touches the circle at point P, Q, R, S.
To Prove: (i)∠AOB +∠COD = 180°
(ii) ∠BOC+∠DOA = 180°
Construction: Join the vertices
of the quadrilateral ABCD to the
centre of the circle.
Proof:
In ΔOAP and ΔOAS, 1
AP = AS (Tangents from the
same point)
OP = OS (Radii of the circle)
OA = OA (Common side)
ΔOAP ≅ΔOAS (SSS congruence
condition)
∴ ∠POA = ∠AOS
⇒∠1 = ∠8
Similarly, we get, ∠2 = ∠3 ∠4 = ∠5 ∠6 = ∠7
Adding all these angles, ∠1 + ∠2 + ∠3 + ∠4 + ∠5 + ∠6 + ∠7 +∠8 = 1
360°
⇒ (∠1 + ∠8) + (∠2 + ∠3) + (∠4 + ∠5) + (∠6 + ∠7) = 360°
⇒ 2 ∠1 + 2 ∠2 + 2 ∠5 + 2 ∠6 = 360°
⇒ 2(∠1 + ∠2) + 2(∠5 + ∠6) = 360°
141
⇒ (∠1 + ∠2) + (∠5 + ∠6) = 180°
⇒∠AOB + ∠COD = 180° 1
Similarly, we can prove that ∠ BOC + ∠ DOA = 180°

30 = sec A (1 – sin A) (sec A + tan A)


1 1 𝑠𝑖𝑛𝐴 1
= 𝑐𝑜𝑠𝐴(1- sin A) (𝑐𝑜𝑠𝐴+𝑐𝑜𝑠𝐴)
1 1
(1 -sin A) (1 + sin A)/ cos A
𝑐𝑜𝑠𝐴
=cos2A/cos2A =1 1
31 Total number of possible outcomes = 52 – 6 = 46
2 1
(i) P (a black king) =46 =23 1
20 10 1
(ii)P (a card of red colour) =46 =23
26 13
(iii)P (a card of black colour) =46 =23 1
32 Let the smaller number be x and the larger be y.
Now, x2 = 4y 1
Also, y2 – x2 = 45
⇒ y2– 4y = 45
⇒ y2– 4y – 45 = 0 1
⇒ y2– 9y + 5y – 45 = 0
⇒y (y – 9) + 5(y – 9) = 0
⇒ (y – 9) (y + 5) = 0 1
⇒ y = 9 or – 5
As natural numbers are not negative, so the larger number is 9.
Thus, x2 = 4(9) = 36 ⇒ x = 6 or – 6 1
Again, as natural numbers are not negative, so the smaller number is 6.
So, the two natural numbers are 9 and 6 1
OR
5 1
Then α+β=4, αβ=−4 2
1
α2+ β2= (α+β)2-2 αβ
5 1 25 1 27
=(4)2-2(-4)= 4 + 2 = 4 2
33 Given: A triangle ABC in which a line
parallel to side BC intersects other two
sides AB and AC at D and E respectively.
To Prove:AD/ DB = AE /EC
Construction: Join BE and CD and then
draw DM ⊥ AC and EN ⊥ AB
Proof:
ar(ADE) = 1 /2 AD × EN 2
Similarly, ar(BDE) = 1 /2 DB × EN,
ar(ADE) = 1/ 2 AE × DM and ar(DEC) = 1/ 2 EC × DM.
ar(ADE) 1/ 2 AD × EN 𝐴𝐵
= 1 /2 DB × EN, =𝐷𝐵 ____________________(I)
ar(BDE)
ar(ADE) 1/ 2 AE × DM 𝐴𝐸
= 1/ 2 EC × DM., =𝐸𝐶 ____________________(2)
ar(CDE)
Note that Δ BDE and DEC are on the same base DE and between the
same parallels BC and DE. So, ar(BDE) = ar(DEC) ___________________(3) 3

142
Therefore, from (1), (2) and (3), we have: AD /DB = AE/ EC
34 Radius of cone and hemisphere, r = 3.5 cm 1
Total height of solid (OC) = 9.5 cm 1
∴ Height of cone, h = 6 cm 1
Volume of solid = Volume of cone + Volume
of hemisphere 2
1 2 1
=3πr2h+3 πr3 =3πr2(h+2r)
1 22
=3 7 ×3.5×3.5(6+2×3.5) =166.83 cm3
OR 1
Total surface area of the block = Surface area of cube + Curved surface
area of hemisphere – Area of base of hemisphere 1
Surface area of cube = 6a2 = 6 × 62 = 216 cm2
Curved surface area of hemisphere – Area of base of hemisphere = 2πr2
– πr2 = πr2 2
22
= 7 . ×3.5×3.5= 9.625 cm2 1
Required total surface area of the block = 216 + 9.625 = 225.625 cm 2
35 The frequency distribution table for the given data can be drawn as:
Class
CLASS FREQUENC C MULATIVE
FREQUENCY
0-10 5 5
10-20 x 5+x 1
20-30 20 25+x
30-40 14 39+x
40-50 y 39+x+y
50-60 8 47+x+y
TOTAL 68
1
Here, N = 68, Median = 27, which lies in the interval 20-30.
𝑁
−𝐶𝐹
Median = l+ ( 2𝑓 )× h 1
34−(5+𝑥)
27=20 +( )×10
20
29−𝑥
⇒7= 1
2
x=15
Also, 47 + x + y = 68 ⇒ y = 68 – 47 – 15 ⇒ y = 6 1
36 (i) Let the numbers on the cards be a, a + d, a + 2d, ...
According to questions, 2
we have (a + 5d) + (a + 13d) = – 76
⇒ 2a + 18d = – 76
⇒ a + 9d = – 38 ------------------------------------------------------(A)
And (a + 7d) + (a + 15d) = – 96
⇒ 2a + 22d = – 96 ⇒ a + 11d = – 48 -----------------------------(B)
On subtracting (A) from (B), we get 2d = – 10 ⇒ d = – 5.
OR
Number on 9th card, a9 = a + 8d = 7 + 8 × – 5 = 7 – 40 = – 33
Number on 15th card, a15 = a + 14d = 7 + 14 × – 5 = 7 – 70 = – 63
Sum of 9th and 15th card = – 33 + (– 63) = – 129 1
(ii) From (A), a + 9(– 5) = – 38 ⇒ a = 7 Number on first card = a = 7
143
(iii)Number on 19th card, a19 = a + 18d = 7 + 18(–5) = – 83 1
37 The co-ordinate of P and R are (2, 5) and (8, 3) is
(i)
𝑚1 𝑥2+𝑚2 𝑥1 1×8+2×2
x= 𝑚1+𝑚2 = 3 =4 2
𝑚1 𝑦2+𝑚2 𝑦1 1×3+2×5 13
Y = 𝑚1+𝑚2 = 3 =3
The coordinates of Q (4, 13/3)
OR
Let A(x, 0) be the point on x-axis which is equidistant from P and Q,
13
then we have PA = QA The coordinate point of Q = ( 4, 3 )
⇒ PA 2 = QA2
13
(2 – x)2 + (5 – 0)2 = (4 – x)2 +( 3 - 0)2
13
X= 9
13
Hence the point( , 0)
9 1
(ii) √(𝑥2 − 𝑥1)2 + (𝑦2 − 𝑦1)2
√(8 − 2)2 + (3 − 5)2 =√62 + (−2)2 =√40 = 2√10 units
𝑥1+𝑥2 𝑦1+𝑦2
(iii) Mid-point of PR = 2 , 2
2+8 5+3 1
, = (5,4)
2 2
38. (i) In ΔOCB, tan 30° = 2

BC =
Hence, BC= 20.78 m
height of section B = 20.78 m
OR
Let 'x' be the height AB
in ΔOAC, tan 45° = AC/OC
1 = AC/36
AC = 36
height of section A = 36 m
(ii) In ∆ACO
𝐶𝑂
cos 45° =𝑂𝐴
1
1 36
=
√2 𝑂𝐴
OA= 36√2 m
(iii) In ∆BCO
𝐶𝑂
cos 30°=𝐵𝑂
√3 36
=
2 𝐵𝑂 1
BO= 24√3m

144
Class - X Session 2022-23
Subject - Mathematics (Basic)
Sample Question Paper 4
Time Allowed: 3 Hours Maximum Marks: 80
General Instructions: As per the previous sample question papers.
Section A
Section A consists of 20 questions of 1 mark each.
S. No MARKS
1 The graph of a polynomial is shown in Figure, then the number of its zeroes 1

is
(a) 3 (b) 1 (c) 2 (d) 4
2 1. The L.C.M. of x and 18 is 36. 1
2. The H.C.F. of x and 18 is 2.
What is the number x ?
(a) 1 (b) 2 (c) 3 (d) 4
3 The pair of equations 21x -13y = 10 and 3x +5y + 1 = 0has 1
(a) a unique solution (b) exactly two solutions
(c) infinitely many solutions (d) no solution
4 Value(s) of k for which the quadratic equation 2x2-kx+k=0 has equal roots 1
is/are (a) 0 (b) 4 (c) 8 (d) 0,
8
5 Mode is the 1
(a) middle most frequent value (b) least frequent value
(c) maximum frequent value (d) none of these
6 In the given figure, DE // BC . The value of EC is 1
(a) 1.5 cm (b) 3 cm (c) 2 cm (d) 1 cm

7 Assertion: For any two positive integers p and q, HCF (p, q) × LCM (p, q) = 1
p×q
Reason: If the HCF of two numbers is 5 and their product is 150, then their
LCM is 40
a.) Both Assertion and Reason are correct and Reason is the correct
explanation for Assertion
b.) Both Assertion and Reason are correct and Reason is not the correct
explanation for Assertion.
c.) assertion is true but the reason is false.
d.) both assertion and reason are false.
8 The distance of the point P( 2, 3) from the y -axis is 1
(a) 2 (b) 3 (c) 1 (d) 5
145
9 If sinα = 1and cos β =1, then the value of (α + β) is 1
(a) 0 (b) 30 (c) 60 (d) 90
10 If HCF (a, b) = 12 and a × b = 1800 then LCM (a, b) is 1
(a) 3600 (b) 900 (c) 150 (d) 90
11 In the adjoining figure, TP and TQ are the two 1
tangents to a

circle with centre O. If ∠POQ = 1100, then ∠PTQ is


(a) 600 (b) 700
(c) 80 0 (d) 900
12 In the given figure, OACB is a quadrant of a circle of 1
radius 7 cm. The perimeter of the quadrant is

(a) 11 cm (b) 18 cm
(c) 25 cm (d) (d)36 cm
13 If two solid hemispheres of same base radius r are joined together along 1
their bases, then curved surface area of this new solid is
(a) 4 π r2 (b) 6 π r2 (c) 3 π r2 (d) 8 π r2
14 A girl calculates that the probability of her winning the first prize in a 1
lottery is 0.08. If 6000 tickets are sold, then how many tickets has she
bought?
(a) 40 (b) 240 (c) 480 (d) 750
15 Consider the following frequency distribution of the heights of 60 students 1
of a class
HEIGHT 150- 155- 160- 165- 170- 175-
(in cm) 155 160 165 170 175 180
No of 15 13 10 8 9 5
students
The upper limit of the median class in the given data is
(a) 165 (b) 155 (c) 160 (d) 170
16 If x tan 45° sin 30° = cos 30° tan 30°, then x is equal to 1
1 1
(a) √3 (b) 2 (c) √2 (d) 1
17 5 sin² A + 5 cos² A + 1 is equal to 1
(a) 6 (6) -5 (c) 1 (d) -4
18 An equilateral triangle and isosceles triangle are always 1
(a) Similar
(b) Congruent
(c) Similar but not congruent
(d) Neither similar nor congruent
19 If the sum of the circumferences of two circles with radii R1 and R2 is equal 1
to circumference of a circle of radius R, then
(a) R1 + R2 = R
(b) R1 + R2 > R
(c) R1 + R2 < R
(d) Can’t say
20 Assertion :The distance point P(2,3) from the x-axis is 3. 1

Reason: The distance from x-axis is equal to its ordinary.

146
a.) Both Assertion and Reason are correct and Reason is the correct
explanation for Assertion
b.) Both Assertion and Reason are correct and Reason is not the correct
explanation for Assertion.
c.) assertion is true but the reason is false.
d.) both assertion and reason are false.
SECTION B
21 On comparing the ratios a1/a2, b1/b2, and c1/c2, find out whether the lines 2
representing the following pairs of linear equations at a point, are parallel
or coincident: 3x + 2y = 5 and 2x – 3y = 7
22 In the given figure, ∆ODC ~ ∆OBA, ∠BOC = 125° and ∠CDO = 70°. Find 2
∠DCO and ∠OAB.

23 Two concentric circles are of radii 5 cm and 3 cm. Find the length of the 2
chord of the larger circle which touches the smaller circle.
24 15 cot A = 8, find sin A andsecA 2
OR
If sin θ + cos θ =√2cosθ, then what is the value of tan θ ?
25 Area of a sector of a circle of radius 14 cm is 154 cm 2. Find the length of the 2
corresponding arc of the sector.

OR

In the given figure, AB is the


diameter where AP = 12 cm and
PB = 16 cm. Taking the value of π as
3, find the perimeter of the
shaded region.
Section C
26 Prove that √2 𝑖𝑠 𝑖𝑟𝑟𝑎𝑡𝑖𝑜𝑛𝑎𝑙 𝑛𝑢𝑚𝑏𝑒𝑟 3
27 Find the zeroes of the quadratic polynomial x2+4x-10, and verify the 3
relationship between the zeroes and the coefficients
28 Is it possible to design a rectangular park of perimeter 80 m and area 400 3
m2? If so, find its length and breadth.
Or
1 2 6
Find for x :𝑥−2 + 𝑥−1 = 𝑥

147
29 Prove that the angle between the two tangents drawn from an external 3
point to a circle is supplementary to the angle subtended by the line
segment joining the points of contact at the
centre.
OR
In figure, PA and PB are tangents to the circle
with centre O such that < APB = 50º. Write
the measure of <OAB
30 𝑐𝑜𝑠𝑒𝑐𝐴 𝑐𝑜𝑠𝑒𝑐𝐴 3
Prove that : :𝑐𝑜𝑠𝑒𝑐𝐴−1 + 𝑐𝑜𝑠𝑒𝑐𝐴+1 = 2 sec2A
31 From a pack of 52 playing cards, Jacks, Queens and Kings of red colour are 3
removed. From the remaining, a card is drawn at random. Find the
probability that drawn card is (i) a black king, (ii) a card of red colour, (iii)
a card of black colour.
SECTION D
32 An express train takes 1 hour less than a passenger train to travel 132 km 5
between Mysore and Bangalore (without taking into consideration the time
they stop at intermediate stations). If the average speed of the express train
is 11 km/h more than that of the passenger train, find the average speed of
the two trains.
OR
If x =− 4 is a root of the equation x2+2x+4p=0 , find the values of k for
which the equation x2+px(1+3k)+7(3+2k) =0 has equal roots
33 State and prove Basic proportionality theorem 5
34 A vessel is in the form of an inverted cone. Its height is 8 cm and the radius 5
of its top, which is open, is 5 cm. It is filled with water up to the brim. When
lead shots, each of which is a sphere of radius 0.5 cm are dropped into the
vessel, one-fourth of the water flows out. Find the number of lead shots
dropped in the vessel.
OR
The rain water from 22m ×20 m roof drains into cylindrical vessel of
diameter 2 m and height 3.5 m. If the rain water collected from the roof fills
4𝑡ℎ
of cylindrical vessel then find the rainfall in cm
5
35 If the median of the following frequency distribution is 32.5. Find the values 5
of f1 and f2 .
CLASS 0-10 10-20 20-30 30- 40- 50- 60- 70-
40 50 60 70 80
FREQUENCY f1 5 9 12 f2 3 2 40
Section E
36 Road Roller : A road roller (sometimes
called a roller compactor, or just roller) 1+1+2
is a compactor-type engineering vehicle
used to compact soil, gravel, concrete, or
asphalt in the construction of roads and
foundations. Similar rollers are used
also at landfills or in agriculture. Road rollers are frequently referred to as
steamrollers, regardless of their method of propulsion.
RCB Machine Pvt Ltd started making road roller 10 year ago. Company
increased its production uniformly by fixed number every year. The
company produces 800 roller in the 6th year and 1130 roller in the 9th
148
year.
(i) What was the company’s production in first year ?
(ii) What was the company’s production in the 8th year ?
(iii) What was the company’s total production of the first 6 years?
OR
In which year the company’s production was 1350 rollers ?
37 Ram, Shyam, Mohan and Kabir are four 1+1+2
friends. They want to play with toy
telephone. Their positions are
respectively A, A', A'', A''' as shown in the
figure. They have their toy telephones
connected to each other, telephones are
connected with the string. With the help
of above conditions, answer the following
questions:-
1) What are the coordinates of A, A', A'',
A'''?
2) Find the distance between Ram and
Shyam, Ram and Mohan.
3) What are the coordinates of, the mid-point of the string which is
connected to Ram and Kabir?
OR
What are the coordinates of, the mid-point of the string which is connected
to mohan and Shyam?
38 Vijay is trying to find the average height of a tower near his house. He is 1+1+2
using the
properties
of similar
triangles.
The height
of Vijay’s
house if
20m when
Vijay’s
house
casts a
shadow
10m long on the ground. At the same time, the tower casts a shadow 50m
long on the ground and the house of Ajay casts 20m shadow on the ground.
1. What is the height of the tower?
2. What will be the length of the shadow of the tower when Vijay’s house
casts a shadow of 12m?
3. When the tower casts a shadow of 40m, same time what will be the
length of the shadow of Ajay’s house?
OR
When the tower casts a shadow of 40m, same time what will be the length
of the shadow of Vijay’s house?

149
Mathematics Basic (241)
Marking Scheme Paper 4
S No Section A MARKS
1 (a) 3 1
2 (d) 4 1
3 (a) unique solution 1
4 (d) 0,8 1
5 (c) maximum frequent value 1
6 (c) 2 cm 1
7 (c)assertion is true but the reason is false. 1
8 (a) 2 1
9 (d) 900 1
10 (c) 150 1
1 (b) 700 1
12 (c) 25 cm 1
13 (a) 4 π r2 1
14 (c) 480 1
15 (a) 165 1
16 (d) 1 1
17 (a) 6D Neither similar nor congruent 1
18 (d) Neither similar nor congruent 1
19 a) R1 + R2 = R 1
20 a.) Both Assertion and Reason are correct and Reason is the correct 1
explanation for Assertion
Section B
21 Given : 3x + 2y = 5 or 3x + 2y – 5 = 0
and 2x – 3y = 7 or 2x – 3y – 7 = 0
Comparing the above equations with a1x + b1y + c1=0
And a2x + b2y + c2 = 0
We get,
a1 = 3, b1 = 2, c1 = -5 1
a2 = 2, b2 = -3, c2 = -7
a1/a2 = 3/2, b1/b2 = 2/-3, c1/c2 = -5/-7 = 5/7
Since, a1/a2≠b1/b2 the lines intersect each other at a point and have
only one possible solution. Hence, the equations are consistent. 1
22 From the given figure
∠DOC+125=180 (linear pair)
∠DOC=180-125=55 1
Now in ΔDOC
∠DCO+∠ODC+∠DOC=180 (Angle sum property of triangle)
∠DCO+70+55=180
∠DCO=180-125=55
Now ΔODC~ΔOBA (Given)
∠OAB=∠OCD=55
Hence ∠DOC=55, ∠DCO=55 and ∠OAB=55 1

150
23 In the given figure, PQ is the chord of the larger circle, which
touches the smaller circle at R)
We have OP=OQ=5cm (radii of larger circle)
And OR=3cm (radii of smaller circle)
Since PQ is tangent to smaller circle
OR⊥ PQ (by theorem)
In ΔOPR and ΔOQR
∠ORP=∠ORQ (each of 90)
OR=OR (common)
OP=OQ (radii of same circle)
ΔOPR≅ΔOQR (by RHS congruence) 1
PR=RQ (CPCT)
In ΔOPR
PR2=OP2-OR2=52-32=16 =42
PR=4cm 1
PQ =2PR=2×4=8cm
24 15 cot A =8
8
Cot A=15
Let AB=8k and BC=15k
Let in right angled ΔABC
AC2=AB2+BC2 (Pythagoras theorem)
AC =(8k) +(15) =64k +225k2=289k2=(17k)2
2 2 2 2
1
AC=17k
𝐵𝐶 15𝑘 15
Sin A=𝐴𝐶 =17𝑘 =17
𝐴𝐶 17𝑘 17
Sec A=𝐴𝐵= 8𝑘 = 8 1
OR
We have sinθ+cosθ=√2cosθ
Dividing both sides by cosθ, we get
sinθ cosθ cosθ
+ = √2 1
cosθ cosθ cosθ
tanθ+1=√2
tanθ=√2-1 1
1
25 lr=154 1
2
1
l×14=154
2
7l=154 ⇒ l=22cm
Length of corresponding arc, 1
l=22cm
OR
From Pythagoras theorem we have
AB =√(16)2+(12)2
= √256 + 144 1
=√400 =20cm
Radius of circle = 10cm
Perimeter of shaded region =πr+AP+PB=3×10+12+16 1
=30+12+16=58

Section C
151
26 Proof of √2is an irrational numbers. 1
Assume, √2is a rational number, it can be written as p/q, in
which p and q are co-prime integers and q≠0,
i.e. √2= p/q. where, p and q are coprime numbers, and q≠0.
On squaring both sides of the above equation;
2= (p/q)2
2 = p2/q2
2q2 = p2 ...(i)
p2 is a multiple of 2 so p is a multiple of 2 ...(ii)
Since, p is a multiple of two.
So p = 2m
p² = 4m² …(iii) 1
Using equation(i) into the equation (iii), we get;
2q² = 4m²
So q² = 2m²
q2 is a multiple of 2 so q is a multiple of 2 ...(iv)
Equation (ii) and (iv), implies that p and qdhave a common factor 2.
It contradicts the fact that they are co-primes which lead from our
wrong assumption that 2is a rational number.
Hence, 2 is an irrational number(proved) 1
27 Let p(x)=x2+3x-10
For zeroes of polynomial p(x) =0
x2+7x-10=0 1
⇒x2+5x-2x-10
⇒x(x+5)-2(x+5)=0
⇒(x+5)(x-2)=0
So x=2 and x=-5
Therefore, α=2, and β=-5 are the zeroes of the given polynomial 1
Verification
−3 𝑐𝑜𝑓𝑓𝑖𝑐𝑖𝑒𝑛𝑡 𝑜𝑓 𝑥
Sum of zero α+β=(2)+(-5)=-3= 1 =- 𝑐𝑜𝑒𝑓𝑓𝑖𝑐𝑖𝑒𝑛𝑡 𝑜𝑓 𝑥2
10 𝑐𝑜𝑛𝑠𝑡𝑎𝑛𝑡 𝑡𝑒𝑟𝑚 1
Product of zero αβ= (2)(-5)=-10= - 1 =𝑐𝑜𝑒𝑓𝑓𝑖𝑐𝑖𝑒𝑛𝑡 𝑜𝑓 𝑥2
28 Let the length of rectangular park be x
Then perimeter of rectangular park=2(length+breadth)
2(x+ breadth)=80 1
Breadth =40-x
Area of rectangular park=length×breadth
x(40-x)=400
40x-x2=400
x2-40x+400=0 1
x2-20x-20x+400=0
(x-20)(x-20)=0
X=20 Thus the rectangular park is possible to design.
So length of park is 20m and its breadth is 40-20=20m 1
OR
1 2 6
We have 𝑥−2 + 𝑥−1 =𝑥
2𝑥−1+2𝑥−4 6
=𝑥 1
(𝑥−1)(𝑥−2)
3x2-5x=6x2-18x+12

152
3x2-13x+12=0
3x2-4x-9x+12=0 1
X(3x-4)-3(3x-4)=0 1
(3x-4)(x-3)=0
4
so x=3 and 3
29 PA and PB are two tangents, A and B are the points of contact of the
tangents.
OA⊥AP and OB⊥BP 1
∠OAP=∠OBP=90
(Radius and tangent are
perpendicular to each other)
In quadrilateral OAPB 1
∠OAP+∠OBP+∠APB∠AOB=360
90+90+∠APB+∠AOB=360
=∠APB+∠AOB=360-180=180
Hence ∠APB and ∠AOB are 1
supplementary angle
OR
We have ∠APB=50
180−50
∠PAB=∠PBA= 2 =65 1
Here OA is radius and AP is tangent at A , since radius is
1
always perpendicular to tangent at point of contact,we have
∠OAP=90
Now ∠OAB=∠OAP- ∠PAB
1
90-65=25
30 𝑐𝑜𝑠𝑒𝑐𝐴 𝑐𝑜𝑠𝑒𝑐𝐴
+
𝑐𝑜𝑠𝑒𝑐𝐴−1 𝑐𝑜𝑠𝑒𝑐𝐴+1 1
𝑐𝑜𝑠𝑒𝑐 𝐴 + 𝑐𝑜𝑠𝑒𝑐𝐴 + 𝑐𝑜𝑠𝑒𝑐 2 𝐴 − 𝑐𝑜𝑠𝑒𝑐𝐴
2

(𝑐𝑜𝑠𝑒𝑐𝐴 − 1)(𝑐𝑜𝑠𝑒𝑐𝐴 + 1) 1
2𝑐𝑜𝑠𝑒𝑐 2 𝐴 2𝑐𝑜𝑠𝑒𝑐 2 𝐴
=
𝑐𝑜𝑠𝑒𝑐 2 𝐴−1 𝑐𝑜𝑡 2 𝐴
2 1
𝑠𝑖𝑛2𝐴
𝑐𝑜𝑠2𝐴 = 2sec2A
𝑠𝑖𝑛2𝐴

31 There are total 52 cards out of which 6 cards are removed. 1 for
Total number of all possible outcomes N(S)=52-6=46 each
Number of black kingn(E1)=2
(i)a black king
𝑛(𝐸1) 2 1
Probability of drawing black king p(E1)= 𝑛(𝑆) =46 =23
(ii) a card of red colour
Total red cards n(e2)=26-6=20
𝑛(𝐸2) 20 10
Probability of drawing red colour card P(E2)= 𝑛(𝑆) =46 =23
(iii) a card of black colour,
Total card of black colour n(e3)=26
𝑛(𝐸3) 26 13
Probability of drawing a black colour card P(E3)= 𝑛(𝑆) =46 =23
Section D

153
32 Let speed of passenger train=x km/h
Then the speed of express train =(x+11) km/h
According to question
132 132
- 𝑥+11 =1 2
𝑥
132(𝑥+11)−132𝑥
=1
𝑥(𝑥+11)
132𝑥 + 1452 − 132𝑥 =x2+11x
x2+11x-1452=0 1
x2+44x-33x-1452=0 1
x(x+44)-33(x+44)=0
(x-33)(x+44)=0
x=33 or x=-44 (rejected)
Hence speed of passenger train =33Km/h 1
And speed of express train =33+11=44Km/h
OR
We have x2+2x+4p=0 1
Since x=-4 is root of above equation It must satisfy it
(-4)2+ (2×-4)+4p=0, p=-2
Since equation x2-2(1+3k)x+7(3+2k)=0 has equal roots 1
4(1+3k)2-28k(3+2k)=0
9k2-8k-20=0 1
−10 1+1
(9k+10)(k-2)=0, therefore K= 9 and 2
33 If a line is drawn parallel to one side of a triangle intersecting the 1
other two sides,then it divides the two sides in the same ratio.
For fig, given, to prove and construction 2
For correct proof 2
34 Radius of cone, r=5cm
Height of cone, h=8cm
1
Volume of water in cone=3π(5)2×8
200𝜋
= cm3 1
3
1 200𝜋 50𝜋
Volume of water flow out=4 × 3 = 3 cm3 1
Radius of one spherical shot=0.5cm
4 𝜋
Volume of I spherical shot=3π(0.5)3 = 6
1
𝑉𝑜𝑙𝑢𝑚𝑒 𝑜𝑓 𝑤𝑎𝑡𝑒𝑟 𝑓𝑙𝑜𝑤𝑠
Number of lead shot dropped=𝑣𝑜𝑙𝑢𝑚𝑒 𝑜𝑓 1 𝑠𝑝ℎ𝑒𝑟𝑖𝑐𝑎𝑙 𝑠ℎ𝑜𝑡
50𝜋 1
3 50×6
= 𝜋 = =100
6
3 1
OR
Let h be the rainfall
Volume of water collected in cylindrical vessel
4 2 4 7
πr h =5×π×(1)2×2
5
44
= 5 m3
Rain water from roof=22×20×h 2
Now
44
22×20×h= 5 1
44 1 1
h= 5 × 20×20 =50 m
154
100
=2cm
50

2
35 CLASS FREQUENCY (f) CUMULATIVE FREQUENCY (cf)
0-10 f1 f1 2
10-20 5 f1+5
20-30 9 f1+14
30-40 12 f1+26
40-50 F2 f1+ f2+26
50-60 3 f1+ f2+29
60-70 2 f1+ f2+31
40
f1+ f2+31=40
f1+ f2=9
f2=9-f1
Since median is 32.5,which lies in30-40,median class is 30-40
𝑁 1
Here l=30, 2 =20, f=12 and cf=14+f1 , median=32.5
𝑁
−𝑐𝑓
2
L+( )×h = median
𝑓 1
20−(14+𝑓1)
30+ ( )×10=32.5
12
6−𝑓1
( )×10 =2.5
12
60−10𝑓1
( 12 ) =2.5
60 − 10𝑓1 =30
10f1=30 so f1=3
f2 =9-3=6 1
Section E
36 (i)a=250 1+1+2
(ii)a8=a+(8-1)d
=250+7×110=1020
𝑛
(iii)Sn= 2 [a+an]
6
=2[250+800]
=3×1050=3130
OR
Suppose company produce 1350 rollers in nth year
Then an=a+(n-1)d
1350=250+(n-1)110
(n-1)110=1350-250=1100
n-1=10
n=11
37 1). (3,2),(3,-2),(-3,2),(-3,-2) 1+1+2
2). 4units,6units
3). (0,0)
OR (0,0)
38 1)100 2)60m 3)16m 1+1+2
OR 8m

155
Class - X Session 2022-23
Subject - Mathematics BASIC (241)
SAMPLE PAPER 5

Time Allowed: 3 Hours Maximum Marks: 80

General Instructions: As per the previous question papers.


Section A
Section A consists of 20 questions of 1 mark each.
S No. Marks
1 The quadratic polynomial, sum of whose zeroes is -5 and their product is 6, is 1
(a)x2 + 5x + 6 b) x2 - 5x + 6 c) x2 + 5x - 6 d) x2 - 5x - 6
2 If LCM(a,18) = 36 and HCF( a, 18) = 2 then a =…..? 1
a)2 b) 3 c) 4 d) 1
3 The pair of equations x+ 2y+ 5= 0 and −3x-6y+ 1 = 0 represents 1
(a) Parallel lines (b) Intersecting lines
(c) Coincident lines (d) None of these
4 Which of the following is a quadratic equation? 1
a) x3−5x2+ 7x −1 = 0 b) x(2x+3) = x2+5
c) x 2x = 0 d) x2+ ½ = 5
5 If p and q are positive integers such that p=a×b×b and q = a×a×b, where a, b 1
are prime numbers, then LCM of (p, q)is
(a) ab (b) a×a×b×b (c)a×a×a×b×b (d) a×a×a×b×b×b
6 If the distance between the points (4,p) and (1,0)is 5, then value of p is 1
(a) 4only (b)±4 (c) -4only (d) 0
7 A square and a rhombus are always 1
(a) similar (b) congruent
(c) similar but not congruent (d) neither similar nor congruent
8 The value of sin60°cos30°+cos60°sin30°is 1
(a)1 (b) –1 (c) 0 (d) none of these
9 The radius of a circle whose circumference is equal to the sum of the 1
circumferences of the two circles of diameters 36 cm and 20 cm is
a) 56 cm (b) 42 cm (c) 28 cm (d) 16 cm
10 Two identical solid cubes of side 𝑘units are joined end to end. What is the 1
volume, in cubic units, of the resulting cuboid?
(a) k3 (b) 2k3 (c) 3k3 (d) 6k3
11 If sin (A +B)=1andcos(A– B)= √3/2, then the value of A and B, respectively are 1
(a)45°and15° (b)30°and60° (c) 60°and30° (d) 15° and 45°
12 In a frequency distribution, the mid value of a class is 15 and the width of the 1
class is 4 . The lower limit of the class is ?
(a) 13 (b) 11 (c) 19 (d) 15
13 If the angle between the radii of a circle is 1200, then the angle between the 1
tangents at the end of these two radii is
(a) 500 (b)600 (c) 800 (d)900
14 The mid-point of the line segment joining the points A(-2,8) and B(-6,-4)is 1
(a) (-4,-6) (b)(2,6) (c)(-4,2) (d) (4,2)

156
15 sinA = cosA is true when A= 1
(a)0° (b)30° (c)45° (d)any angle
16 The class marks of a frequency distribution are given as follows: 1
15, 20, 25, ....The class corresponding to the class mark 20 is:
(a) 12.5 -17.5 (b) 17.5-22.5 (c) 18.5 -21.5 (d) 19.5 -20.5
17 If the probability of an event is p, the probability of its complementary event 1
will be:
1
(a) 𝑝 (b)-p (c) 1-p (d)1+p
18 The ratio of area of two circles whose ratio of circumference is 3:1 will be 1
a) 3:1 b) 1:3 c) 1:9 d)9:1
Direction for question 19 & 20: In question numbers 19 and 20, a statement of
Assertion (A) is followed by a statement of reason (R). Choose the correct
option.
19 Assertion: 2 is an example of a rational number. 1
Reason: The square roots of all positive integers are irrational numbers.
(a)Both Assertion (A) and Reason (R) are true and Reason (R) is the correct
explanation of Assertion (A).
(b) Both Assertion (A) and Reason (R) are true but Reason (R) is not the
correct explanation of Assertion (A).
(c) Assertion (A) is true but Reason (R) is false.
(d) Assertion (A) is false but Reason (R) is true.
20 Assertion: The point (3,0) lies on x-axis. 1
Reason: The y-coordinate of a point on x_ axis is zero.
(a)Both Assertion (A) and Reason (R) are true and Reason (R) is the correct
explanation of Assertion (A).
(b) Both Assertion (A) and Reason (R) are true but Reason (R) is not the
correct explanation of Assertion (A).
(c) Assertion (A) is true but Reason (R) is false.
(d) Assertion (A) is false but Reason (R) is true.
Section B
Section B consists of 5 questions of 2 marks each.
21 The inner circumference of a circular track is 220 m. The track is7m wide 2
everywhere. Calculate the cost of putting up a fence along the outer circle at the
rate of ₨ 2permetre.
OR
Find the area of a sector of a circle of radius 28 cm and central angle 45°.
22 Find the value of 2cos260°+3sin 245°-3sin 230°+2cos2900 2
OR
If x.tan60°cos60° = sin60°cot60° ,then find the value of x.
23 Prove that: The lengths of tangents from an external point to a circle are equal. 2

24 For what value of k, do the equations 2


3x– y+ 8=0and 6x-ky=−16represent coincident lines?

157
25 In given figure, ΔODC ~ ΔOBA, ∠ BOC = 125° and ∠ CDO = 70°. Find ∠ DOC and 2
∠ OAB.

Section C
Section C consists of 6 questions of 3 marks each.
26 Given that √2 𝑖𝑠 𝑖𝑟𝑟𝑎𝑡𝑖𝑜𝑛𝑎𝑙, prove that 3+2√2 is also irrational. 3
27 Find the zeroes of the following quadratic polynomials and verify the 3
relationship between the zeroes and the coefficients : 6x 2 – 3 – 7x
28 4 chairs and 3 tables cost Rs2100 and5 chairs and 2 tables cost Rs1750. Find 3
the cost of one chair and one table separately.
OR
The sum of the digits of a two digit number is 11 . The number obtained by
interchanging the digits exceeds the original number by 27. Find the number.
29 Prove that the parallelogram circumscribing a circle is a rhombus. 3

30 Prove that: If x =acosθ–bsinθ and y = asinθ+bcosθ, then 3


a 2+ b2=x2+ y2
OR
𝑠𝑖𝑛𝜃 𝑐𝑜𝑠𝜃
+ 1−𝑡𝑎𝑛𝜃 = sin 𝜃+ cos 𝜃
1−cot 𝜃
31 All the black face cards are removed from a pack of 52playing cards. The 3
remaining cards are well shuffled and then a card is drawn at random. Find
the probability of getting a:
(i) face card (ii)red card (iii) black card
Section D

Section D consists of 4 questions of 5 marks each.


32 Find the values of k for which the quadratic equation 5
(3k + 1)x2 + 2(k + 1)x + 1 = 0 has equal roots. Also find the roots.
OR
Ram takes 6 days less than Bhagat to finish a piece of work. If both of them
together can finish the work in 4 days, in how many days Bhagat alone can
finish the work..
33 Sides AB and AC and median AD of a triangle ABC are respectively proportional 5
to sides PQ and PR and median PM of another triangle PQR. Show that
ΔABC ~ ΔPQR.

158
34 A rocket is in the form of a right circular cylinder closed at the lower end and 5
surmounted by a cone with the same radius as that of the cylinder. The
diameter and height of the cylinder are 6 cm and 12 cm, respectively. If the
the slant height of the conical portion is 5 cm, find the total surface area and
volume of the rocket [Use π = 3.14].
OR
A solid right circular cone of height 120 cm and radius 60 cm is placed in a
right circular cylinder full of water of height 180 cm such that it touches the
bottom. Find the volume of water left in the cylinder, if the radius of the
cylinder is equal to the radius of the cone
35 The mean of the following distribution is 62.8. Find the missing frequency x. 5
C. I. 0-20 20-40 40-60 60-80 80-100 100-120
F 5 8 x 12 7 8
Section E
Case study based questions are compulsory.
36 Raman is a contractor; he is 1+1+
constructing a building of many 2
floors. The building has interesting
design on each floor. It has open
space thus number of rooms
decreases as we go up. The
number of bricks required for
construction of a room is 1500.
The number of bricks required for
ground floor is 30,000 on 1 stfloor, it is 27,000 on2nd floor, it is 24,000 on third
floor and so on. Answer the following questions based on the above situation:
(i) The given situation is an AP or not? If it is an AP then find the
common difference?
(ii) Find the total number of rooms on the ground floor.
(iii) Find the maximum numbers of floors that can be build?
OR
Find the cost of bricks for first three floors at the rate of ₨10 per brick?
37 Two friends Seema and Aditya work in the same office at Delhi. In the
Christmas vacations, both decided to go to their hometown represented by 1+1+
Town A and Town B respectively in the figure given below. Town A and Town B 2
are connected by trains from the same station C in Delhi. Based on the given
situation answer the following questions

(i) Write the coordinates of town A and town B from the graph.
(ii) Find the distance travelled by Seema to her home town.
159
(iii) Write the coordinates of midpoint of town A and town C?
OR
Who travelled more distance, Seema or Aditya to reach their hometown?
Explain why?
38 A person is white washing a wall with the help of a ladder which is kept as 1+1+
shown in the following figure; 2

When he wants to paint at higher or lower points, he should change the


position of ladder and 𝜃.
(i) If the person wants to whitewash at a higher point on the wall, what
happens to the angle made by the ladder with the ground?
(ii) If the person wants to white wash at a higher point on the wall, what will
be the change in the distance AB?
(iii)Suppose one end of the ladder of length 3 m reaches the top of a wall,
other end is on the ground and makes an angle of 450 with the ground. Find
the height of the wall.
OR
The ladder of length 3 m reaches the top of a wall, other end is on the
ground and makes an angle of 450 with the ground. Find the distance of foot
of the ladder from the wall.

160
Marking Scheme
Class- X, Session- 2022-23
SAMPLE PAPER Basic 5

Section A
Q No. Marks
1 (a)x2 + 5x + 6 1
2 (c) 4 1
3 (a)Paralle llines 1
4 (b) x(2x+3) = x2+5 1
5 (b) a×a×b×b 1
6 (b)±4 1
7 (d) neither similar nor congruent 1
8 (a)1 1
9 (c) 28 cm 1
10 (b)2K3 1
11 (c) 60°and30° 1
12 (a) 13 1
13 (b)600 1
14 (c)(-4,2) 1
15 (c)45° 1
16 (b) 17.5-22.5 1
17 (c) 1-p 1
18 (d)9:1 1
19 (c) Assertion (A) is true but Reason (R) is false. 1
20 (a) Both Assertion 1
SECTION B
21 Circumference of inner side = 220m
220×7
⇒2πr=220⇒r= 44 =35m
1
Now, width of track =7m
∴ Outer radius =35+7=42m
22 ½
Therefore outer circumference =2πR=2× 7 ×42=264m ½
∴ Cost of fencing = Rs. (264×2) = Rs. 528
OR
Radius of sector r=28 cm
Central angle θ=45∘ 2
θ
Area of sector = 360° ×πr2
45° 22
=360°× 7 ×28×28=308cm2
5
22 Putting correct values and getting answer 4 2
OR
1
X=
√3
23 For given, to prove , construction and correct figures. 1
For correct proof. 1
24 The system of equations has infinitely many solutions( coincident lines), 1
a 𝑏 c
if 𝑎1 = 𝑏1 = 𝑐1 .
2 2 2 161 1
3 −1 8
, 6 = −𝑘 = 16
So ⇒k= 2
25 ∠DOC +125° = 180° (linear pair) ½
⇒∠DOC = 180° - 125° = 55°
In △DOC, ∠DCO + ∠CDO + ∠DOC = 180°
(sum of the angles of △ODC)
⇒∠DCO + 70° + 55° = 180°
⇒∠DCO + 125° = 180°
⇒∠DCO = 180° - 125° = 55°
Now, we are given that △ODC∼△OBA
⇒∠OCD = ∠OAB 1½
(Corresponding angles of similar triangles)
⇒∠OAB = ∠OCD = 55°
∴∠OAB =55°
Section C
26 Let us assume, to the contrary, that3+2√2 is rational.
So, we can find integers p and q (q ≠ 0), such that 1½
𝑝
3+2√2=𝑞 .
𝑝−3𝑞
⇒√2= 2
⇒√2isarationalnumber but it is given that √2 is irrational
Hence our assumption is wrong 1½
So, we conclude that3+2√2 is irrational.
27 Comparing given polynomial with the general form ax2+bx+c,
We get, a = 6, b = - 7 and c = -3
We have 6x2−7x−3 = 6x2−9x+2x−3
= 3x(2x - 3) +1(2x - 3)
= (3x + 1) (2x - 3) 1½
So 3x + 1 = 0 or 2x - 3 = 0,
1 3
i.e., x=−3 or x = 2
1 3 7 1½
Sum of zeroes = -−3 +2=6=−(Coefficient of x)/Coefficient of x2
1 3 1 3
Product of zeroes =-−3 x 2=−2=−6=Constant term/Coefficient of x2.
28 Let the cost of a chair be Rs. x and that of a table be Rs. y. Then, 1
4x+3y=2100
and, 5x+2y=1750 1
Solving we get x= 150 and y= 500 1
∴ cost of the 1 chair is Rs150 and the cost of 1 table is Rs500.
OR
Let the unit digit and tens digits of the number be y and x
Number = 10x + y 1
Number after reversing the digits = 10y + x
According to the question,
.⇒ x + y = 11 ... (i) 1
Also, 10y + x = 10x + y +27
⇒x−y=−3 …(ii)
Solving equation (i) and (ii), we get
⇒y=7 1
Putting the value in equation (i), we get
162
⇒x=4
Hence, the number is 10 × 4 + 7 = 47.

29 Since ABCD is a parallelogram,


AB = CD …(i) and BC = AD …(ii)
As the lengths of tangents drawn from an external point to a circle are
equal 1
DR = DS, CR = CQ , BP = BQ, AP = AS
Adding all these equations, we obtain
DR + CR + BP + AP = DS + CQ + BQ + AS 1
(DR + CR) + (BP + AP) = (DS + AS) + (CQ + BQ)
CD + AB = AD + BC
On putting the values of equations (i) and (ii) in this equation, we obtain
2AB = 2BC or AB = BC …(iii) 1
Comparing equations (i), (ii), and (iii), we obtain
AB = BC = CD = DA, Hence, ABCD is a rhombus.
30 LHS =x2+y2
= a2 𝑠𝑖𝑛² 𝜃 + 2ab 𝑠𝑖𝑛 𝜃𝑐𝑜𝑠 𝜃 + b2 𝑐𝑜𝑠² 𝜃 + a2 𝑐𝑜𝑠² 𝜃 - 2
2ab 𝑠𝑖𝑛 𝜃𝑐𝑜𝑠 𝜃 + b2 𝑠𝑖𝑛² 𝜃
= a2( 𝑠𝑖𝑛² 𝜃 + 𝑐𝑜𝑠² 𝜃) + b2( 𝑠𝑖𝑛² 𝜃 + 𝑐𝑜𝑠² 𝜃) 1
= a2+ b2 as 𝑠𝑖𝑛² 𝜃 + 𝑐𝑜𝑠² 𝜃 =1
Or
𝑐𝑜𝑠 𝜃 sin θ
+ 1 − cot θ
1 − tan θ

𝑐𝑜𝑠 𝜃 𝑠𝑖𝑛 𝜃 1
= sin θ + cos θ
1− 1−
cos θ sin θ
𝑐𝑜𝑠² 𝜃 𝑠𝑖𝑛² 𝜃
= -
cos θ − sin θ 𝑐𝑜𝑠 θ − sin θ
2
(𝑐𝑜𝑠 𝜃 − 𝑠𝑖𝑛 𝜃)(𝑐𝑜𝑠 𝜃 + 𝑠𝑖𝑛 𝜃
= ) = cos θ + sin θ
(cos θ − sin θ)

31 All the black face cards are removed from the pack. Then, 1 for
Number of remaining cards = 52 − 6 = 46 each
∴ Total number of outcomes = 46
(i) There are 6 face cards (2 red jacks, 2 red queens and 2 red kings) in
the remaining pack of cards.
So, the favourable number of outcomes are 6.
𝐹𝑎𝑣𝑜𝑢𝑟𝑎𝑏𝑙𝑒 𝑛𝑢𝑚𝑏𝑒𝑟𝑜𝑓𝑜𝑢𝑡𝑐𝑜𝑚𝑒𝑠 6 3
∴P(getting a face card) = = =
Totalnumberofoutcomes 46 23
(ii) There are 26 red cards in the remaining pack of cards.
So, the favourable number of outcomes are 26.
26 13
∴P(getting a red card) =46=23
(iii) Number of black cards in the remaining pack of cards = 26 − 6 = 20
So, the favourable number of outcomes are 20.
20 10
∴ P(getting a black card) =46=23

163
Section D
32 Here, a=3k+1, b=2(k+1) and c=1.
As we know that D=b2-4ac
D=4k2+8k+4-12k-4=4k2-4k
The given equation will have real and equal roots, if D = 0
Thus, 4k2-4k=0 1
⇒4k(k-1)=0⇒k=0 or k-1=0⇒k=0 or k=1
Therefore, the value of k is 0 or 1.
Now, for k = 0, the equation becomes 1
x2+2x+1=0⇒x2+x+x+1=0⇒x(x+1)+1(x+1)=0⇒(x+1) (x+1)=0⇒x=-1, -1
or k = 1, the equation becomes 1
4x2+4x+1=0⇒4x2+2x+2x+1=0⇒2x(2x+1)+1(2x+1)=0⇒(2x+1)
(2x+1)=0⇒x=-½, -½
Hence, the roots of the equation are -1 and -½. 2
OR
Let Ram takes x days to complete work
So, Bhagat takes x+6 days to complete work
1
Ram can do a part of work in 1 day = 𝑥
1
Bhagat can do a part of work in 1 day = 𝑥+6
We are given that together can finish the work in 4 days 2
1 1 1
So, 𝑥 + 𝑥+6= 4 1
8x+24=x2+6x 2
x2−2x−24=0
Solving x= -4, 6 Discarding –ve value we get x=6
Ram takes 6 days to complete work
Bhagat takes x+6 = 6+6 =12 days to complete work
33

Given, two triangles ΔABC and ΔPQR in which AD and PM are medians such that
𝐴𝐵 𝐴𝐶 𝐴𝐷
= = 1
PQ PR PM
To Prove that ΔABC∼ΔPQR
Construction: Produce AD to E so that AD = DE. Join CE.
Similarly produce PM to N such that PM = MN, also Join RN.
Proof: In ΔABD and ΔCDE, we have
AD = DE [By Construction] 1
BD = DC [∵ AD is the median]
And, ∠ADB=∠CDE [Vertically opposite angles]
∴ΔABD≅ΔCED [By SAS criterion of congruence]
⇒AB=CE[byCPCT]...(i) 1
Also, in ΔPQM and ΔMNR, we have
PM = MN [By Construction]
QM = MR [∴PM is the median]
164
And, ∠PMQ=∠NMR [Vertically opposite angles]
∴ΔPQM=ΔMNR [By SAS criterion of congruence]
⇒PQ=RN[CPCT]...(ii) 1
𝐴𝐵 𝐴𝐶 𝐴𝐷
Now,PQ=PR=PM
𝐶𝐸 𝐴𝐶 𝐴𝐷
⇒RN=PR=PM...[From(i)and(ii)]
𝐶𝐸 𝐴𝐶 2𝐴𝐷
⇒RN=PR=2PM
𝐶𝐸 𝐴𝐶 𝐴𝐸
⇒RN=PR=PN...[∴2AD=AE and 2PM=PN]
∴ΔACE∼ΔPRN [By SSS similarity criterion] 1
Therefore, ∠2=∠4
Similarly, ∠1=∠3
∴∠1+∠2=∠3+∠4
⇒∠A=∠P...(iii)
Now, in ΔABC and ΔPQR, we have
𝐴𝐵 𝐴𝐶
= (Given)
PQ PR
∠A=∠P[From(iii)]
∴ΔABC∼ΔPQR [By SAS similarity criterion]
34 h2=l2−r2=4cm
Total surface area of rocket= Curved surface area of cylinder + Curved surface area
of cone + Area of base circle of the cylinder 1
=2πrh+ πrl+ πr2
= 301.44 cm2
Hence, total volume of rocket= volume of cylinder+volume of cone
1 2
= πr2h +3πr2h′
=376.8m3
OR 2
1 2
Volume of a right circular cone = 3πr h=144000 π cm3
Given that , height of a right circular cylinder = 180 cm
And radius of a right circular cylinder = Radius of a right circular cone
= 60 cm
∴ Volume of a right circular cylinder =πr2×h
2
=π×60×60×180
=648000 π cm3
So, volume of a right circular cylinder = Total volume of water in a cylinder
=648000π cm3
From point (iii)
Volume of water left in the cylinder
= total volume of water in a cylinder – Volume of water fell
2
=648000π−144000π
1
=504000π=504000×22/7=1584000 cm3
=1.584 m3 (1m3= 106cm3)
Hence, the required volume of water left in the cylinder is 1.584 m3

165
35 Marks 𝑥𝑖 𝑓𝑖 𝑓𝑖 𝑥𝑖
0-20 10 5 50 3
20-40 30 8 240
40-60 50 X 50x
60-80 70 12 840
80-100 90 7 630
100-120 110 8 880
∑𝑓 𝑥 1
Mean= ∑ 𝑓𝑖 𝑖
𝑖
2640+50𝑥
62.8 = 40+𝑥
1
X = 10
36 (v) Yes, d= -3,000
(vi) 20 1+1+2
(vii) 10 or Rs 8,10,000
37 (i) A(1,7) and B(4,2)
(ii) AC= √34
3 11
(iii) (-2, - 2 ) 1+1+2
OR
BC=√68> AC= √34 so Aditya travelled more
38 (i) Angle increases.
(ii) Distance decreases 1+1+2
(iii) H= 3/√2

166

You might also like